HEENT- PANCE

Réussis tes devoirs et examens dès maintenant avec Quizwiz!

vitreous hemorrhage

bleeding into vitrea; can be caused by retinal tear, macular degeneration, straining, etc.; causes sudden loss of vision & floaters; tx w/ surgery

ciprodex

combination of ciprofloxacin hydrochloride & dexamethasone; used to treat otitis externa; okay to use with TM perforation

congenital nasolacrimal duct obstruction

common in infants; blocked tear duct

optic neuritis

common in multiple sclerosis sudden loss in 1 eye MRI --> demyelination Tx: methylprednisone IV and neuro consult

What finding would help to differentiate between labyrinthitis and vestibular neuronitis?

Labyrinthitis commonly has hearing loss - Vestibular neuronitis does not have hearing loss

treatment for otitis media

amoxicillin macrolide augmentin

What is the first line antibx for OM?

amoxicillin, treatment with cephalosporin, bactrim or azithromycin may be acceptable in resistant cases, cefaclor or augmentin may be beneficial

blowout fracture

-sx- decreased visual acuity, diplopia with upward gaze, orbital emphysema, ENOPHTHALAMOS -dx- CT scan -tx- initial- nasal decongestants, avoid blowing nose, predisone---Opthlo Referal!!!

It's rare, but these two bugs can cause bacterial conjuncitvitis. Normal bugs are (strep pneumo, staph aureus, haemophilus aegyptius and moraxella)

chlamydia tachomatis and neisseira gonorrheae

blepharitis

chronic bilateral inflammatory condition of lid margins; presents with eye irritation, crusting, scaling, red-rimming of eyelid, eyelash flaking, etc

blepharitis

chronic inflammation of lid margins

8

chronic sinusitis is a sinusitis that lasts >___ weeks

describe open angle glauccoma

chronic, asymptomatic, and potentially blinding disease due to increased intraocular pressure

chalazion

chronic, nontender inflammation of a Meibomian gland;

What are RFs for developing sinusitis?

cigarette smoke or exposure to secondary smoke, hx of trauma and presence of a foreign body

treatment for otitis externa

cipro drops; if the external auditory canal is edematous use an ear wick and removve in 2-3 days

sx of allergic conjunctivitis

cobblestoning, stringy discharge

triamcinolone

common topical steroid used to treat aphthous ulcers

S&S of acute OM

ear pain, decreased hearing

visual acuity

how clearly the patient sees

This condition can occur with inadequate treatment of OM. Signs and symptoms are spiking fever and post auricular pain, erythema and fluctuant painful mass.

mastoiditis

posterior ischemic optic neuropathy

neuropathy involving the retrobulbar optic nerve; doesnt cause any optic disk swelling; can be caused by severe blood loss, nonocular surgery or dialysis

Your diabetic pt has venous dilation, microaneurysyms, retinal hemorrhages, retinal edema, hard exudates is this a sign of non proliferative or proliferative diabetic retionopathy

non

tx of macular dengeration

none laser tx or monoclonal AB (anti-VEGF) injections may slow progression

lymphadenopathy

normal lymph nodes <1cm in length; can be mono (posterior) or strep (anterior); treat underlying condition

early SCC

raised, firm pearly or verrucous plaque in the mouth

sx of barotrauma to ear

slight hearing loss, ear pain, dizzy, feeling of fullness in ear

lymphadenopathy

solitary lymph node > 1 month? consider lymphoma

common orgs in dacryocystitis

staph aureus, beta-hemolytic strep, h flu

preseptal orgs

staph aureus, staph epi, strep sp.

postseptal orgs

staph aureus, strep pneumo, anaerobes

Pterygium

starts from nasal conjunctiva triangular surgery

What is usu. the bacterial etiology of sinusitis? (bugs)

step pneumo, h. influenzae and less often s. aureus

enophthalmos

sunken eyes

sx of macular dengeration

drusen deposits, loss of nutritional supply, atrophy, neovascularization, gradual vision loss, metamorphopsia (wavy vision), mottling and hemorrhages on retina

trachoma

effect from chlamydial conjunctivitis; #1 cause of infectious blindness worldwide

what defines chronic otitis media

effusion greater than 3 months and refractory to antibiotics. 3 or more infections in 6 months 4 or more infections in one year

pterygium

elevated, superficial, fleshy, triangular-shaped "growing" fibrovascular mass - MC in inner corner/ nasal side of eye and extends laterally - thickening of bulbar conjunctiva - associated with increased UV exposure in sunny climates as well as sand, wind, dust - observation in most cases (artificial tears) - removal if growth affects vision

cause of Retinal artery occlusion

emboli, THROMBOTIC event, vasculitis

What a serious consequence of untreated otitis media in the pediatric patient? (conjunctivitis, meningitis, mastoiditis, sepsis)

mastoiditis

Septum, turbinate, and floor

mucosa is pink and moist without lesions

Patency

nasal passageways are patent

Neck motion

neck is supple

External Eye

no exophthalmos, ptosis, periorbital discoloration, edema or skin lesions

tx of Retinal artery occlusion

ohpthalmologist-->vessel dilation and paracentesis workup and manage atherosclerotic ds

erythromycin, bacitracin

ointment used to treat hordeolum, acute anterior blephraitis, corneal abrasion & bacterial conjunctivitis

clinical features of a blowout fracture

swelling and misalignment. movement restricted especially inability to look up due to entrapment of infraorbital nerve. double vision and sub-q emphysema

labyrinthitis

swelling of inner ear from infection, autoimmune

Etiology of apthous ulcers

Idiopathic or herpes

Risk factors of Macular degeneration?

Age Smoking CAD Fam history

Retinal artery occlusion

Amaurosis fugax. Pale retina

Small, mildly painful nodule within a gland on the eyelid

Hordeolum

preseptal cellulitis associated with what illness

URI

cyclosporin (restasis)

immunosuppressant treatment for keratoconjunctivitis

the inflammation of what gland is involved with a chalazion

meibomian gland

What is the final step in Glaucoma work-up?

1. Cup:disc ratio > 0.5 2. Tonometry > 21mmHg 3. Peripheral field testing

Painless nodule of the eyelid?

Chalazion

Laryngitis

typically viral and follows URI

A white plaque-like oral lesion which does scrape off?

Oral candidiasis

ocular steroids

- increase IOP, cataracts, scleral thinning and corneal metling - predisnolone

for conjunctitvis must rule out

- ueitis, acute glaucoma, corneal disorders

chalazion

PAINLESS, indurated lesion, redness tx warm compress, I&D if not resolved

entropion

turns in

Retinal detachment:

"FLOATERS" or flashes of light. field loss. curtain.

herpes simplex tx

- oral acyclovir - topical antivirals - topical steroids may worsen disease

Acute OM -1st line Abx

-Amoxicillin

Complication of OM

-Mastoiditis

A patient has a 2-day history of an itchy red left eye and marked tearing. There is no history of injury. On physical exam the conjunctiva appear markedly erythemic. What physical finding would help most in differentiating this as a viral conjunctivitis?

A palpable pre-auricular lymph node is most often seen with viral conjunctivitis and rarely seen in bacterial conjunctivitis.

Most Conjunctivitis is likely caused by which type of microorganism?

A virus (adenovirus, herpes zoster), which is *very* contagious!

What type of antibiotic (delivery method) should be used for corneal abrasions/ulcers?

Abx Ointment

tx of auricular hematoma

All should be drained ASAP after injury >7 days refer to an otolaryngologist or plastic surgeon for debridement of new perichondrial growth and any remaining hematoma.

preseptal cellulitis tx

Amoxicillin/Clavulanic Acid or 1st gen ceph

Which type of Glaucoma is an *acute* issue?

Angle-closure

Treatment for brachial cleft cyst?

Antibiotics for acute infection; surgical excision for definitive treatment

Cold Thyroid Nodule

Associated with thyroid cancer

Treatment for acute labyrinthitis?

Bed rest hydration diazepam and short course steroids if needed

An unvaccinated 6 year old male presents with bilateral parotid swelling and pain with eating. What is the most likely pathogen causing his condition?

Paramyxovirus

Acute onset of copious purulent discharge from both eyes

Bacterial Conjunctivitis clinical features

If you suspect mastoiditis after an OM infection what can help you with the dx?

CT

What is imaging test done for orbital cellulitis?

CT

dx of cavernous sinus thrombosis

CT, MRI or MR venogram CBC blood cultures and sinus cultures

How will retinal artery occlusion appear on fundoscopic exam?

Cherry red spot on macula

Mucopurulent discharge with a marked follicular response on inner lids (bacterial conjunctivitis)

Chlamydia bacterial conjunctivitis

Painful eye and loss of vision Anterior chamber narrowed Intraocular pressure elevated EMERGENCY

Closed-Angle Glaucoma clinical features

IV carbonic anhydrase inhibitor (Acetazolamide) Topical BB Osmotic diuresis (mannitol)

Closed-Angle Glaucoma treatment

What is a Cataract?

Clouding of the lens

"Pink eye" is aka...

Conjunctivitis

18

Come closer - look at all 4 quadrants - brace with hand

What is the treatment for a pt undergoing Retinal artery occlusion?

Considered a form of stroke - Stroke protocol must be taken

peritonsillar abcess

MC- S. virdians -sx- hot potato voice, uvular devated to contrlateral side -dx- CT -tx- I and D. clindamycin

Strabismus

Correct by 4 months. Cover/uncover test

Sinutitis

MC= maxillary. chronic= staph A. MC, mucurmycosis in pts with DM -sx- cocurrent rhinitis that follows URI. sinus tenderness on palpation. 1-4 weeks -dx- CT scan. Xray= waters view -tx- amoxicillin 10-14 days, intranasal glucocorticoids (nasacort)

What is the treatment for a ptergyium?

Excision if it interferes w/ vision, recurrence is common

What is the preferred treatment for a thyroglossal duct cyst?

Excision of cyst and tract (after acute infection resolves)

What is the first line treatment for bacterial sinusitis?

First line empiric treatment: Amoxicillin-clavulanic acid - Consider doxycycline, levofloxacin or moxifloxacin

Introduction

I am PA-S Rock star

5 - EARS

Inspect - external ears - mastoid process

Allergic Conjunctivitis

Itch, bilateral eye condition Tx with oral antihistamines.

Macula

Look into the light: macula is without lesions

sinusitis

MCC is VIRAL Dx: plain film Water's view Gold Std = CT SCAN adult = MAXILLARY pediatric = frontal

What is contraindicated for hyphema?

NSAIDS - increased bleeding

What is the condition of acute inflammation & demyelination of the optic nerve?

Optic Neuritis

Ptosis, eyelid edema, exophthalmos Purulent discharge, conjunctivitis EMERGENCY

Orbital Cellulitis clinical features

Organism which causes malignant otitis externa?

Pseudomonas

Most common type of laryngeal cancer?

Squamous cell

Findings seen on lateral neck X-ray in patient with epiglottitis?

Thumbprint sign

Name BB's used for OA Glaucoma tx

Timolol Betaxolol

Other tx for acute angle closure glaucoma?

Timolol Pilocarpine Apraclonidine Laser iridotomy

Which of the following examinations is a major component of routine monitoring of chronic, open angle glaucoma?

Tonometry, gonioscopy, monitoring of the disc-to-cup ratio, and visual field examination are the routine exams done when monitoring primary open angle glaucoma.

Treatment for otitis externa?

Topical antibiotic drops to affected ear (fluoroquinolone or other covering pseudomonas)

Lesion of the RIGHT optic nerve will cause what visual symptoms?

Total blindness of the right eye.

Patients who have nasal packing placed to control epistaxis must a have close follow up to remove packing in order to decrease the chance of what complication?

Toxic Shock Syndrome

What are the TORCH infections?

Toxoplasmosis Other (Syphilis) Rubella Cytomegalovirus Herpes Simplex Virus

Pharyngitis

Treatment of this condition with ABX is intended to prevent rheumatic myocarditis, scarlet fever, local abscess formation, and glomerulonephritis. May present with a sore throat, odynophagia, fever, anterior cervical adenopathy, tonsillar exudates, scarlatiniform rash. Dx with rapid antigen testing and culture. CBC may show leukocytosis. Most commonly caused by Group A Beta Hemolytic Strep (Strep pyogenes). Other causes may be N gonorrhea, mycoplasma, Chlamydia, corynebacterium, diphtheria, clostridium. Tx with Pen VK for 10 days. Erythromycin for PCN allergic pts. Supportive treatment with analgesics, NSAIDs, salt water gargling, and anesthetic lozenges.

T/F: Cataracts tend to be *bilateral*?

True

Tx of Hordeolum?

Warm Compress Oral abx c/ staph coverage only if local cellulitis

Hordeolum

Warm compress

bacterial keratitis

bacterial infection of the cornea; diagnosed with a hazy cornea with a central ulcer & adjacent stromal abscess; treated wtih topical fluoroquinolones

How do you treat a hordeolum (stye)?

Warm compress several times a day for 48 hrs, topical antibiotics if secondary infection develops Incision and drainage may be indicated if it does not resolve

What's the tx for a Chalazion?

Warm compresses, possibly incision & curettage

What MUST be considered when a hyphema occurs but no mention of trauma?

Underlying blood dyscrasia or coagulopathy

Dacroadenitis may be caused by stone, debris or dacryostenosis. Dacrostenosis may be congenital malformation and the duct does not open in the first year of life. What is tx in that case?

Usually self-resolving within first year of life.

15

Visual acuity (Snellen chart)

water's view

X-ray view to diagnose acute sinusitis

treatment for macular degeneration

anti-VEGF

nasal polyps

benign growths in the nose; unknown cause; seen with allergic rhinitis; causes chronic nasal obstruction, snoring & post-nasal drip; treated with intranasal or oral corticosteroids, polypectomy

vocal fold paralysis

can be caused by thyroid/neck surgery, or tumors; causes breathy dysphonia & effortful speech; treated with speech therapy

SCC of larynx

cancer of the larynx almost exclusively seen in smokers; causes change in voice quality

aphthous ulcers

canker sores; causes single/multiple, painful, small, round ulcers; treated with topical analgesics or steroids

acetazolamide

carbonic anhydrase inhibitor used to treat acute angle-closure glaucoma & retinal artery occlusion

slow loss of vision, contrast sensitivity, glare, halo around lights. what dx

cataract

hematoma of external ear

cauliflower ear must drain blood and pressure to prevent deformity boxers, trauma

infectious mononucleosis

caused by EBV; causes severe tonsillitis, splenomegaly/hepatomegaly, fatigue, posterior lymphadenopathy, etc; diagnosed with monospot; treated with supportive care, corticosteroids, pain relievers

fungal sinusitis

caused by aspergillus; almost always seen in immunocompromised patients; symptoms similar to bacterial rhinosinusitis; treated with surgical debridement & amphotericin B IV

chronic otitis media

complication of acute otitis media; caused by p. aeruginosa, proteus, & staph aureus; causes purulent aural drainage & conductive hearing loss; treated with removal, earplugs, & antibiotics

herpes zoster opthalmicus

complication of herpes zoster on the face that affects ophthalmic division of trigeminal nerve; treated with acyclovir, valcyclovir, famcyclovir

Peritonsillar abscess

complication of strep throat

artificial tears

eye drops used to treat keratoconjunctivitis sicca & pterygium

tonal tinnitus

hearing "tone" sounds

uveitis

intraocular inflammation

macular degeneration

leading cause of permanent vision loss in older population; presents with graudal bilateral blurred or loss of central vision, scotomas, micropsia, etc; treated with vitamin supplements, laser coagulation & intra-vitreal injections

sialogogues

lemon drops; used to treat sialadenitis & sialolithiasis

dacryocystitis tx

lid hygiene topical bacitracin and erythromycin OR systemic if severe with tetracycline

MCC dental abscess

periapical abscess, and the second most common is a periodontal abscess

sx of orbital vs periorbital cellulitis

periorbital: NONTENDER EOM, normal visual acuity orbital: TENDER EOM, impaired visual acuity, proptosis, OPTHALMOPLEGIA -hx of sinusitus, strep pneumo

acute angle closure glaucoma

peripheral vision loss

Leukoplakia

precancerous white oral plaque

presbyopia

the natural loss of accommodative capacity with age (starts ~45); corrected with plus lenses

Trachea positon

trachea is midline and mobile

causes of hyphema

trauma eye cancer DM sickle cell

consists of bogginess of the nasal mucosa associated with a complaint of stuffiness and rinorrhea. symptoms are labile and can clear quickly

vasomotor rhinits

AV nicking

venous compression at artery-venous junction by increasing arterial pressure; may lead to central retinal vein occlusion; seen with hypertensive retinopathy

true or false: ectropion and entropion are common in older people

true

wet vs dry macular dengeration

wet: new blood vessels grow under macula, leak dry: light sensitive cells in macula slowly break down, straight lines appear crooked

If a patient is having a POSTERIOR nasal bleed, what is the usu. location?

woodruff plexus anterior would be keissenbachs plexus

ectropion

turns out

treatment for chronic otitis media

tympanostomy tubes

traction

type of retinal detachment; adhesions separate retina from its base

tx of papilledema

underlying cause

How would Fundoscopic exam show with retinal vein occlusion?

"Blood and thunder retina" Superficial retinal hemorrhages

Otitis Externa

"Swimmers Ear" Caused by gram-negative rods (Pseudomonas, Proteus), or fungi (Aspergillus). May present with a history of water exposure or mechanical trauma. Painful erythema and edema or external auditory canal. Purulent exudate and pruritus. May evolve into osteomyelitis of skull base (malignant variant) in diabetic or immunocompramised patients (think pseudomonas). Dx with CT and radionuclide scanning. Prevent exposure to moisture or trauma. Remove purulent debris. Tx with drying agent, otic abx drops (aminoglycoside or quinolone with or without corticosteroid). Treat malignant variant with prolonged IV or oral antipseudomonal abx.

orbital floor fracture

"blowout fracture"; fracture of the orbital floor due to trauma; presents with decreased visual acuity, enophthalmos, diplopia with upward gaze & orbital emphysema; treated with nasal decongestants, oral antibiotics, ice packs & surgical repair

follow up for corneal abrasion

- 48 hours and avoid contacts for 1 week after healing

cataract locations and tx

- cortical, nuclear, posterior subcapsular - surgery - improvement in 95% of patients

blowout fracture

- direct trauma to zygomatic prominence or to soft tissue of the orbit - pressure blows out weak orbital floor - double vision: entrapment of inferior rectus or inferior oblique - limited upward gaze - infraorbital anesthesia - diagnosis: plain films and CT scan - tx: surgery may be required

corneal abrasion

- superficial irregularities of the cornea - caused by foreign bodies - injury, welders arc, contact lens - pain, PHOTOPHOBIA, redness, blurry vision, small pupil - check visual acuity first - flourescein stain - stained abrasion with cobalt filter

Resistant Acute OM -Abx

-Amox-clavulanate (Augmentin)

*growth of keratinizing aquamous epithelium of middle/mastoid process

-Cholesteatoma

Otitis Externa x3 MC pathogens

-Pseudomonas aeruginosa -Proteus -Fungi

Systemic sclerosis

-sx= tight, shiny, thickened skin. Crest syndrome= calinosis cutis, raynouds, claw hand (Sclerodactyly) sclerosis- trunk and proximal extremities -dx- + anti-centromere Ab, * Anti-SCL -tx- dmards, raynouds- CCBs

do not patch for more than

24 hours

Metallic foreign bodies may cause which finding in the eye?

A rust-colored discoloration at the point-of-entry

Unilateral, progressive hearing loss Impaired speech discrimination Continuous Vertigo

Acoustic Neuroma clinical features

Amoxicillin

Acute Otitis Media treatment

Demographically speaking, in whom is glaucoma *most common*?

African american pts >40 yrs who have a + Family Hx

Name the risk factors of developing Cataracts

Age Smoking ETOH Chronic systemic steroid use Sunlight exposure DM Metabolic syndrome Congenital TORCH infections

Which of the following is the third component of the atopic triad, besides allergic rhinitis and asthma?

Atopic dermatits, or eczema, is the third chronic finding, along with asthma and allergic rhinitis, in patients who are atopic. Urticaria are common in acute and chronic allergies.

Recurrent episodes of vertigo exacerbated by head movements?

BPPV (Benign Paroxysmal positional vertigo)

A 25 year old female has dizziness and a positive Dix-Hallpike. Her initial work up is negative. What is the most likely diagnosis?

Benign Paroxysmal Positional Vertigo

Who is at increased risk of developing Corneal Abrasions?

Contact lens users

Most common cause of blindness from retinal disease is a result of having...?

Diabetes

Which medical condition predisposes patient to malignant otitis externa?

Diabetes

Blindness in adults

Diabetic Retinopathy is the leading cause of...

Common complaints with Macular degeneration?

Difficulty driving & reading

Pharyngitis with progressive growth of grey-white membrane?

Diphtheria

Mono

EBV. maybe CMV fever/ cervical LAD/ pharyngitis

sx of blow out fracture

Enophthalmos (the eye is receded into the orbit) Orbital dystopia (the eye on the affected side is lower in the horizontal plane than the other) A significant consequence of fractures of the orbital floor is entrapment of the inferior rectus muscle and/or orbital fat

Blow out fracture

Enopthalmos and Distopia. Periocular ecchymosis, diplopia, infraorbital nerve damage.

Lid and lashes turned in secondary to scar tissue or spasm of orbicularis oculi muscles

Entropion

History & Physical/ENT/Ophthalmology A patient presents with the complaint of irritation of the left eye one day after gardening. He states "I think there is something in my eye." Which of the following findings is consistent with your suspected diagnosis? A. increased intraocular pressure B. rust ring C. hazy cornea D. fluorescein uptake

Explanations (u) A. Elevated intraocular pressure is seen with glaucoma. (u) B. Rust ring is seen with metallic foreign bodies. (u) C. Hazy cornea is seen with glaucoma. (c) D. Fluorescein dye uptake is diagnostic for corneal abrasion.

Inflammation and infection of the Moll or Zeis glands

External Hordeolum

Tx of hyphema?

Eye protection rest HOB at 30 degrees Beta adrenergic blockers or Carbonic anhydrase inhibitors

T or F. Coryza, hoarseness and cough are suggestive of strep pharyngits.

FALSE

T/F: Topical anesthetics are recommended for tx'ing Corneal Abrasions.

FALSE!! Do *NOT* give topical anesthetics for a Corneal Abrasion!!

What gastrointestinal disorder can lead to laryngitis if not treated?

GERD can cause laryngitis as acid may burn the laryngeal structures at night

Use of which of the following medications can result in hearing loss?

Gentamycin is an aminoglycoside, and can cause ototoxicity. Peak and trough levels must be drawn to determine the lowest effective dose. The remaining medications do not interfere with vestibular function.

Which multiple endocrine neoplasms (MEN) have medullary thyroid cancer?

MEN 2a and MEN 2b

Low sodium diet Diuretics (acetazolamide)

Meniere Disease treatment

S pneumoniae

Most common bacterial pathogen of Acute Sinusitis

Viral

Most common pathogen of Acute Pharyngitis

Retinal Vein Occlusion

Most commonly occurs secondary to a thrombosis event. May present as sudden, painless, unilateral blurring of vision, or complete vision loss. Afferent papillary defect on exam with "BLOOD AND THUNDER" retina (dilated veins, hemorrhages, edema and exudates). Vision typical resolves with time, at least partially. Thrombotic workup is warranted. 2 types: BRVO and CRVO

Known as "swimmer's ear"

Otitis Externa

Ear pain (esp with movement of tragus, auricle) Redness, swelling of ear canal Purulent exudate Associated with water exposure

Otitis Externa clinical features

Antibiotic drops - fluoroquinolone, aminoglycosides

Otitis Externa treatment

A 65-year-old male presents to you with complaints of decreasing hearing, along with difficulty discerning words when in conversations in noisy environments, such as restaurants. His only medication is simvistatin for hyperlipidemia. The following is his audiogram. He has bilateral decreased high frequency hearing loss, and decreased speech recognition. What is the most likely diagnosis?

Presbycusis is age related bilateral loss of high frequency hearing, and decreased word recognition. Presbystasis is age related balance disorder. Vestibular schwannoma (acoustic neuroma) causes unilateral hearing loss. Vestibulobasilar insufficiency results from atherosclerosis of the vertebral arteries, and can cause many symptoms including double vision, speech defects, vertigo, ataxia, and drop attacks.

Cimetidine

Recurrent Aphthous Ulcers treatment

Acute onset of painless blurred/blackened vision "curtain sign"

Retinal Detachment clinical features

Retinal artery occlusion

SUDDEN ONSET vision loss in 1 eye (monocular vision loss = amaurosis fugax)

List two infectious diseases which may cause hearing loss?

Syphilis - Lyme disease

Which pharmacologic agents have phototoxicity as a side effect? A. Tetracycline, penicillin, and metoprolol B. Penicillin, ketoconazole, and metrogel C. Doxycycline, hydrochlorothiazide, and naproxen D. Cephalexin, acetaminophen, and metoprolol

The answer is C. EXPLANATION: A phototoxic reaction is one that results from exposure to the drug and exposure to UV light. The reaction occurs in exposed sites.

A 73-year-old female with type 2 diabetes, hypertension, and hyperlipidemia presents to the outpatient clinic complaining of left ear pain, and a yellowish-green, foul-smelling discharge that began about 3 weeks ago. On physical examination, the patient is afebrile and examination reveals a markedly edematous left ear canal draining purulent, green discharge. The tympanic membrane is unable to be visualized. Which of the following is the most likely causative agent for this patient's diagnosis? A. Escherichia coli B. Moraxella catarrhalis C. Pseudomonas aeruginosa D. Staphylococcus aureus

The answer is C. EXPLANATION: Malignant otitis externa is most commonly caused by pseudomonas (C). E coli (A) and S aureus (D) are less common causes of otitis externa, while S pneumoniae (E) and M catarrhalis (B) are common etiologies of acute otitis media.

Which physical exam finding differentiatesacne rosacea from acne vulgaris?

The characteristic lesions of acne rosacea are small papules,papulopustules, and telangiectasias with flushing. There are nocomedones present in acne rosacea.

An 8-year-old male with a history of atopicdermatitis presents with a localized rash, consisting of vesiclesand eroded lesions. He has a low-grade fever, but no other symptoms. Whatis the appropriate treatment?

The classic lesion of eczema herpeticum is described as a "punchedout" lesion, which refers to vesicles that have becomeeroded. Mild cases of eczema herpeticum can be treated on an outpatientbasis with oral acyclovir. More severe cases must be treated onan inpatient basis with IV acyclovir and oral antibiotics if superinfected.

A 54 year old male presents with dark thickened skin and soft pedunculated papules around his neck. He states that the lesions are asymptomatic. What disease process are the findings commonly associated with?

The thick dark plaque around the patient's neck is acanthosis nigricans. The papules are acrochordons (or skin tags). Both conditions are associated with metabolic syndrome. (Wolff & Johnson, p231)

A 46-year-old male comes into the emergency department complaining of severe left eye pain after a champagne cork hit his eye while trying to open a bottle on his honeymoon night. Visual acuity to the affected eye is limited to identifying finger movements only. A physical exam reveals the following findings. What is your diagnosis?

This image demonstrates a traumatic hyphema. There is an air-fluid level of blood in the anterior chamber of the eye. Treatment includes elevation of the head and dilation of the pupil. A corneal abrasion would demonstrate fluorescein staining of the cornea. A hypopyon is a collection of wbc's, or pus in the anterior chamber. A retinal detachment would only be visualized in the posterior eye with a direct ophthalmoscope.

Foreign Body in Auditory Canal

This might be where you find Barbie's missing shoe. Frequent in children. Remove material with hook or loop. Irrigation may be attempted unless material is organic (beans, insects) due to potential for item to swell. Immobilize insects with lidocaine before removal.

Gold standard to diagnose bacterial pharyngitis?

Throat culture

A 7 year old male presents with a soft, mid-line neck mass which rises with protrusion of his tongue. What is the most likely diagnosis?

Thyroglossal duct cyst

What condition must be ruled out in a patient who presents with vertigo and syncope?

Vertigo + syncope = vertebral basilar insufficiency until proven otherwise

When would Cobblestoning of palpebral conjunctiva occur?

Viral conjunctivitis - Recent URI? (Adenovirus most common) Allergic conjunctivitis - Other allergic triggers?

Most common site for posterior nose bleeds?

Woodruff's plexus

nystagmus

a rhythmic oscillating movement of the eyes & may be vertical or horizontal in nature; common with vertigo

viral conjunct common org

adenovirus 3

chronic OM tx

cipro abx drops

dx of cholesteatoma

clinical CT to check bony defects

neonatal conjunctivitis

conjunctival inflammation occurring within the first 30 days of life

photopsia

flashing lights

conductive hearing loss

otosclerosis, cerumen

hordeolum

painful. MCC staph

if you see a rust ring on when examining for foreign body of eye what is the treament

remove with rotating burr or refer to ophthalmologist

sx of papilledema

swollen disc blurred margins obliterated vessels asx or transient visual alterations

nasal polyps

sx- allergies, asprin allergy causing severe bronchospasms TRIAD- polys, allergies, asthma -dx- MUST R/O CYSTIC FIBROSIS -tx- 3 month course of topical corticosteroids

Gout/ pseudogout

sx- gout- 1st MTP. pseudogout- Knee. -dx- gout- rod shaped neg, birefinged urate crystals. mouse bite xray pseudogout- rhomoid shaped crystals. chondracacinosis xray -tx- NSAIDS ( indomethcin). pseudo- costicosteroid injections.

papilldema

sx- swelling of optic disk due to increase intracranial pressure. swollen optic disk, blurred disk cup margins****** -dx-MRI/CT to rule out head mass. LP (CSF increased pressure)*** -tx- underlying disorder. diruetics= acetazolamide

Hair and scalp inspection

symmetrical hair distribution, hair and scalp are without infestation, scaling, and lesions

Cranial nerve 12

tongue is midline with protrusion

physical exam for cataract

translucent yellow discoloration of lens. appears black on a red background on fundoscopy

causes of epistaxis

trauma, dry mucosa, coagulopathy

decongestants

treatment for ET dysfunction, barotrauma

30

treatment for chemical burns of the eye includes immediate irrigation for _____ mins or at least 2L of NSS

chemical labyrinthectomy

treatment for meniere syndrome; causes destruction of vestibular apparatus while retaining hearing

complete labyrinthectomy

treatment for meniere syndrome; results in complete loss of hearing

cyclopentolate

treatment for ultraviolet keratitis

vincent's angina

trench mouth

BPPV tx

watch and see 2 months eppley maneuver

test to help differentiate bettween conductive and sensorineural hearing loss

weber test

ectropion

- advanced age, trauma, infection, or palsy of the facial nerve

dry eye treatments

- artificial tears - lubricating eye ointment - restasis (cyclosporine)

chalazion

- painless granuloma of the internal meibomian sebaceous gland -sx- hard nontenfer eye swelling -tx- eyelid hyigene

allergic conjunctivits

- seasonal - patients with atopy (asthma, eczema) - hayfever - pruritis (severe) - PE: injection, mucoid discharge tx: topical vasoconstrictors or antihistamines - topical mast cell stabilizer

entropion

- secondary to scar tissue or spasm of the orbicularis oculi muscles

vernal conjunctivitis

- seen in teens/young adults - associated with systemic atopy - mainly seen in the spring - cobblestone apperance - tx: topical steroids

carpal tunnel syndrome

-sx- parasthesias and pain of 1st 3rd digits esp thenar muscle wasting -dx- tinels sign-percussion of median nerve proceduces symptoms phalens sign- flex both wrists for 30-60 sec reproduces symptoms -tx- volar splint

Achilles tendon rupture

-sx- sudden heel pain with push- off movement, pop, sudden, sharp calf pain, -dx- +thompsons test- weak absent plantar flexion -tx- surgical management

the end

...

age of epiglottitis

2-7 years or 45-65 years

Prevalence of age related macular degeneration increases after what age

50

What is the most common primary source for metastasis to the lymph nodes of the neck?

85% of metastasis to the lymph nodes of the neck originates from primary tumors of the aerodigestive tract

Acoustic Neuroma

8th Cranial nerve. Benign tumor. Sensorineural hearing loss.

What is a Pterygium?

A growth that starts from the nasal conjunctiva and grows toward and into the cornea.

You are treating a 20-year-old female with multiple aphthous ulcers. She complains of a moderate amount of pain. You decide to prescribe "magic mouthwash" for the patient to swish and spit. Which of the following combinations of medicines is appropriate?

A very commonly used combination of medicines to promote relief of discomfort and healing include liquid diphenhydramine, antacid, tetracycline, and 2% viscous xylocaine.

A 45-year-old male who presents to the emergency department with sudden onset of lip swelling, which began shortly after awakening this morning. He denies any history of allergies and denies any new medications. His current medications include hydrochlorothiazide (HCTZ), captopril, atenolol, atorvastatin, and fexofenadine. What is the most likely cause of this gentleman's symptoms?

ACE inhibitor angioedema is a potentially life-threatening known side effect of captopril. The patient may need emergency intubation to ensure that his airway remains patent. Patients with a history of ACE inhibitor angioedema should not be placed on any ACE inhibitor or an ARB due to the possibility of similar reaction. HCTZ (a diuretic) and atenolol (a beta blocker) are antihypertensives that are generally unlikely to be associated with angioedema. Atorvastatin (a statin) and Fexofenadine (an antihistamine) are unlikely to present with angioedema.

What is the Dx? Recurrent nasal congestion associated with pale/bluish boggy turbinates and allergic shiners.

Allergic Rhinitis

Most thyroid cancer is successfully treated and has a very good prognosis. What type of thyroid cancer is aggressive and has a poor prognosis?

Anaplastic thyroid cancer

Worst and most aggressive type of thyroid cancer?

Anaplastic thyroid cancer

What is the typical cause of Hyphema?

Blunt or penetrating trauma

Imaging for blowout fx?

CT - emergent surgery

Macular Degeneration

Central vision loss. >65. Smoking. Amsler grid. Drusen. AREDS 11 formulation

Chalazion

Chronic painless nodule

Pain, sensation of a foreign body Photophobia Tearing Injection Blepharospasm

Corneal Abrasion clinical features

malignant otitis externa

DM pseudomonas

causes of retinal detachment

DM, injury, age

A 5 year old female presents with large insect which is still moving in his ear. What must be done prior to further action?

Drown the insect before removal

Which (Dry or Wet) Mac Degen presents with Drusen on PE?

Drusen - yellow retinal deposits Found on dry Mac Degen

Extra ocular movements, and near reaction

EOM's are full and equal, eyes converge equally and pupils constrict with near focus

eustachian tube dysfunction

ET swelling inhibits the ability to auto-insufflate leading to negative pressure; often follows URI or allergic rhinitis; presents with ear fullness, popping of ears, intermittent sharp ear pain, disequilibrium, etc; treated with decongestants

A 6 year old male with chronic sinusitis presents for ENT evaluation. Enlargement of what structures is likely to be found?

Enlarged adenoids

Ceftizoxime and dexamethasone

Epiglottitis treatment

A 2-day-old infant presents with numerousred macules with central vesicles and pustules. The rash sparesonly the palms and soles. The infant has no fever and is nursingnormally. What is the most likely diagnosis?

Erythema toxicum is a benign rash seen in newborns. The causeof the rash is unknown and resolves spontaneously. The rash appearsas erythematous macules, which may develop central vesicles within24 to 48 hours. The palms and soles are spared.

History & Physical/ENT/Ophthalmology A 4 year-old child presents with a rapid onset of high fever and extremely sore throat. Which of the following findings are suggestive of the diagnosis of epiglottitis? Answers A. Croupy cough and drooling B. Thick gray, adherent exudate C. Beefy red uvula, palatal petechiae, white exudate D. Inflammation and medial protrusion of one tonsil

Explanations (c) A. A croupy cough with drooling in a patient who appears very ill is consistent with epiglottitis. Examining the throat is contraindicated, unless the airway can be maintained. (u) B. Thick gray adherent exudate is suggestive of diphtheria. (u) C. Beefy red uvula, palatal petechiae, and white exudate are findings suggestive of streptococcal pharyngitis. (u) D. Inflammation with medial protrusion of the tonsil is suggestive of a peritonsillar abscess.

Clinical Therapeutics/ENT/Ophthalmology A patient with Type 1 diabetes mellitus was treated for otitis externa of the right ear for 2 weeks with topical ear drops. The patient presents today with persistent, foul aural discharge, granulations in the ear canal, and deep ear pain. Which of the following is the proper treatment at this time? A. ciprofloxacin (Cipro) IV B. cefuroxime (Zinacef) IV C. ampicillin-sulbactam (Unasyn) PO D. azithromycin (Zithromax) PO

Explanations (c) A. IV antibiotics directed against Pseudomonas, the most likely etiology, is needed for the treatment of malignant otitis media. (u) B. Cefuroxime, ampicillin-sulbactam, and azithromycin have no activity against Pseudomonas. (u) C. See B for explanation. (u) D. See B for explanation.

Diagnosis/ENT/Ophthalmology A 58 year-old patient presents with spells of dizziness which is described as a spinning sensation. This has occurred several times a day for the last month. The patient also complains of some mild hearing loss, fullness, and a blowing sound in the right ear. Which of the following is the most likely diagnosis? A. Meniere's syndrome B. Labyrinthitis C. Benign paroxysmal positioning vertigo D. Vestibular neuronitis

Explanations (c) A. The classic findings of Meniere's syndrome consists of episodic vertigo, with discrete vertigo spells lasting 20 minutes to several hours in association with fluctuating low-frequency sensorineural hearing loss, tinnitus, and a sensation of aural pressure. (u) B. Labyrinthitis is an acute onset of continuous, usually severe vertigo lasting several days to a week, accompanied by hearing loss and tinnitus. (u) C. Benign paroxysmal positioning vertigo is a type of vertigo associated with changes in head position, often rolling over in bed. (u) D. Vestibular neuronitis is a paroxysmal, usually single attack of vertigo that occurs without accompanying impairment of auditory function and will persist for several days to weeks before clearing.

Clinical Intervention/ENT/Ophthalmology A 10 year-old boy was playing with sparklers (magnesium sulfate) and got some of the "sparkle" in his right eye. Which of the following is the most appropriate initial treatment? A. irrigate the eye for at least 20 minutes B. apply Bacitracin ointment and patch the eye C. remove the sparkle with a moistened cotton swab D. protect the eye with a metal shield and refer to an eye ophthalmologist

Explanations (c) A. The magnesium from the sparkler combines with tears, producing an alkaline injury and should be treated with prolonged irrigation. Irrigation should be the first step in management of this case. (u) B. See A for explanation. (u) C. See A for explanation. (u) D. See A for explanation.

Scientific Concepts/ENT/Ophthalmology Bitemporal hemianopia is noted on physical examination in a patient with visual changes over the past 2 years. The central field of vision is spared. The lesion is located in the A. optic nerve. B. optic chiasm. C. temporal optic radiation. D. optic tract.

Explanations (u) A. A lesion in the optic nerve would result in loss of vision in the affected eye only and include loss of central vision. (c) B. A lesion in the optic chiasm would result in the loss of vision in the bilateral temporal fields and spare the central field of vision. (u) C. A lesion in the temporal optic radiation would produce superior contralateral quadrantopia. (u) D. A lesion in the optic tract would result in loss of vision in the temporal field of the ipsilateral eye.

History & Physical/ENT/Ophthalmology Which of the following is diagnosed by use of the cover/uncover test? Answers A. Adie's pupil B. Strabismus C. Glaucoma D. Myopia

Explanations (u) A. Adie's pupil is a sluggish pupil reaction to light and accommodation, evaluated by papillary reaction to light. (c) B. The cover/uncover test is used to diagnose strabismus. (u) C. Tonometry is used to measure intraocular pressure to evaluate for glaucoma. (u) D. Myopia is evaluated by using a Snellen chart.

History & Physical/ENT/Ophthalmology The most reliable sign of acute otitis media (AOM) is A. bulging of the tympanic membrane. B. loss of tympanic membrane mobility. C. reddening of the tympanic membrane. D. air bubbles behind the tympanic membrane.

Explanations (u) A. Bulging and air bubbles behind the TM represent OM with effusion. (c) B. Loss of tympanic membrane mobility during pneumoinsufflation is the most reliable sign for diagnosing acute otitis media. (u) C. Reddening of the eardrum is not reliable as it may be due to crying or other vascular changes. (u) D. See A for explanation.

Clinical Therapeutics/ENT/Ophthalmology Use of systemic corticosteroids can cause which of the following adverse effects in the eye? A. Cortical blindness B. Optic atrophy C. Glaucoma D. Papilledema

Explanations (u) A. Cortical blindness is a rare adverse effect when prescribing salicylates. (u) B. Optic atrophy can occur as an adverse effect with lead compounds, amebicides, and MAO inhibitors. (c) C. Glaucoma can be caused by the long-term use of steroids. (u) D. Papilledema can be a side effect to many systemic medications.

Scientific Concepts/ENT/Ophthalmology Which of the following does the macula provide? Answers A. Night vision B. Color vision C. Peripheral vision D. Central vision acuity

Explanations (u) A. Night vision is a function of rod photoreceptors, which are found in the peripheral retina. (u) B. Color vision is a function of cone photoreceptors. (u) C. The peripheral retina is responsible for peripheral vision. (c) D. The macula is responsible for central visual acuity.

Health Maintenance/ENT/Ophthalmology At what age does the first tooth usually erupt in an infant? A. 2-4 months B. 6-8 months C. 10-12 months D. 14-16 months

Explanations (u) A. See B for explanation. (c) B. The first tooth in an infant to erupt is the central incisor at the average age of 6-8 months. (u) C. See B for explanation. (u) D. See B for explanation.

Health Maintenance/ENT/Ophthalmology The most common cause of conductive hearing loss is Answers A. otosclerosis. B. cholesteatoma. C. impacted cerumen. D. chronic serous otitis media.

Explanations (u) A. See C for explanation. (u) B. See C for explanation. (c) C. The most common cause of conductive hearing loss is impacted cerumen. (u) D. See C for explanation.

Scientific Concepts/ENT/Ophthalmology Which of the following is the most likely organism in a 2 year-old child with acute otitis media? A. Staphylococcus aureus B. Moraxella catarrhalis C. Pseudomonas aeruginosa D. Streptococcus pneumoniae

Explanations (u) A. See D for explanation. (u) B. See D for explanation. (u) C. See D for explanation. (c) D. The most common pathogens in children with acute otitis media are Streptococcus pneumoniae, Haemophilus influenzae, and Streptococcus pyogenes.

Health Maintenance/ENT/Ophthalmology In addition to tobacco products, which of the following is also considered a major risk factor in the development of oral cancer? A. Sun exposure B. Alcohol abuse C. Occupational exposure D. History of oral candidiasis

Explanations (u) A. Sun exposure is a risk factor for cancer of the lip, but is not considered a major risk factor for oral cancer. (c) B. Major risk factors for development of oral cancer are use of tobacco products and alcohol abuse. (u) C. While occupational exposures and presence of premalignant lesions, such as leukoplakia, are risk factors for development of oral cancer, they are not considered major risk factors. (u) D. History of oral candidiasis has no correlation to development of oral cancer.

Diagnostic Studies/ENT/Ophthalmology A 26 year-old male presents with headache, sinus pressure, and sinus congestion for over a month. He has a thick nasal discharge in the mornings, but this improves as the day goes on. He is afebrile. On exam, there is tenderness over the face. TMs have normal light reflex. Nasal mucosa reveals thick yellowish discharge. Neck is supple, without lymphadenopathy. Which of the following is the diagnostic study of choice? A. transillumination of sinuses B. routine sinus films C. CT scan of sinuses D. nasal culture

Explanations (u) A. Transillumination is used in the initial evaluation of chronic or acute sinusitis, but is not sensitive or specific. (u) B. See C for explanation. (c) C. CT scan is more sensitive than plain films for the diagnosis and management of chronic sinusitis, and is considered the gold standard for sinus imaging. (u) D. Nasal culture is not indicated in the evaluation of chronic sinus infections.

Strabismus

Eye condition in which binocular fixation is not present. May occur in one eye or both and corneal light reflex will reveal misalignment. Cover-uncover test may reveal latent variant which may not otherwise be readily apparent. Estropia - inward misalignment Extropia - outward misalignment May be corrected with eye exercises (patch therapy) or in severe cases with surgery. Amblyopia will result if left untreated after age 2.

What type of oropharyngeal infection are patients who use inhaled steroid inhalers likely to have?

Fungal

Systemic corticosteroids

Giant Cell Arteritis treatment

Defined as an increased intraocular pressure w/ optic nerve damage.

Glaucoma

Increased intraocular pressure is AKA . . .

Glaucoma

Increased intraocular pressure with optic nerve damage

Glaucoma

Sx of acute sinusitis

HA, pain in face, worse with leaning forward, purulent drainage, fever, malaise, tooth pain

A 16 year old with infectious mononucleosis asks if he can play football this weekend. What should you advise him?

He may not - Activity must be limited due to the increased chance of splenic rupture

What disease is unilateral painless persistent cervical lymphadenopathy often linked to?

Hodgkin's lymphoma

Painful, (+/-) purulent eyelid nodule

Hordeolum

This is an acute development of small, mildly painful nodules or pustules within a gland in the upper or lower eyelid.

Hordeolum (STYE)

Slit-lamp Fluorescein stain

How do you look for a corneal abrasion?

How to tx recurring aphthous ulcers

Hydrocortisone

mastoiditis Tx

IV abx: ampicillin, cefuroxime

Painless, white oral lesion Can't be scraped of

Leukoplakia clinical features

What symptoms comprise the classic triad of Meniere's?

Low frequency hearing loss - Tinnitus with aural fullness - Vertigo

How is Optic Neuritis confirmed?

MRI

Symptoms related to distention of the inner ear's endolymphatic compartment

Meniere Disease

Recurrent vertigo Tinnitus One-sided aural pressure

Meniere Disease clinical features

Candida albicans

Most common pathogen of Oral Candidiasis

S pneumoniae

Most common pathogen of Orbital Cellulitis

Pseudomonas

Most common pathogen of Otitis Externa

Tx for dacryocystitis?

Oral Abx

Throat, mouth pain Creamy white patches Can be scraped off to reveal underlying erythematous mucosa

Oral Candidiasis clinical features

A white plaque-like oral lesion which does not scrape off?

Oral leukoplakia

What is the Dx? Acute painful swollen auricle tragal tenderness and otorrhea.

Otitis externa

What're the s/x of a Corneal Abrasion?

Pain, photophobia, redness, blurry vision, small pupil. The abrasion will also be visible w a fluoroscein stain

Most otitis media is caused by viral agents. What is the recommended first line treatment for a patient who has a bacterial otitis media?

Penicillin based antibiotics such as amoxicillin

fleshy triangle shaped protrusion on the inner bulbar conjunctiva

Pterygium is a complication of exposure to ultraviolet light and wind. It consists of hyaline and elastin tissue. If it encroaches on the cornea, surgical removal is indicated.

Complications of Strep throat (Group A strep)?

Rheumatic fever - Post streptococcal glomerulonephritis

Allergic Conjunctivitis

Ropy or stringy mucoid discharge. Pruritis

What are the three most common organisms involved in otitis media?

Streptococcus pneumoniae - Hemophilus influenzae - Moraxella catarrhalis

Which salivary gland is most often affected with sialolithiasis?

Submandibular gland

TM mobility

TM's are mobile

TMJ

TMJ is not tender and without swelling

A 54-year-old female presents complaining of decreased visual acuity to her right eye over the past few hours. She denies pain, and describes having wavy vision and seeing flashes of light. Her visual acuity in the affected eye is 20/200. What condition best describes the following physical finding? A. Retinal detachment B. Central retinal artery occlusion C. Open angle glaucoma D. Angle closure glaucoma E. Optic neuritis

The answer is A. EXPLANATION: The image demonstrates a detached retina. The superior aspect of the retina appears wavy and flowing.

Which of the following conditions is a cause for central vertigo? A. Meniere syndrome B. Labyrinthitis C. Vestibular neuronitis D. Acoutic neuroma E. Perilymphatic fistula

The answer is D. EXPLANATION: Acoustic neuroma, or eight cranial nerve schwannomas, are among the most common intracranial tumors, and a cause for central vertigo.

A 26-year-old female presents with a whitish coating on her tongue and lips. When you attempt to rub the plaque with a tongue depressor, a small amount of bleeding is noted from the oral mucosa. Her past medical history includes asthma. What diagnostic test would you perform to confirm your diagnosis?

The history and physical are consistent with oral thrush. A wet mount potassium hydroxide preparation will reveal fungal spores and nonseptated mycelia. A culture and sensitivity is used for suspected bacterial infections. A gram stain is used to identify whether a bacterial pathogen is gram positive or gram negative.

Name surgical option for glaucoma

Traveculoplasty

Describe Herpes simplex Keratitis

Under fluorescein stain, Dendritic ulcers

What is important to test in patients w a foreign body in their eye?

Visual Acuity (use that Snellen chart!)

Critical: Visual Acuity

Visual acuity is 20/20 for OD OS OU

Weber and Rinne test

Weber is midline, AC is greater than BC bilaterally

When is surgery indicated for a pterygium?

When it begins to effect vision (Obstruct cornea or induce astigmatism)

Oral candidiasis

White patch. Bleeds when wiped. Oral nystatin.

anterior (blepharitis)

_______ blepharitis involves the skin & eyelashes; commonly caused by staph aureus; treated with eyelid hygeine

vanco/clindamycin

antibiotics used to treat orbital cellulitis caused by MRSA

nortriptyline

antidepressant drug for tinnitus

hyperopia

far-sightedness, can see objects in the distance but close objects are blurry; corrected with plus lenses

dacryoadenitis

inflamed nasolacrimal duct Tx: gentamicin, tobramycin, erythromycin

sx of corneal abrasion

pain, foreign body sensation, photophobia, tearing, injection, blepharospasm

chalazion

painless

sx of blepharitis

red rims, adhered eyelashes, dandruff, scales

Critical: Head and face palpation

there are no masses and no tenderness elicited, the temporal artery is not tender

Trismus

"Lock Jaw" associated with peritonsillar abscess, ludwigs angina, dental abscess and tetanus.

allergic rhinitis

"hay fever"; causes pale/bluish nasal mucosa, possible nasal polyps or nasal crease; treated with intranasal corticosteroids & antihistamines

epistaxis

"nosebleed"; many causes; treated by direct pressure for 15 mins, nasal packing, possible surgery

vertigo

"room is spinning" Tx: meclizine + Romberg test = central vertigo

acute angle closure glaucoma

"steamy cornea" fixed DILATED pupil increased pressure in anterior chamber halo around lights pain decreased visual acuity decreased peripheral vision emergency

otitis externa

"swimmer's ear"; infection of the external ear canal; caused by swimming, mechanical trauma; causes ear pain, itching, & purulent discharge; treated with ear wick & antibiotics

Tx of Ectropion/Entropion?

(Entropion image) Surgical repair if needed

Define these two terms: Ectropion Entropion

(Image) Ectropion: eyelid propelled outward (Cause dryness) Entropion: Eyelid turns inward causing conjunctival irritation

external hordeolum

(stye) arises from eyelash follicles or a lid-margin tear gland

What're 3 common species of bacteria that cause Bacterial Conjunctivitis? What're two less common (but extra gross) causes?

*Common*: S. pneumo S. aureus H. flu *Uncommon*: Chlamydia Gonorrhea

orbital cellulitis

- 2try to sinus infection (ethmoid 90%) -sx- decreased vision, pain with ocular movement -dx- CT -tx- Vanco, clinda, cefotaxime DD-Preseptal cellulitis= URI, not associated with vision changes and ocular pain****

Acoustic neuroma

- CN8 schwannoma -sx- unilateral hearing loss in acoustic neuroma until proven otherwise -dx- MRI or CT -tx- surgery

What's the tx for a Corneal Abrasion?

- Cycloplegics (to dilate pupil and relieve pain) - Topical ABX (Erythromycin) - Maybe a pressure patch

scaphoid fracture

- FOOSH, MC fracture -sx-pain along radial surface, snuffbox tenderness -dx- fracture maynot be seen on xray for up to 2 weeks -tx- thumb spica

oral herpes simplex

- HSV1, MC 6 months- 5 yrs. - gingivitis- friable, bleeding gums -tx- self limiting

Osgood schlatters disease

- MC 8-15 -sx- Anterior knee pain with swelling of tibial tuberosity. increases gradually over time. worse when going upstairs -dx -lateral radiography is usually normal. may show fragmentation of tibial tubercle -tx- abstain from physcial activiity for as long as several monthes. RICE

diabetic retinopathy

- MC cause of new, permament vision loss/blindness in 23-74 Nonproliferative: venous dilation, microaneurysms (blot and dot hemorrhages, flame shaped hemorrhages, cotton wool spots, hard exudates) - not associated with vision loss and treated with laser therapy proliferative: neovasculization, vitreous hemorrhage - VEGF inhibitors, laser photocoagulation, tight glucose control

bacterial conjunctiviits

- MC staph/strep, h. flu, moraxella - purulent d/c, lid crusting, usuallly no visual changes (mild pain) - absence of ciliary injection - secondary bacterial keratitis can develop tx: erythmycin topical - if contact lens - (FQ or tobrex) - gonnoccocal (5d ceftriaxone IV)- unilateral - chlamydia (azithromycin)- follicular response, nontender preauricular adenopathy common - if rare pathogen suspected: grain stain and giemsa stain ( no organisms seen with chlaymida, intracellular gram-negative diplococci in gonorrhea) neonatal: day 1: AgNO3 day 2-5: gonococcal day 5-7: chlamydia day 7-11: HSV

Scolisis

- Right thoracic MC, Idiopathic Sclosis MC spinal deformity evaluated -sx- Right thoracic and left lumbar prminence with flank crease with forward bend -dx- Adams forward bend test -Single, standing AP Radiograph if >5 degrees on scoliometer -tx-10-15 angle- 6-12 month follow up. clinical+ possible x rays 15-20- AP 3-4 month follow up for large, 6-8 monthes for small Curves ≥20 degrees require orthopedic referral - difference of 5% between 2 measurements- treat with brace

osteomylistis

- Staph Arues #1!!! Lumbar spine is where is spreads hemtagenous. hip 1!!! -samonella= sickle cell**** -sx- inflammation over bone. increased ESR and WBC**** -dx- gold standard= bone aspiration, MRI, xray- periosteal reaction, sequestrum. -tx- IV abx 4-6 weeks= 4 months strep B= Nafcillin or oxacillin+ 3rd gen stap a more than 4 month= nafcillin or oxacillin Samonella= FQ or 3rd, MRSA resistant= Vanco -puncture wound= cipro

foreign bodies

- TEST VISUAL ACUITY FIRST - may be on cornea or under upper lid - local anesthetic (proparacaine) - fluorescein - remove with fine gauge needle (avoid wet cotton tip? - steel foreign bodies look for rust ring- may be removed with a rotating burr or the pt referred - polymycin-bacitracin or erythromycin - pain releieved with instillation of analgesic drops - do not send pts home with local analgesics

optic neuritis

- acute inflammatory demyelination of the optic nerve - MC young pts 20-40 - multiple sclerosis MC, meds (ethambutol for TB, chloramphenicol), autoimmune - loss of color vision, visual field defects (central scotoma/blind spot), loss of vision over a few days (usually unilateral) - associated with ocular pain that is worse with eye movement PE: Marcus Gunn pupil: relative afferent pupillary defect- when bright light is shone from the unaffected eye to the affected eye, pupils appear to dilate rather than constrict (delayed response of affected nerve) Fundoscopy: 2/3 normal (retrobulbar neuritis) or 1/3 with optic disc swelling/blurring (papillitis) - MRI! IV methylprednisolone followed by PO steroids - vision usually returns with tx

macular degeneration

- age related to d/t toxic effects of drugs (chloroquine or phenothiazine) - leading cause of irreversible central vision loss - increases after age 50, white, females, smoking - Drusen deposits in Bruch membrane - can also have wet - new abnormal vessels grow which leak and bleed= retinal scarring- rarer but more progressive - GRADUAL bilateral blurred or loss of central vision (including detailed and colored vision), scotomas (blind spots, shadows), metamorphopsia (wavy or distorted vision can be measured with Amsler grid) - micropsia (object seen by affected eye smaller than unaffected) - visual loss deteriorates quickly once neovascular degeneration - mottling, serous leaks, and hemorrahges - scarring develops in end-stage disease TX - no effective treatment - if detected early - laser therapy or intravitreal injections of monoclonal antibody drugs may slow progression - vitamins, antioxidants, zinc and copper, and omega-3 fatty acids may reduce progression

glaucoma

- any impediment to flow of aqueous humor through trabecular meshwork and canal of Schlemm will increase pressure in the anterior chamber

strabismus

- binocular fixation not present- should be by 2-3 months - may occur in one or both eyes - corneal light relfex test - cover-uncover test - possible diplopia, scotoma inward misalignment (esotropia) outward misaligment (exotropia) TX- eye exercsies (patch therapy) or surgery in severe cases - if left untreated after the age of 2- amblyopia will result

tx for hyphema

- blood in anterior chamber - place at 45 degrees to keep RBCs from staining cornea

hydroxycholorquine and sildenafil

- cause cause blindness - sildenafil mostly color changes

blepharitis

- chronic inflammation of the eyelids - burning and itching of the eyes - no change in vision - eyelids show scaling and crusting, in seborrheicd the scales are greasy

Osteoarthritis***

- chronic- articular cartilage damage and degeneration Obesity RF. MC in weightbearing joints -sx- evening joing stiffness-- pain worsens throughout day, hard bony joint -herb nodes (DIP joints!)- barn-PIP -dx- XRAY- osteophytes -tx- acetaminophen! in elderly with bleeding risks. NSAIDS more effective, CAPSIN (Pts who cannot tolerate orals) corticosteroid injections****

dacryostenosis

- common in the newborn after the first month of life and occurs when the duct does not open - obstruction usually resolves by 9 months TX: warm compresses and massage; surgical probe

etiology of cataracts

- congenital (rubella, CMV) - traumatic - systemic disease (diabetes) - medications (steroids, lovastatin) - senile (most common > 60)

keratitis

- corneal ulcler/inflammation - MC d/t bacteria, viruses, fungi, (acanthamoeba in contact lens wearers) - associated with rapid progression and sight threathening - exposure keratitis (bells palsy) - pain, photophobia, reduced vision, tearing - conjunctival injection/erythema, limbic flush (ciliary injection), corneal ulceration, purulent or watery discharge - Bacterial: hazy cornea, ulcer, stromal abscess, +/- hypopyon Tx: FQ, no patching HSV keratitis: dendritis lesions tx: trifluidine, vidarabine, acyclovir

Ankle sprain

- dislocation- posterior MC- anterior talofibular, calcanrofibular MC -sx- pop swelling, inability to weight bare -tx- RICE, NSAIDS

herpes zoster ophthalmicus

- due to infection with varicella-zoster - infects trigeminal nerve the ophthalmic branch - shingles noted on tip of nose think eye involvement - fluorescein stain to look for dendritic ulcers - refer!

angle-closure pearls

- elderly, hyperopes (far-sighted), Asians precipitating factors: mydriasis (pupillary dilation further closes the angle)- dim lights, anticholinergics, sympathomimmetics - eye feels hard to palpation - tonometry shows > 21 mm HG - "cupping" of optic nerve -

chemical burns

- every minute counts! - irrigation stated ASAP - alkali burns worse than acids (liquefactive necrosis), denatures proteins and collagen, causes thrombosis of vessels - ex: fertilizers, household cleaners, drain cleaners acid burns: coagulative necrosis (H+ precipitates protein barrier)- industrial cleaners, batteries tx: irrigation- LR or NS (LR is better b/c closer to eye PH) for 30 minutes or at least 2 liters - check pH and acuity after irrigation: irrigate until eye pH 7.0-7.3 x 30 minutes - broad spectrum abx and cyloplegic agent

otitis externa

- excess water or local trauma changes the normal acidic pH of the ear, causing bacterial overgrowth - pseudomonas MC, staph and strep - aspergillus (fungal) - 1-2 d of ear pain, pruritis in ear canal, auricular discharge, pressure/fullness (recent activity of swimming) - hearing usually preserved PE: pain on traction of the ear canal/tragus, external auditory canal erythema/edema/debris TX: - protect ear against moisture (drying agents include isopropyl alcohol and acetic acid) - ciprofloxacin/dexamethasone ( ofloxacin safe if associated TM perf) - aminoglycoside combo: neomycin/hydrocortisone otic ( not used in suspected TM perf)

central retinal artery occlusion

- eye emergency - poor prognosis even with treatment - causes: emboli, thrombosis, vasculitis - must be differentiated from giant cell arteritis (fever, HA, scalp tenderness, jaw claudication, visual loss) - sudden, painless, marked unilateral loss of vision - fundo: pallor of the retina, arteriolar narrowing, cherry red macula (red spot) - veins may show segmentation (box car apperance) - may see emboli 20% - no hemorrhage - usually preceded by amaurosis fugax TX: recumbent position and gentle ocular mssage may help reduce damage - vessel dilation and paracentesis - workup for athersclerosis and arrythmias - decrease IOP with acetolamide

Juvenile rheumatoid arthririts

- females over males -sx-- "Morning stiffness" more than 6 weeks for over 30 min. incidous onset systemic= fever spikes, polyathralgias, salmon piink maculopapular rash, weight loss. -pauciarticular= 4 or fewer joints -polyarticular- 5 or more joints, low grade fever, fatigue, rhuematoid nodes, anemia -dx- + RF, anti-CCP- systemic=ESR/CRP increased - pauarticular- ANA pos -tx- NSAIDS, steroids PT, OT. methotrexate

orbital floor "blowout" fractures

- fractures of the orbital floor ( maxillary, zygomatic, palatine) - decreased visual acuity - enophthalmos (sunken eye) - diplopia esp. with upward gaze (inferior rectus entrapment) - orbital emphysema (eyelid swelling with blowing the nose from connection to maxillary sinus)- may have exopthalmos - epistaxis, dyesthesias, hyperalgesia, anesthesia to anteriormedial cheek (d/t stretch of intraorbital nerve) - CT scan TX: initial: nasal decongestants (decreases pain), avoid blowing nose, prednisone (to reduce edema), abx ( unasyn or clinda); surgical repair. optho referall

chronic otitis media

- from repeated epsidoes of acute otitis media, trauma, or cholesteatoma - different organisms ( pseudomonas, s. aureus, proteus, anaerobes) - PERFORATED TM WITH CHRONIC EAR DISCHARGE with or without pain - TM and/or ossicular damage can lead to CONDUCTIVE HEARING LOSS TX: removal of infected debris, avoidance of water exposure, and topical abx drops - surgery (TM repair/reconstruction)

bacterial conjunctivitis labs and tx

- gram stain of eye TX - sulfonamide 10% ophthalmic solution - GC: IV ceftriaxone, topical erythromycin - chlamydia: oral tetracycline, doxy, axithromycin

chalazion

- granulomatous inflammation of a meibomian gland - chronic disease - painless, hard, non-tender swelling of upper or lower lid - can distort vision - tx: warm compresses, incision and curettage

dacryocystitis

- infection of lacrimal sac - obstruction of nasolacrimal system - acute: s. aureus, beta hemolytic strep, h. flu - infants and adults > 40 - pain, swelling, tenderness, redness in tear sac area - usually unilateral - may have purulent discharge - tx: remove obstruction, systemic abx ( clindamycin + 3rd gen. cephalosporin - dacrocystorhinoscopy

acute otitis media

- infection of middle ear, temporal bone and mastoid air cells - MC preceded by URI - STREP PNEUMO, H. FLU, M. CATT, STREP PYOGENES (same as bronchits and sinusitis) - peak age 6-18 months - URI causes eustachian tube edema leading to transudation of fluid and mucous in middle ear which is colonized by bacteria risk factors: young, eustachian tube dysfunction, day care, bottle use, parental smoking, symptoms: fever, otalgia, ear tugging in infants, conductive hearing loss, stuffiness - if TM perforation - rapid relief of pain and otorrhea (usually heals in 1-2 days) PE: bulging, erythematous TM with effusion and DECREASE TM MOBILITY on pneumatic otoscopy - loss of landmarks - if bullae on TM- suspect mycoplasma TX 1. amoxicillin x 10-14d DOC 2. augmention 2nd line 3 if PCN allergic - erythromycin, azithroxymycin, bactrim 4. myringtomy (drainage) if severe otalgia or mastoiditis 5. tympanostomy if recurrent or persistent for recurrent cases- iron def. anemia w/u, CT scan

mastoiditis

- inflammation of the mastoid air cells of the temporal bone - prolonged or inadequately treated AOM s/sx: deep ear pain (usually worse at night), mastoid tenderness, may develop cutaneous abscess complications: hearing loss, labyrinthitis, vertigo, CN VII paralysis tx: 1. IV abx with myringotomy (ampicillin), 2. followed by full course of oral abx 3. refractory = mastoidectomy

polymyosititis

- inflammatory disease of proximal limbs, neck, pharynx -sx- Progressive symmetrical proximal muscle weakness -dx- increased aldolase!!!, + anti-jo 1 Ab, +anti-srp ab -Dermatomyosis= Helotrope rash (blue discoloration on eye lids) Gottrons papules!!! (fingers)

entropion

- inward turning of the lower lid - seen in the elderly - eyelashes seen against the eyeball- can cause scarring - have redness and tearing - surgical tx

cataracts

- lens opacity, usually bilateral - visual loss, yellow discoloration center of lens - absent red reflex

lesions of the optic nerve

- lesions anterior to the optic chaism will affect only 1 eye - lesions at optic chiasm will affect both eyes partially (bitemporal hemianopsia) - posterior to optic chiasm left lesion (right sides of both visual fields)

hordeolum

- local abscess of the eyelid margin (external sebaceous gland) - staph aureus 90-95% - painful, warm, swollen, red lump on eyelid - warm compressed mainstay of tx (most eventually point and drain) - +/- topical abx ointment (erythomycin, bacitracin) if actively draining - I and D if no drainage within 48 hours

bacterial conjunctivtis

- more common in adults - S. pneumo, S. aureus, h. flu - transmit via direct contact, autoinoculation - chlamydia, gonorrhea (ocular emergency) - purulent discharge, mild loss visual acuity

viral conjunctivits

- more common in children, midsummer to early fall - adenovirus type 3, herpes - CONTAGIOUS, transmit by direct contact - clinical: URI, sore throat, fever, malaise common early - starts unilateral but becomes bilateral in 3-5 days - copious watery discharge, erythema, periauricular lymphadenopathy TX: sulfonamide ointment, cool compresses - follicullar along with chylamydia

Lunate fracture

- most serious fracture -avascular necrosis of lunate bones= kienbocks disease -tx- thumb spica

papilledema

- optic nerve swelling from increased ICP (classically bilateral) etiology: - idiopathic intracranial HTN (psuedotumor cerebri) - space- occupying lesions (tumor, abscess) - increased CSF production - cerebral edema, severe HTN exam: headache, nausea/vomiting, vision usually well preserved, may have enlarged blind spot or transient visual alterations diagnosis: swollen optic disc with blurred margins MRI or CT scan first to rule out mass effect- LP puncture TX: diuretics (acetazolamide which decreases aqeuous humor and CSF), treat cause

ectropion

- outward turning of the lower lid (basset hound) - common in the elderly - lower lid margin droops outward - may have excessive tearing, but eyes typically dry - surgical tx

corneal ulcers

- pain, photophbia, and tearing - risk factors include trauma, contact lens use, and poor lid apposition - exam will reveal circumcorneal injection and watery to purulent discharge - fluorescein staining reveals dense corneal infiltrate with overlying epithelilal defect - dendritic lesions indicates herpes keratitis - all should be refered - lesion should be stained and cultured to identify cause - avoid topical steroids b/c cause further tissue loss and increase risk of perforation

angle-closure

- painful eye and loss of vision PE: circumlimbal injection, steamy cornea, fixed mid-dilated pupil, decreased visual acuity, and tearing - anterior chamber narrows- IOP acutely elevated - nausea, vomiting and sweating are common TX: start IV carbonic anhydrase inhibitor (acetazoalmide), topical beta blocker and osmotic diuresis (mannitol) - do not give mydriatics - laser or surgical iridotomy

chalazion

- painless granuloma of the internal meibomian sebaceous gland - focal eyelid swelling - often these are larger, firmer, slower growing and less painful than syte - hard, nontender eyelid swelling on conjunctival surface of eyelid TX: eyelid hygiene, warm compresses. abx usually not necessary. injection of steroid or incision and curettage

open-angle

- people over 40 and more common in AA and in pts with a family history of glaucoma or diabetes - chronic, asymptomatic, and potentially blinding disease that affects 2% - increased IOP, defects in the peripheral visual field, and increased cup to disc ratios - asymptomatic until late in disase - loss of peripheral vision and halos around lights - elevated IOP without optic disc damage known as ocular hypertension- close monitoring required - optic nerve damage without increased IOP seen- subsequent monitoring typically reveals increasing IOP TX: optho - topical and/or systemic meds to decrease IOP by decrease aqeuous production (beta blockers, carbonic anhydrase inhibitors) and/or increasing outflow (prostaglandins, cholinergics, epi) - alpha agonists (brimonidine) provide both mechanisms

cataracts

- progressive increase in the proportion of insoluble protein - secondary to natrual aging process (senile) or d/t trauma, congential (torch), systemic disease ( DM), or meds (steroids, statins) - excess sun exposure - gradual dimunution of vision - possible double vision, excess glare, fixed spots, or reduced color perception - typically bilateral - translucent, yellow discoloration in lens Fundo: dark against a red background - once mature- retina no longer visible tx: intracapsular or extracapsular extractions with lens replacement

glaucoma meds

- prostaglandin analogs - beta blockers (timolol) - alpha agonists - carbonic anhydrase inhibitors - pilocarpine (also used for dry mouth)

Barotrauma

- rapid pressure changes, inability of ET to equilize pressure -scuba, airplane -tx- oral degongetants, auto insufflation

allergic conjunctivitis

- red eyes - may have viral sx (rhinorrhea, fever, malaise, pharyngitis) - cobblestone mucosa appearance to inner/upper eyelid, ITCHING, tearing, redness, stringy d/c, may have photophobia, visual loss - usually bilaterla - possible chemosis (conjunctival swelling) tx: topical antihistamines,

amblyopia

- reduced visual acuity not correctable by refractive means - MC caused by strabismus; uremia, or toxins such as alcohol, tobacco, lead,

retinal detachment

- separation of the retina from the pigmented epithelial layer, causing the detached tissue to appear as flapping in the vitreous humor - tears most commonly begins at the SUPERIOR TEMPORAL retinal area - spontanoulsy or d/t trauma; extreme myopia (nearsigthed); or inflammatory changes in the vitreous, retina, or choroid - acute onset of painless blurred or blackened vision that occurs over several minutes to hours and progresses to complete or partial monocular blindness - bilateral detachment 20% - curtain being drawn over the eye from top to bottom - may sense floaters or flashing lights at the beginning - IOP is normal or reduced PE: relative afferent pupillary defect (marcus gunn) fundo exam may reveal the ridges of the displaced retina in the vitreous humor TX: emergency consult regarding possible laser surgery or cryosrugery - SUPINE with the head turned to the side of the retinal detachment - prognosis is good: 80% recover without recurrence 15% retreatment 5% never attach

hordeolum

- staph infection of the meibomian gland (internal) or glands of zeis or moll (external) - a stye (external) - red, swollen, tender area on upper or lower lid - tx: warm compresses, abx ointment (E-mycin, bacitracin) - if no improvement in 2 days can I and D

drugs that increase glaucoma risk

- steroids and anticholinergics closed angle: steroids, anticholinergics, sympathomimetics, topiramate, antihistamines, antidepressants, phenothiazines

treatment for blepharitis

- supportive, lid scrubs ( baby shampoo) - antibiotics: oral (tetracycline), eye ( E-mycin, Bacitracin)

SLE

- sx- MALAR, DISCOID RASH. serositis= pericarditis, plueritis*** drug induced= procainamide, INH, Hydralazine -dx-sensitive= ANA (BEST). specific= anti-sm Abs, DS-DNA. -tx- regular exercise, sun protection, NSAIDS, MTX (gastritis), hydroxychloquine (diahrea, rash)

amaurosis fugax

- temporary monocular vision loss (lasting minutes) with complete recovery - d/t retinal emobli ior ischemia - EXTERNAL CAROTID ARTERY commonly involved

globe rupture

- the outer membrane of eye disrupted by blunt or penetrating trauma - EMERGENCY - ocular pain (maybe absent), diplopia PE - misshaped eye with prolapse of ocular tissue from sclera or corneal opening - markedly reduced visual acuity (maybe light perception only) - ENOPHTHALMOS (recession of the globe within the orbit), foreign body maybe present, may have exopthalmos - SEVERE CONJUNCTIVAL HEMORRHAGE ( 360 bulbar) - corneal/sclera: prolapse of the iris through the cornea - positive Seidels test ( parting of the fluorescein dye by a clear stream of aqueous humor from anterior chamber) - obscured red reflex - teardrop or irregularly shaped pupil - hyphema TX: rigid eye shield (protect eye from applied pressure, impaled object should be left undistrubed) immediate refer - IV ABX - avoid topical eye solutions

central retinal vein occlusion (CRVO)

- thrombus leads to fluid backup in retina - sudden monocular vision loss - risk factors: HTN, DM, glaucoma, hypercoaguable states PE: afferent pupillary defect, optic disc swelling, blood and thunder retina (dilated veins, hemorrhages, edema, and exudates) - vision typically resolved with time at least partially - workup for thrombosis - neovasculization treated with vascular endothelila growth factor inhibitors

Hip Dislocation

- trauma MC cause avascular necrosis= complication*, sciatic nerve injury , posterior MC -sx- hip pain with leg shortened internally and rotated -tx- orthopedic emergency!

ocular abx

- used for blepharitis, conjunctivits, stys, - gentamycin, tobramycin, erythromycin, besifloxacin

orbital cellulits

- usually secondary to sinusitis (ethmoid 90%) - s. pneumo, GABHS, h. flu, s. aures - may be caused by dental infections, bacteremia, dacrycystitsi, facial infections - MC in children (7-12) - DECREASED VISION, PAIN WITH OCULAR MOVEMENTS, PROPTOSIS, chemosis (swelling of bulbar conjunctiva), increased IOP, visual changes, eyelid edema, exopthalmos, erythema, discharge diagnose with CT scan - infection of fat and ocular muscles - IV abx (vanc, clinda, cefotaxime) until fever subsides then complete 2 to 3 weeks of oral abx - MC strep, h. flu, s. aureus - can lead to meningeal or cerebral infection ddx: preseptal cellulitis (infection of eyelid and periocular tissue) - associated with URI (no visual changes or pain with ocular movement)

spondylisis/spondylthesis

- young athlete MC -spondylisis- defect in pars interarticular from repetive hyperextension trauma MC L5/S1. seen on oblique films -spondylithesis- forward slipping of vertebrae on another. 10-15 y/o MC -tx- spondylisis- activity restriction, PT spondylisthesis- PT. high grade- surgery

Mastoiditis -Dx test

-CT

hearing impairment

-Conductive - Webber lateralized to AFFECTED ear. Rinne B > A -SensoriNeural -Webber lateralize to UNAFFECTED ear. Rinne A > B conductive-causes- external or middle earCerumen Impaction, URI, Acute otitis external, Otosclerosis (abnml new bone forms), Otitis media -tx- audiological testing -sensorineural- inner ear

Otitis Externa -1st line Tx

-Fluoroquinolone +/- steroid (Ciprodex)

ACL

-MC Injured knee ligament -sx- noncontacting pivoting injury, pop swelling and swelling with hemarthrosis -dx- lachmans test- most sensative

optic neuritis

-MS- MC -sx- loss of color vision, unilateral- ocular pain worse with eye movement -dx- marcus gunn pupil***- pupils dilate rather than constrict when light is shown -fundoscopy- optic disc swelling -tx- IV methyprednisolone with PO steroids

Mastoiditis -x3 Tx

-Myringotomy for drainage & culture -IV abx -Followed by oral abx

Chronic Otitis Media x4 MC pathogens

-Staph aureus -Pseudomonas aeruginosa -Proteus -Anaerobes

Acute Otitis Media x4 MC pathogens

-Strep pneumoniae -Staph pyogenes -H. influenzae -M. Catarrhalis

Otitis externa x3 MCC

-Water exposure -Trauma to canal -Exfoliative skin conditions (eczema or psoriasis)

Sjogren syndrome

-autoimmune system attacks exocrine glands -sx- dry eyes, dry mouth (xerostoma) enlarged parotid gland -dx- ANA, +RF, schirmers test (decreased tears production) -tx- artificial tears, Pilocarpine for xerostoma cevimeline (evoxac)

tibial plateau fractures

-axial loading/rotation/direct trauma (MC in children MVA- bumper) MC lateral plateau -sx- check peroneal nerve functions (foot drop) -tx- non displaced- conservative tx. displaced- ORIF

colles/smiths fracture

-colles- dorsal angulation,FOOSH wrist extension, EPL tendon rupture. tx- sugar tong -smiths fracture-ventral angulation, FOOSH wrist flexion

patellar fracture/ dislocation

-fractures- MC direct blow-dx- sunrises view X ray -dislocation- MC females, usually laterally, tx- closed reduction

Hip fracture

-high incident of avascular necrosis with femoral neck fractures, high incidence of DVT and PE -sx- hip pain with leg shortened, externally rotated, abducted -tx- ORIF

parotitis

-if viral- mumps, fever, anorexia fever -if bacterial- chewing argrivates pain of swelling of gland -dx- CT and MRI w/ gladodlinium -tx- symptomatically, local heat massage

lateral/medial epicondylitits

-lateral epicondylitits- inflammation of ECRB (extensor carpi radial brevis. gripping and forearm pronation -medial epicondylititis- pronator teres-flexor carpi radialis. worse with pulling activitis

Legg-pegg/ avascular necorsis

-most important and severe complication of DDH. -sx- 4-10. painless limping* dull achy throbbing pain at groin at end of day, lateral hips, buttocks. loss of rotation, abduction, anagenic limp -dx- MRI -tx-protected weight bearing. little benefit from bracing. Alendronate.

vertigo

-peripheral vertigo- labrinyth or verstibular nerve- BPV, Menieres, vestibular neuritits, labrithyisis, -tx- antihistamines, metoclopramide (antagonizes D2 receptors), anticolingergics- scopolamine - central vertigo- Vertical nystagmus, graual onset, migranes, CNS signs - tx- prophylaxis

Knee dislocations

-severe limb threatneing emergencies= Popliteal artery injuries!- 1/3 of patients- ATERIOGRAPHY+ tibial nerve injury -tx- immediate ortho consult

Skiers thumb/ boxers fracture

-skiers- UCL injury! instability of MCP joint* -sx- thumb far away from other digits - thumb spica -boxer- fracture of 5th metcarpal- always check for bite wounds* -tx- unlnar gutter splint

Humeral shaft fractures

-sx- FOOSH Must r/o radial nerve injury*** -tx- sugar tong splint

meniscal tear

-sx- US c/o locking, popping, giving way, effusion with activities. + mcmurrays sign- pop of click while tibia is externally and internally rotated -tx- NSAIDS

olecranon bursitis

-sx- abrupt "goose egg" swelling, limited ROM with flexion -tx- rest, NSAIDS

lumbosacral sprain/strain

-sx- back muscle spasms, no nuerological changes (no pain below knee) -tx- brief bed rest- 2 days

spinal stenosis

-sx- back pain, worsened with extension (walking) relieved with flexion -tx- lumbar epidural injections of steroids

Osteoid osteoma

-sx- benign tumor in children age 5 to 20, presents with increasing pain, worse at night and relieved by aspirin. spine, tarsal bone, phalanx, Trabecular ossification, osteolytic nidus

polymyalgia rhumatica

-sx- bilateral proximal joint stiffness, (shoulder and pelvic girdle pain and stiffness, a lack of skin findings) Closely related to Giant Cell arthritis!!! -dx- clinical -tx- cortiocosteroids (10-20 mg)

hyphema

-sx- bleeding anterior chamber, ENOPHTHALAMUS, tear drop pupil -dx- evaluate for global rupture injury- Positive Seidels test- parting of dye -tx- Rigid eye shield, restrict movement, bed elevate to 45 degrees, opthal. NO NSAIDS.

Nursemaids elbow

-sx- child presents with arm slightly flexed, refises to use arm -tx- reduction- pressure on radial head with supination and flexion

Cholestertoma

-sx- chronic negative middle ear pressure--- TM retracts medially. has potential to errode bone -dx- granular tissue seen with ottoscope -tx- surgical referal

retinopathy

-sx- copper/ silver knicking, atervenous nicking, cotton wool spots, retinal hemmorhages -dx- fundscopy -tx- optimize BP and glucose control

oral canidida

-sx- cottage cheese patches, can be scraped away to reveal erythemus mucosa -tx- Antifungal (Nystatin) - oral. Diflucan (last resort). treat moms nipple too!!!

AC joint disolcation

-sx- direct blow to adducted shoulders, pain with lifteing arn -dx- xrays taken with weights -tx- brief sling immobilization

Olecranon fractures

-sx- direct blow, inability to extend elbow -dx- all considered intrarticular and need reduction ulnar nerve dysfunction*

corneal abrasion

-sx- foreign body sensation, injection, photophobia. pain relieved at instillation of opthal analgesics drops -dx-1. visual acuity!!! slit lamp with flourscine -tx- topical anthestic (no rx!!!)+ patch for 24 hrs. topical abx (polymixicn/ bactericim), genta/ cipro for contacts. opthalamology follow up in 1-2 days. remove rust ring at 24 hrs

nystagmus

-sx- horizontal eye movement

septic arthritis

-sx- infection in joint cavity. staph areus MC -dx- athrocentesis (WBC 50,000, cloudy) -tx- -gram pos cocci- nafcillin - gram neg cocci- ceftriaxone - gram neg rods- ceftriaxone - no organism seen- naficillin

radial head fractures

-sx- lateral elbow pain, inability to fully extend elbow -dx- fat pad sign- posterior or increased anterior -tx- non displaced- sling long arm splint

Osteosarcoma

-sx- malignant bone tumors that present with pain and swelling. No improvement is noted with conservative therapy. METAPHYSIS -DX- Metaphysis, sunburst/Cloud-like bone formation, humerus, fibula, iliac bone. Surgery and chemotherapy - 50-50 chance of survival

Rheumatoid arthrititis

-sx- morning stiffness less than 1 hr, fever, weakness, muscle pain.. NO DIP -dx-RF 1st!!!! anti- CCP(95%)+ RF (75%) -tx- DMARDS 1st!!!, MTX (MONTIOR CBC diff. gastrisi MC SE)

Thoracic outlet syndrome

-sx- nerve compression ulnar side of hand, swelling, doscoloration of the arm with abduction of arm, + adson -dx- MRI -tx- pt

dupuytren contracture

-sx- nodes over distal palmar crease or proximal phalynx MCP!

Glaucoma

-sx- steamy cornea, fixed mid dialted pupil, tunnel vision loss -dx-increased intraocular pressure narrow anterior chamber -tx-acute closed- azcelomide (1st line), timlol (1st in open angle), pilocarpine, azcelomide (1st in angle closure), mannitol--- surgical irrtotomy---trabulectomy chronic open- prostagladin analogs

retinal vascular occulsion

-sx- sudden monocular vision loss -dx- fundscopy- blood and thunder appearance, retinal hemorrages -tx- no effective treatment

dental abcess

-sx- swelling at base of tooth, sharp gnawing pain -dx- xray for deeper abcesses -tx- drain and abx

laryngitis/mastoiditis

-sx- usually viral- rhino follows URI. hoarsenss -tx- Fluids, rest= if more than 3 weeks horesness evaluate for cancer- refer to ENT -mastoditis= CT, persistent adult- must R/O cancer, ENT referal tx- ampicillin

strabismus

-sx-"CROSS EYED", turns into amplytopia (must fix this lazy eye before 1 years) eso= inward. exo=outward -dx- cover/uncover -tx- patch good eye. under 3 monthes refer

slipped capital ephysis

-sx-ages 11-16, obese boy, limp with hip/thigh/knee pain. with limp, "Ice cream cone falling off" -dx- FROG leg LATERAL Pelvis= kliens lines -tx- pinning in situ= surgical

allergic rhinitis

-sx-boddy, shiner -dx- ige medated allergy test -tx- intranasal flucusalone/corticosteroids--- antihistamines

meniere disease

-sx-excessive fluid in middle ear -recurring vertigo min-hours -triad- low freq SNHL, fluctuating tinnitus, Episodic vertigo -tx- low sodium diet and diruetics---scoplamine, meclizine

polyarthritis nodosa

-sx-fever, hip, and shoulder muscle weakness.... renal hypertension, renal failure, fever, arthritis, neuropathy, livedo reticularis- pt will have Hep B, purapura -dx- - increased ESR -Angiography recommended (will show rosary sign) - CLASSIC is ANCA Negative! - mesenteric angiography will show microaneursism in small arteries -tx- prednisone and cyclophosphamide

Vertigo caused by problems with the inner ear or vestibular system, which is composed of the semicircular canals, the otolith (utricle and saccule), and the vestibular nerve is called "peripheral", "otologic" or "vestibular" vertigo.The most common cause is benign paroxysmal positional vertigo (BPPV), which accounts for 32% of all peripheral vertigo.Other causes include Ménière's disease (12%), superior canal dehiscence syndrome, labyrinthitis and visual vertigo.

...

what is normal audiometry measurement

0-20 decibels

hypertensive retinopathy

1. arterial narrowing: abnormal light reflexes on dilated tortuous arteriole shows up as colors - Copper wiring: moderate - Silver wiring: severe 2. AV nicking: venous compression @ arter-venous junction by increased arterial pressure. may lead to CRVO 3. flame shaped hemorrhages, cotton wool spots (soft exudates which are lighter than hard exudates) 4. papilledema (malignant HTN)

menieres dz classic triad

1. episodic vertigo 2. low frequency hearing loss 3. tinnitus

prostaglandin analogs

1st line class of treatment for chronic glaucoma

antihistamines

1st line treatment class for vertigo

tx of dental abscess

1st--ABs 2nd--root canal 3rd--tooth extraction

remove rust ring at

24 hours

vestibular system

3 semicircular canals originating in the vestibule responsible for balance

Viral infection of the conjunctiva is usu. caused by adenovirus type ___, __ or ____

3, 8 , 19

When does Glaucoma screening begin?

40 years old - asymptomatic prior to optic nerve damage and subsequent visual changes

pathogens of acute sinusitis

40% S. pneumoniae 30% H. influenzae 20% m. catarrhalis 10% other

open angle glaucoma

65+ y/o must be screened increased or normal IOP cut to disc ratio > 0.5 TONOMETRY > 21 optic disc cupping Tx: Lantaprost, BB, timolol, Brimodidine

normal IOP

8-21 mmHg

Sinusitis

80% of cases will improve within 2 weeks without antibiotic therapy. MCC is viral. Results from mucociliary clearance and sinus obstruction due to mucosal edema. Mucus accumulation becomes secondarily infected. Bacterial pathogens are similar to those of Ottis media (S pneumoniae, Other strep, Hflu, S aureus, Moraxella). May present with unilateral facial fullness, pressure, tenderness in cheek area. Pain may refer to upper incision or canines. Prudent yellow-green nasal drainage or expectoration. Fever, malaise halitosis, HA and cough. MC site is MAXILLARY for adults, FRONTAL for peds. Bacterial variant will persist more than 10 days. Viral variant will tend to be improving at 10 days. Tx with NSAIDs, oral or nasal decongestants. If symptoms persist for more than 10 days, ABX are indicated. 1st line = amoxicillin, macrolide, or tetracycline therapy if PCN resistance or allergy. Tx for 10 days.

acoustic neuroma

8th cranial nerve dx: MRI

Acute sinusitis

<4 weeks. Viral

Chronic sinusitis

> 12 weeks. Anosmia. CT scan. Glucocorticoids, antibiotics (weeks), nasal saline

What is the Tonometry pressure concerning for Open-Angle Glaucoma?

> 21 mmHG

open angle glaucoma

>40, chronic, asx, can cause blindness

A 30-year-old male presents to your office complaining of sinus and facial pain, congestion, and purulent nasal discharge for one month. He has been treated with two courses of different antibiotics by another provider, and does not feel any improvement in his symptoms. What diagnostic test is indicated?

A CT scan is the current preferred method for sinus imaging of chronic sinusitis. CT imaging has better visualization of mucosal thickening air-fluid levels and bone structures. Plain radiographs and CT scans are of limited use in acute sinusitis, because viral pathogens that cause sinus abnormalities are indistinguishable from bacterial causes.

is characterized by a pale retina, as well as a cherry red spot on the macula.

A central retinal artery occlusion

A 66-year-old female has a chief complaint of vision loss in her left eye. She denies pain and states that this occurred over the past few hours. Her past medical history includes hypertension, high cholesterol, and peripheral vascular disease. Upon funduscopic exam, you note marked hemorrhages in all quadrants and disc edema. The contralateral eye shows only mild hypertensive vascular changes. What is your diagnosis?

A central retinal vein occlusion is characterized by a "blood and thunder" fundus, with marked hemorrhages, tortuous vessels, and optic disc edema.

A 45-year-old male presents with a non-tender nodule protruding from his lower eyelid. There is some surrounding erythema to the conjunctiva, but no discharge is seen. He states that it has been there for one month. He has no visual problems. What is your diagnosis?

A chalazion is a sterile, chronic, and non-painful granulomatous nodule, caused by a previous acute infection in a meibomian gland. It can develop over a period of a few weeks. Treatment is intralesional steroids or surgical curettage.

Hyphema

A collection of red blood cells in the anterior chamber of the eye. Results from trauma to the ciliary body, iris, or anterior structures causes bleeding. May present after incurring trauma to the eye. Perform CT and US to exclude associated orbital fracture or foreign body. If no increase in IOP, limit activity for 72 hours. If increased IOP, topical oral ocular hypotensives, and cycloplegics (atropine).

A 45-year-old presents with a markedly tender nodule protruding from the edge of his upper eyelid. He states that this has been present for 12 hours. No discharge is seen. He denies visual problems. What is the most likely diagnosis?

A hordeolum (sty) is caused by an acute infection of the Zeis or Moll's glands of the eyelid. Symptoms include pain and tenderness. An "internal hordeolum" points to the inner conjunctiva of the lid and an "external hordeolum" points to the skin surface of the eyelid.

A 12 year old male presents with lesions on the palmar surface. They have been present for less than a week. Prior to the lesion appearing there was a small lesion that was assumed to be a bug bite. The lesion bleeds profusely with minimal provocation. What is the most likely diagnosis?

A pyogenic granuloma (PG) occurs at the site of minor trauma such as a bug bite or scratch. The PG grows rapidly forming a popular lesion with a collarette of scale. It will bleed profusely with minimal provocation. The only effective treatment is shave excision with curetting and ablation of the blood vessels that form the base of the PG.

A 35-year-old homeless male presents with a painful red right eye and decreased visual acuity, which occured over the past 48 hours. He doesn't recall any trauma, and prior to this his vision was good. His past history includes alcoholism and liver disease. On physical exam you notice a white opacity in the center of his right cornea. You are unable to do an ophthalmoscopic exam due to the opacity, and a fluorescein staining is strongly positive. What is the likely etiology of the white opacity?

A rapidly progressing central corneal ulcer must be considered first and treated aggressively. An ulcer will show fluorescein staining, due to a break in the corneal epithelium. Pseudomonas, strep pneumonia, herpes, and fungus must be considered as possible causes. An emergent ophthalmology consult can be sight saving.

Corneal Ulcer

A red eye with circumcorneal injection. Commonly a result of infection (bacterial, viral, fungal, or amoebas). Noninfectious causes inculde neurotrophic keratitis, exposure keratitis, severe dry eyes or allergic eye disease, and various ocular or systemic inflammatory disorder. May present with pain, photophobia, tearing, and reduced vision. PE reveals a red eye with mainly circumcorneal injection and purulent watery discarge.

A 24-year-old intoxicated male presents to the emergency department after being in a fight. He was punched in the nose, and now has mild deformity of the nose and some epistaxis. An x-ray reveals a fractured nasal bone. During his physical exam, what must you look for in order to prevent permanent destruction of his nasal septum?

A septal hematoma can cause ischemic necrosis of the nasal septal cartilage if not identified and drained. A deviated septum can be expected with a nasal bone fracture, and must be addressed by the otolaryngologist. Excessive epistaxis that does not resolve with direct pressure and anterior packing may indicate a posterior bleed.

dequervains tenosynovitis

APL and EPB -sx- pain along radial aspect of wrist -dx- finklestein test- pain with ulnar deviation or thumb extension -tx- thumb spica splint

A 26 year old male has dark urine 2 weeks after pharyngitis. UA shows red blood cell casts. What laboratory test can be drawn to support your suspicions?

ASO titer in patient with signs of post-streptococcal glomerulonephritis

central vertigo causes

AV malformation, tumor of brain stem or cerebellum, MS, vertebrobasilar migraine syndrome

Treatment for Open-angle glaucoma are many. Name the Topical Carbonic Anhydrase Inhibitors used for treatment:

Acetazolamide Dorzolamide

Describe some presentation symptoms of Acute Angle Closure Glaucoma

Aching, throbbing pain - severe "halos" around light Decreased visual acuity Injected conjunctiva* Cloudy/"steamy" cornea* Fixed, dilated pupil* HA/N/V

Intracranial benign tumor affecting CN VIII

Acoustic Neuroma

Slow growing benign tumor of the eight cranial nerve?

Acoustic neuroma/Schwannoma

Penicillin or cefuroxime PCN allergy: macrolides (erythromycin)

Acute Bacterial Pharyngitis treatment

Follows URI Purulent nasal discharge, facial pain, nasal obstruction Fever

Acute Bacterial Sinusitis clinical features

Viral URI that leads to eustachian tube dysfunction or blockage

Acute Otitis Media

Fever, ear pain, ear pressure, hearing impairment TM erythema

Acute Otitis Media clinical features

A 54-year-old male presents to you with a sudden onset of severe left eye pain and blurred vision. He states that he is nauseated and vomited twice. He denies any history of eye problems, other than having to wear glasses for reading. His only recent problem has been a mild upper respiratory infection, for which he is taking an over-the-counter decongestant. On physical exam, the vision in the affected eye is 20/200. His pupil is mid-sized and non-reactive to light, and the conjunctiva is markedly injected. What diagnosis must you consider first?

Acute angle closure is characterized by sudden onset of severe eye pain, blurred vision, nausea, vomiting, visual halos, and headache. Physical exam findings can include conjunctival injection, a rock hard ocular globe on palpation, a cloudy cornea, and a mid-position fixed pupil. Normal intraocular pressure is below 21mm Hg. Acute angle closure glaucoma can develop pressures of 60 to 80mm Hg.

Hordeolum

Acute development of a painful nodule or posture within a gland of the upper or lower lid. MCC by Staph, is not contagious. Internal - caused by inflammation or infection of the meibomian gland with pustular formation, situated deep to the palpebral margin. External (Sty) - caused by inflammation or infection of the glands of Moll and Zeis with pustular formation. Situated immediately adjacent to the palpebral margin. Presents with acute onset of pain and edema of the involved eye. Palpable, infuriated area in the involved eyelid, which has a central area of prudence with surrounding erythema. Tx with warm compress several times per day for 2 days. Topical ABX if infected. I&D may be required.

What is the DX? Acute vertigo associated with N/V tinnitus hearing loss in patient with no HX of vertigo.

Acute labyrinthitis (usually post infectious/viral)

What is the Dx? Acute deep ear pain erythematous bulging tympanic membrane middle ear fluid and hearing loss.

Acute otitis media

Sialadenitis

Acute pain and swelling of the parotid or submandibular gland. Ducal obstruction by mucus plus and salivary stasis leads to secondary infection, most commonly by Staph Aureus. May present with acute gland swelling, increased pain and swelling with meals, tenderness, and erythema of duct opening. Pus can be expressed from the duct. Occurs in the setting of dehydration or chronic illness. Dx with US or CT I&D is supportive variant develops. Tx with IV ABX with transition of orals. Less severe cases may be treated with orals alone. Stimulation of salivary flow with hydration, warm compress, sialogogues such as sour candies and gland massage. Treatment failure may indicate abscess formation, duct stricture, stone, or tumor.

How will Optic Neuritis present?

Acute vision loss (mono) Pain in affected eye Sluggish pupillary response

Mumps

Acute, self-limited, systemic viral illness characterized by the swelling of one or more of the salivary glands, typically the parotid glands May present with fever, headache, malaise with onset within 24 hours. May complain of ear pain, worse with chewing. May cause thyroiditis, pancreatitis (rare), or orchitis (50%) in postpubertal males, with 30% being bilateral. May also cause Oopheritis in postpubertal females. Associated with fever, nausea and vomiting, and low abdominal/pelvic pain. Will have a markedly enlarged parotid gland, submaxillary glands may be swollen and Stensons duct may be enlarged at the opening. Self limiting, supportive care, fluids, analgesics, ice packs or warm compresses to soothe swollen glandular areas.

Labyrinthitis

Acute, severe, continuous vertigo Cause is unknown. Pt may have had cold like symptoms that resolved around 1 week prior to onset of dizziness. Presents as acute onset of continuous vertigo lasting several days to a week. Hearing loss and tinnitus are present. Vertigo gradually improves, but hearing improvement varies. Tx with ABX for febrile patients with suspected bacterial infection. Vestibular suppressants (diazepam, meclizine) during acute phase.

Demographically speaking, in whom are cataracts *most common*?

Adults > 60 yrs old

You are asked to examine an 88-year-old female resident of a nursing home, who presents with a red eye. Her notes from the nursing home say that the patient has had this problem for six months, but now seems to be getting worse despite using daily artificial tears and occasional topical antibiotic drops. On physical exam you notice markedly injected conjunctiva to the right eye, with no discharge. The lower lid appears to be curled in toward the bulbar conjunctiva, with the eyelashes pointing inward. What is the name of this condition?

Aging causes a relaxation in the lower lid retractors, resulting in an entropion. This causes chronic irritation to the bulbar conjunctiva and corneal abrasions. Treatments include taping the lower lid to the cheek, botulinum toxin injection, or surgery.

A 45-year-old female presents with a sudden onset of vertigo, nausea, and vomiting. Upon physical exam, you note that she is holding on to the rails of the bed, and her pain gets worse when you attempt any movement of her head. Neurologic exam is grossly normal. Which combination of the following medications is indicated to treat the patient's symptoms?

All other combinations include an ototoxic medication: furosemide, gentamycin, aspirin, and cisplatin. Treatment of acute vertigo is more effective using a combination of vestibular suppressants (benzodiazepines), anti-emetics (prochlorperazine), and anticholinergics (diphenhydramine or scopolamine).

Your patient has symptoms consistent with perennial allergic rhinitis, and after performing a history and physical examination, you elect to perform an IgE-specific serum antibody test for both food and respiratory allergens. The results return and the patient does not have an IgE positive response to a single allergen tested, yet the patient's total serum IgE is elevated dramatically. What would be the most appropriate next step in the diagnosis and treatment of this patient?

Allergen-specific serum IgE testing is an easy and accurate method for determining the presence of atopic allergy, and with newer in vitro technology available, in vitro testing is at least equivalent to skin testing in efficacy. In vitro assays are safe, specific, cost-effective, and reproducible, and do not require the patient to be free of antihistamines and other medications that may interfere with skin testing. They are also easy and quick and are therefore preferred, especially in children and in anxious patients.

Allergic shiners Rhinorrhea, watery eyes, sneezing, nasal congestion Dry cough Pale mucosa Watery/clear discharge

Allergic Rhinitis clinical features

A patient presents to your office with a sudden onset of headache, right eye pain, decreased visual acuity, nausea and vomiting. His intraocular pressure is 47. Which of the following classes of medications are indicated for treatment of this condition?

Alpha agonists and Beta blockers Ophthalmic alpha-agonists (brimonidine) and beta blockers (timolol) decrease aqueous humor production, and decrease intraocular pressure. They facilitate aqueous flow through outflow tract and the canal of Schlemm.

A pt that complains of sudden loss of vision in one eye is experiencing what (term)?

Amaurosis Fugax

Which of the following diagnostic studies is indicated for a patient with amaurosis fugax?

Amaurosis fugax is a monocular vision loss that appears like a curtain passing over the eye, and comes from carotid artery disease. A CT of the head is indicated for lateralizing stroke symptoms. Intraocular pressure is taken for evaluation of chronic or acute glaucoma. A temporal artery biopsy is taken if giant cell arteritis is suspected. An ocular fluorescein angiogram is done to evaluate retinal disorders.

Topical abx for dacryocystitis?

Aminoglycosides (Gentamicin, Tobramycin) Fluoroquinolones (Cipro, Moxi, Oflox, Norflox) Macrolides (Erythromycin)

The American Academy of Pediatrics (AAP) recommends which of the following treatments for a two-year-old child with an acute otitis media who has a fever of 103.7˚F, had an ear infection two months ago, and in whom you suspect penicillin-resistant strep bacteria?

Amoxicillin-clavulanate 45 to 90 mg/kg, divided into BID dosing and administered for 10 days

An 18-year-old female presents with two weeks of severe sore throat and fatigue. Her exam shows an exudative tonsillitis. A mono-spot test is positive, and a rapid strep test is positive. Which of the following medications should be avoided?

Ampicillin should be avoided, because a high percentage of mononucleosis patients develop a fine, non-allergic maculopapular rash when given ampicillin class drugs. The remaining antibiotics are appropriate for treating group A strep. Prednisone is used to reduce the pain and inflammation associated with severe tonsillitis.

How is central loss vision tested?

Amsler Grid

This is an opthalmic emergency characterized by painful eye and loss of vision. Physical exam will reveal circumlimbal injection, steamy cornea, fixed mid-dilated pupil, decreased visual acuity and tearing.

Angle closure glaucoma

What're the 2 types of Glaucoma?

Angle-closure Open-Angle

Is the most common type of nosebleed an anterior or posterior bleed?

Anterior bleed

The source of most cases of epistaxis comes from what anatomic location?

Anterior nasal septum. 95 percent of epistaxis come from Kesselbach's plexus, which is a superficial, fragile group of arterioles and veins that are the most likely cause of nosebleeds. Five percent are posterior bleeds that originate along the sphenopalentine artery.

Which class of meds puts older pts at risk of angle-closure glaucoma?

Anti-ACh meds

Painful, round ulcers with yellow-gray centers and red halos Recurrent

Aphthous Ulcers clinical features

Topical corticosteroids

Aphthous Ulcers treatment

A mother presents with a 2-month-old infantwith a concern of bald spots in the child's scalp. Shestates that there were ulcerated areas present at birth that healedwithin a few days. What is the most likely diagnosis?

Aplasia cutis congenita is a rare condition that is present atbirth, and presents as asymptomatic ulcerations of the scalp. Theseulcerations heal with scarring in a matter of weeks. The cause isbelieved to be incomplete neural tube closure or cessation of skindevelopment of the embryo.

PAtient presents with a single or multiple painful, round ulcers with yellow gray centers and red halos. likely dx?

Apthous ulcers (canker sores, ulcerative stomatitis)

Critical: Retinal vessels

Artery to vein ratio is 2 to 3 without nicking or spasms

What is the most common pathogen in fungal otitis externa?

Aspergillus causes 90% of cases

What underlying disorders may be related to nasal polyps?

Asthma and ASA or NSAID allergy

Rheumatic fever

Autoimmune inflammatory process that develops as a sequela of streptococcal infection with extremely variable manifestations. The most severe complication is heart disease, usually occuring after multiple recurrences. May present with the following: Sore Throat (35-60%) Polyarthritis (75%) Carditis (30-60%, more common in children) Sydenham Chorea (25% of children, rare in adults) Erythema Marginatum (10% of children, rare in adults) Tx is primarily supportive at time of acute episode. Tx of primary infection is best prevention. IV immunoglobulin may help reduce risk of development of rheumatic heart disease.

What is the treatment for small tympanic membrane perforation?

Avoid water until healed in all types of rupture - oral antibiotics if caused by infection/AOM

Urinary Retention

Avoidance of oral antihistamines in patients with this condition.

Your 11 year old patient presents with acute onset of copious purulent dc from both eyes. They have a mild decrease in visual acuity and mild discomfort. The eys are "glued" shut upon awakening. Started 10 days ago. Likely dx?

BACTERIAL conjunctivitis

What is the most likely type of vertigo in a patient with vertigo who has a positive Dix Hallpike maneuver?

BPPV (Benign Paroxysmal positional vertigo)

A 48-year-old female complains of ear fullness, episodes of tinnitus, and vertigo. She also complains that her hearing is not as good as it used to be. She states that this has occurred sporadically over the past year. What is the most likely diagnosis?

BPPV is characterized by sudden vertigo, made worse with head position change, and accompanied by nausea and vomiting. Meniere syndrome is characterized by episodic severe vertigo, fluctuating sensorineural hearing loss, tinnitus, and ear "fullness." Pathologically, there is distention of the endolymphatic system throughout the inner ear, presumably due to dysfunction of the endolymphatic sac. Labyrinthitis is characterized by severe vertigo and hearing loss, and is likely a result of a viral inner ear infection. Vestibular neuronitis is also a result of a viral inner ear infection, with symptoms of severe vertigo, nausea, and vomiting, without hearing loss. Both labyrinthitis and vestibular neuronitis resolve in one to two weeks. Presbycusis is age related hearing loss.

Polymorphonuclear cells (PMNs)

Bacterial Conjunctivitis diagnostics

Acute ear pain hemotympanum hearing loss after flying?

Barotrauma

You are evaluating a patient who is complaining of facial drooping , and inability to close his eye. During the cranial nerve exam you notice he is unable to wrinkle his forehead. Based on this information what is the most likely diagnosis?

Bell's palsy affects cranial nerve VII, the facial paralysis conforms to the all branches of the peripheral nerve including the side of the face, eyelid and forehead muscles. An acute cerebrovascular accident would present only with a facial droop, the ability to close the eye and wrinkle the forehead would be preserved and there would likely be other focal weakness on physical exam. Horner's syndrome is miosis, ptosis and facial flushing and hyperhydrosis caused by abnormalities of the supercervical ganglion along the internal carotid artery.

What is a Pinguecula?

Benign, yellow growth that does not cross onto cornea

What're the tx options for open-angle glaucoma?

Beta-blockers (Timolol/Betaxolol) Prostaglandin Analogs Carbonic-Anhydrase Inhibitors (Acetazolamide) Laser Surgery

Viral conjunctivitis

Bilateral mucoserous discharge

Lesion of the optic chiasm will cause what visual symptoms?

Bitemporal heteronymous hemianopsia

Chronic inflammation of the eyelid margins

Blepharitis

Inflammation of the eyelids with no sign of palpable lumps...?

Blepharitis - Red, swollen eyelids

Swelling, misalignment of the eye Movement of globe restricted Double vision Subcutaneous emphysema Exophthalmos

Blow-Out Fracture clinical features

Direct trauma to the zygomatic prominence or the soft tissue around the orbit may cause which type of fx?

Blowout Fx

As a diver descends for a deep water dive, at about 10 feet of depth he begins to feel nausea, severe ear pain, and develops vertigo and vomiting. What is the most likely cause of his symptoms?

Boyle's law states that as a diver descends, the increasing external pressure causes an equal decrease in pressure in the middle ear, which must be equalized during the descent. If the middle ear pressure is not equalized, the tympanic membrane becomes severely retracted, due to the negative middle ear pressure. This can result in hemotympanum, hemorrhage, or tympanic membrane perforation. Ascent causes increased pressure in the middle ear as the external pressure is decreased. Equalization techniques must also be used to prevent a tympanic membrane perforation. Decompression sickness occurs on ascent, when nitrogen gas bubbles are forced into the middle ear, and vascular and lymphatic spaces.

Describe a Corneal Ulcer. What is a common cause?

Breakdown, necrosis & thinning of cornea. Herpes simplex keratitis

Name Alpha-agonists for OA Glaucoma tx

Brimonidine

A 25 yowm who wears soft contact lenses presents to the ER w a Corneal Abrasion x 4 hours after a fall. In addition to advising him not to wear contacts x 1 wk, which abx choice would appropriate? A- Bacitracin B- Mupirocin C- Tobramycin D- PCN

C- Tobramycin. Soft-contact lens wearers are at an increased risk for INFX w *Pseudomonas*, so abc-choice should cover for it. A Fluoroquinolone would also be appropriate.

sx of meniere's ds

CHRONIC and PROGRESSIVE: hearing loss, tinnitus, vertigo, N/V

Retinopathy in HIV pt is most likely caused by...?

CMV Toxoplasmosis

labyrinthitis

CN8, MC viral -sx-horizontal Nystagmus, continuous vertigo,+ hearing loss/tinnitus -tx- corticosteroids (methylipredinisone 22 days)

What diagnostic test can best confirm the diagnosis of mastoiditis?

CT scan

how to dx orbital cellulitis

CT scan

test of choice for mastoiditis

CT scan

test of choice to dx sinusitis

CT scan

what is the next step after visualizing papilledema on an ophthalmologic exam?

CT with contrast

dx of orbital cellulitis

CT, CBC, blood culture, drainage culture

What is first line tx for acute angle closer glaucoma?

Carbonic Anhydrase Inhibitors: Acetazolamide (Diamox) Methazolamide (Neptazane)

vertigo

Cardinal symptom of vestibular disease. "room is spinning" sensation. Presents with a sensation of motion without actual motion, or an exaggerated sense of motion. Peripheral variant - sudden onset, associated with tinnitus, hearing loss, and horizontal nystagmus. Central variant - Gradual onset, without audiometry symptoms, will have + Romberg. Dx with audiogram, electronystagmography, videonystagmography, MRI. Tx depents on etiology, meclizine may help.

Which of the following diagnostic studies is indicated for a patient with amaurosis fugax?

Carotid ultrasound

Opacity of the natural lens of the eye due to progressive increase in the proportion of insoluble protein

Cataract

Gradual diminution of vision Double vision, excess glare, fixed spots, reduced color perception Translucent, yellow discoloration of the lens

Cataract clinical features

"Clouding of the lens" is AKA . . .

Cataracts

Aphthus Ulcer

Cause is unknown, associated with HHV 6. Also referred to as ulcerative stomatitis. Part of the diagnostic criteria for SLE (lupus), seen in Crohn's, and affects woman more than men. Found on buccal and labia mucosa, sparing attached gingival and palpated mucosa. Single or multiple, recurring painful small round ulceration with yellow-gray fibrinoid center surrounded by a red halo. Tx with topical corticosteroids (triamcinolone 0.1%), topical diclofenac, mouthwashes containing amyloglucosidase and glucose oxidase. Short course of oral corticosteroids if necessary.

What is the potential problem with untreated orbital cellulitis?

Cavernous sinus thrombosis & Intracranial extension of orbital cellulitis. Can be life-threatening.

'fever over 38C, tender anterior cervical adenopathy, LACK OF COUGH and pharngotonsillar exudate' Presence of all 4 of these strongly suggest strep. What is this criteria called?

Centor criteria 3/4 rapid strep test sensitive > 90% 1/4 strep unlikely

Pharyngitis

Centor criteria. Penicillin

Sudden, painless, marked unilateral vision loss EMERGENCY

Central Retinal Artery Occlusion clinical features

Optic disc swelling 'blood and thunder' retina (dilation, hemorrhage, edema, exudates)

Central Retinal Vein Occlusion clinical features

Gradual onset Nystagmus NO auditory symptoms

Central Vertigo clinical features

is characterized by a sudden, painless vision loss. A cherry red spot is characteristic on the macula, along with pallor to the retina

Central retinal artery occlusion

_______ is vertigo due to a disease originating from the central nervous system (CNS). In clinical practice, it often includes lesions of cranial nerve VIII as well. Individuals with vertigo experience hallucinations of motion of their surroundings.

Central vertigo

Keflex

Cephalexin is a cephalosporin antibiotic used to treat certain infections caused by bacteria such as pneumonia and bone, ear, skin, and urinary tract infections. Antibiotics will not work for colds, flu, or other viral infections.

Painless, indurated lesions deep from palpebral margin Secondary to internal hordeolum

Chalazion

Sensory Hearing Loss

Chochlear deterioration due to loss of hair cells. Gradual, progressive high-frequency hearing loss with advancing age (presbycusis). Other causes include excess exposure, head trauma, systemic diseases. Usually not correctable but may be preventable. May be treated with corticosteroid therapy.

Perforated TM and chronic ear discharge +/- pain

Chronic Otitis Media clinical features

A 22-year old female presents to your office complaining of itchy red welts all over her body now fading. She is vacationing from Florida. She has no past medical history and her only medication is an oral contraceptive, but she did take an over-the-counter dipenhydramine four days ago on the flight from Florida to calm her nerves. The welts began after swimming in the ocean in New England three days ago, lasted a few hours, then disappeared spontaneously. They reoccurred Saturday morning again shortly after swimming, lasted a little longer, and again resolved. She relates that they were intensely itchy, red, and raised. She ate out at a restaurant and had seafood Saturday night, and thought that she might be allergic to the seafood, although she ate nothing new or unusual. From the history, which of the following is the likely cause of her urticaria?

Cold urticaria is a hypersensitivity to cold exposure (ie, wind, freezer compartments, water) resulting in histamine release. The hypersensitivity usually presents as localized redness, burning, pruritus, and urticaria in the exposed areas, or the response may progress to generalized systemic reaction, shock, and death. This condition may be familial or acquired. Familial cold urticaria is an autosomal dominant inflammatory disorder (including the Muckle-Wells syndrome), manifested as a burning sensation of the skin occurring about 30 minutes after exposure to cold. Acquired cold urticaria may be associated with medication (ie, griseofulvin) or with infection. Cold urticaria may occur secondarily to cryoglobulinemia or as a complication of syphilis. Most cases of acquired cold urticaria are idiopathic. For diagnosis, an ice cube is usually applied to the skin of the forearm for 4 to 5 minutes, then removed, and the area is observed for 10 minutes. As the skin rewarms, an urticarial wheal appears at the site that may be accompanied by itching. Second-generation antihistamines have been used as first-line treatment. Ebastine is also reported to safely and effectively prevent symptoms from acquired cold urticaria. Use of antileukotrienes in cold urticaria is anecdotal.

An 18-year-old female presents with a history of large open comedones and painful abscesses in both axillae. She has been treated by her primary care provider with oral antibiotics. A thorough physical exam may show which other affected sites?

Common areas of involvement in hydradenitis suppurtiva include the axillae, breasts, anogenital region, perineum, and scalp.

Barotrauma

Common risk factors include air travel, rapid altitude changes, and deep diving. Poor eustachian tube function decreases ability to equalize pressure. Most acute during airplane descent. Swallowing, yawning, or autoinflating ears during airplane descent helps. Slow descent while diving. Myringotomy for severe otalgia and hearing loss. Ventilating tubes for frequent episodes Avoid diving during episodes of URI, allergies, or TM perforation.

vincent's angina

Common, non-contagious infection of the gums with sudden onset. The main features are painful, bleeding gums, and ulceration of inter-dental papillae (the sections of gum between adjacent teeth). TRENCH MOUTH, Acute Necrotizing Ulcerative Gingivitis. Presents with severe gingival pain, Profuse gingival bleeding that requires little or no provocation, and Interdental papillae are ulcerated with necrotic slough. All three must be present to Dx. Tx with irrigation and debridement of infected areas. Oral ABX (metronidazole) for systemic involvement.

Gingival Hyperplasia

Commonly caused as a result of drug side effects of CCBs, Phenytoin (and many other anticonvulsants), and Cyclosporine.

Allergic Rhinitis

Condition is associated with pale or violaceous nasal turbinates. Exposure to airborne allergens activates humoral (B-cell) and cytotoxic (T-cell) responses. IgE causes release of inflammatory mediators (histamine, prostaglandins, kinins). Seasonal variant is associated with pollens and spores. Perennial variant is associated with dust, mites, pollution, and pet dander. May present with clear rhinorrhea, sneezing, tearing, conjunctival erythema, eye irritation, and pruritus. Accompanying symptoms may include cough, bronchospasm, and eczematous dermatitis. Yellow boggy masses of nasal polyps seen with chronic cases. Strong family history of atopy or allergies. Allergic Salute Dx with RAST - Radio allergosorbent test. Tx by reducing exposure referral to allergist for immunotherapy for resistant cases. Use of antihistamines, intranasal corticosteroids and saline, antileukotriene and anticholinergic agents.

What type of hearing loss will the Weber and Rinne tests assist in diagnosing?

Conductive

What type of hearing loss is experienced with cerumen impaction?

Conductive hearing loss

What type of hearing loss occurs with tympanic membrane rupture?

Conductive hearing loss

Which of the following will cause conductive hearing loss? A. Mumps B. Syphilis C. Multiple sclerosis D. Otitis media E. Medications

Conductive hearing loss is the result of blockage of sound waves from the external canal to the inner ear. Causes include cerumen, middle ear effusion, otitis media, and occiscle disruption. Multiple sclerosis causes VIIIth cranial nerve disruption and neural hearing loss. Mumps and syphilis can cause sensoryneural hearing loss.

Branchial Cleft Cyst

Congenital epithelial cysts, which arise on the lateral part of the neck from a failure of obliteration of the second branchial cleft in embryonic development, related to gill apparatus of fish. Commonly presents as a solitary, painless mass in the neck of a child or a young adult. A history of intermittent swelling and tenderness of the lesion during upper respiratory tract infection may exist. Discharge may be reported if the lesion is associated with a sinus tract. Tx with ABX if abscess if suspected. Definitive treatment is surgical excision.

A 35-year-old homeless male presents with a painful red right eye and decreased visual acuity, which occured over the past 48 hours. He doesn't recall any trauma, and prior to this his vision was good. His past history includes alcoholism and liver disease. On physical exam you notice a white opacity in the center of his right cornea. You are unable to do an ophthalmoscopic exam due to the opacity, and a fluorescein staining is strongly positive. What is the likely etiology of the white opacity?

Corneal Ulcer

Usu. caused by minor trauma such as from a fingernail, contact lens, eyelash or foreign body. Patient will present with pain and sensation of a foreign body and it can be accompanied by photophobia, tearing, injection and blepharospam.

Corneal abrasion

Patient presens to your clinic with pain, photophobia an tearing. Examination reveals circumcorneal injection and watery to purulent discharge. Fluorescein staining will reveal a dense corneal infiltrate with overlying epithelial defect. Dx? (corneal abrasian, blehpariits, corneal ulcer)

Corneal ulcer

What is inflammation of nasolacrimal duct?

Dacryoadenitis

An obstruction/INFX of the Lacrimal sac is AKA . . .

Dacryocystitis

Inflammation of the lacrimal gland caused by obstruction

Dacryocystitis

Obstruction of the lacrimal system

Dacryostenosis

How does a pt present when having orbital cellulitis

Decreased ROM Pain with eye movement Proptosis (Forward eye protrusion)

What does shingles in the eye look like w fluorescein stain?

Dendritic (branching) ulcers

A 28-year-old male presents with a rashon his hands that is unresponsive to topical steroids. On physicalexam you notice periocular violaceous erythema and edema. He also exhibitssignificant proximal muscle weakness. What is the most likely diagnosis?

Dermatomyositis is an autoimmune disease that targets the skinand skeletal muscle. Skin lesions usually consist of a periorbitalheliotrope rash, which can have associated edema. There can alsobe flat topped violaceous papules (Gottron papules) located on theneck, shoulders, and knuckles. Periungual erythema with telangiectasiasmay also occur. Possible muscular symptoms include muscle tenderness,muscle atrophy, and progressive proximal muscle weakness.

A 16-year-old male was hit on the left side of his face by a line drive baseball. Marked swelling is noted externally to the left eye. There was no loss of consciousness. Upon physical exam, he complains of diplopia during extraocular motion testing. Enophthalmos is noted, as well as decreased sensation of the left cheek. Plain x-rays of the face demonstrate an air-fluid level in the left maxillary sinus, and a fracture of the orbit. Based on this information, what is the most likely diagnosis?

Diplopia is common in an orbital blow out fracture, due to entrapment of the inferior rectus and inferior oblique muscles. Loss of infraorbital sensation occurs from disruption or swelling of the infraorbital nerve. A Le Fort I fracture describes a transverse fracture separating the body of the maxilla from the pterygoid plate and nasal septum. A Le Fort II fracture describes a pyramidal through the central maxilla and hard palate. Movement of the hard palate and nose occurs, but not the eyes. A Le Fort III fracture describes a craniofacial disjunction, wherein the entire face is separated from the skull due to fractures of the frontozygomatic suture line, across the orbit and through the base of the nose, and ethmoids. The entire face shifts, with the globes held in place only by the optic nerve.

benign paroxysmal positional vertigo

Dix-Hallpike to Dx Epleys maneuver to Tx lasts seconds to minutes otoliths are stuck in semicircular canals tx: meclizine

drusen deposits

Drusen are tiny yellow or white deposits in a layer of the retina called Bruchs membrane. They are the most common early sign of dry age-related macular degeneration.

While you are doing a funduscopic exam on an 80-year-old female with progressive vision loss, you notice drusen formations on her retinas. What is the most likely diagnosis?

Drusen are yellow colored collagen deposits in Bruch's membrane of the retina. They can be diffuse, discrete, or confluent. Retinal pigment changes and atrophy are see in "dry" macular degeneration. "Wet" macular degeneration demonstrates choroidal neovascularization, or serous retinal pigment hemorrhages and retinal detachments.

centor criteria

Dx and Tx GAS: fever cervical adenopathy lack of cough tonsillar exudate 2+ need culture and 3+ strep likely

Conductive Hearing Loss

Dysfunction of external of middle ear impairing passage of vibrations to inner ear. Obstruction (cerumen), mass loading (effusion), stiffness (otosclerosis), discontinuity (ossicular disruption). Correctable with medical or surgical therapy. May be treated with amplification devices if not medically correctable.

Retinal artery occlusion

ER!

closed angle glaucoma

ER! loss of vision, pain, circumlimbal injection, steamy cornea, fix dilated pupil, decreased visual acuity, N/V, diaphoresis, HALOS

A 12-year-old male begins to sneeze, and develops itchy, watery eyes about 15 minutes after being exposed to a cat. There is no respiratory difficulty. What phase of allergic response is he in?

EXPLANATION: The humeral or early phase occurs in the first 15 minutes of being exposed to an allergen. The symptoms are caused by release of histamine. The cellular phase is the late phase, and occurs after four to six hours of allergen exposure. Seasonal allergic rhinitis occurs in a regular pattern each year, corresponding to pollen exposure. Perennial rhinitis occurs year round, and may be more linked with indoor allergen exposures.

Lid everts secondary to trauma, infection, palsy of the facial nerve

Ectropion

A 65-year-old female presents with a red irritation in her right eye. She states that this has been occurring intermittently for about two years. She also states that her eyelids are "droopy," and that she needs plastic surgery. On physical exam you notice a diffusely injected conjunctiva and an outwardly tilted lower eyelid. What is the most likely diagnosis for the abnormal physical finding?

Ectropion Ageing causes a relaxation of the obicularis oris muscle, and will cause the lower eyelid to sag outwardly. This prevents the lower lid from protecting the eye, and frequently results in exposure conjunctivitis and keratitis. Treatment is surgical.

Sore throat, fever, drooling Children: tripod or sniffing posture to improve air exchange 'Thumb sign' on lateral xrays

Epiglottitis clinical features

Most common cause of Mononucleosis?

Epstein-Barr Virus (EBV)

Acoustic Neuroma

Evaluate for this intracranial mass in patients with unilateral or asymmetric sensorineural hearing loss. Benign lesion which arise within the internal auditory canal and eventually grow to compress the pons, resulting in hydrocephalus. Presents with unilateral hearing loss, deteriorating speech, disequilibrium. Dx with enhanced MRI. Tx with observation, microsurgical excision, stereotactic radiotharpy. Bevacizumab for treatment of tumors in patients with neurofibromatosis type 2.

A 21 year old female presents with a solitary firm right cervical node for the last 2 months. What is the most appropriate next step?

Excisional biopsy and pathology to rule out Hodgkin lymphoma

Diagnosis/ENT/Ophthalmology A 56 year-old female presents complaining of intense left eye pain associated with unilateral headache, nausea, and colored rings around lights. On examination you note decreased visual acuity, a pupil that is fixed and mid-dilated, and ciliary flushing. Which of the following is the most likely diagnosis? A. Acute glaucoma B. Migraine C. Episcleritis D. Acute uveitis

Explanations (c) A. Acute glaucoma is an ocular emergency that presents as an acutely painful eye and elevated intraocular pressure. Patients typically complain of acute eye pain associated with unilateral headache, nausea/vomiting, cloudy vision, and colored rings around lights. On exam the pupil is fixed and mid dilated with prominent ciliary flush. (u) B. Migraine headaches have associated unilateral headache and nausea however there would be no pupillary changes. (u) C. Episcleritis is an inflammation of the thin layer of connective tissue between the conjunctiva and sclera. Episcleritis resembles conjunctivitis but is a more localized process and discharge is absent. (u) D. Acute uveitis is frequently due to systemic disorders associated HLA-B27-related conditions ankylosing spondylitis, reactive arthritis, psoriasis, ulcerative colitis, and Crohn's disease. The pupil is usually small, inflammatory cells and flare within the aqueous are present.

Diagnosis/ENT/Ophthalmology A patient presents with a nontender, painless, nodule involving a meibomian gland. Which of the following is the most likely diagnosis? A. Chalazion B. Dacryocystitis C. Entropion D. Hordeolum

Explanations (c) A. Chalazion is characterized by a hard, nontender swelling on the upper or lower lid with redness and swelling of the adjacent conjunctiva and is due to granulomatous inflammation of a meibomian gland. (u) B. Dacryocystitis is an infection of the lacrimal sac due to obstruction of the nasolacrimal system. (u) C. Entropion is an outward turning of the lower lid. (u) D. A hordeolum is a bacterial inflammation of the base of the eyelash.

Clinical Intervention/ENT/Ophthalmology There is considerable debate about the use of tympanostomy tubes in the management of recurrent otitis media in children. Tympanostomy tube placement has been proven to Answers A. improve hearing. B. prevent mastoiditis. C. prevent recurrence of effusion. D. prevent delayed language development.

Explanations (c) A. Hearing is improved with tympanostomy tubes by eliminating middle ear effusion when the tubes are functioning properly. (u) B. Mastoiditis is prevented by early treatment of otitis media with antibiotics. (u) C. Effusion can occur even with tympanostomy tubes in place. (u) D. Tympanostomy tubes have not been proven to prevent delayed language development.

Diagnosis/ENT/Ophthalmology A patient presents with eye pain and blurred vision. Snellen testing reveals vision of 20/200 in the affected eye and 20/20 in the unaffected eye. Fluorescein staining reveals the presence of a dendritic ulcer. Which of the following is the most likely diagnosis? A. Viral keratitis B. Fungal corneal ulcer C. Acanthamoeba keratitis D. Bacterial corneal ulcer

Explanations (c) A. Herpes Simplex virus is a common cause of dendritic ulceration noted on fluorescein staining. (u) B. Fungal corneal ulcers have an indolent course with intraocular infection being common but fluorescein staining is negative for a dendritic pattern. (u) C. Acanthamoeba keratitis has a waxing and waning course over several months and has no fluorescein staining in a dendritic pattern. (u) D. Bacterial corneal ulcers can progress aggressively resulting in corneal perforation. Fluorescein staining does not occur in a dendritic pattern.

Diagnosis/ENT/Ophthalmology Which of the following is a staphylococcal infection characterized by a localized red swollen and acutely tender abscess of the upper or lower eyelid? A. Hordeolum B. Uveitis C. Chalazion D. Dacryocystitis

Explanations (c) A. Hordeolum (stye) is a staphylococcal infection characterized by a localized red swollen and acutely tender abscess of the upper or lower eyelid. (u) B. Uveitis is an intraocular inflammation involving the uveal tract. (u) C. Chalazion is a granulomatous inflammation of the meibomian gland. (u) D. Dacryocystitis is an infection of the lacrimal sac due to obstruction of the nasolacrimal system.

History & Physical/ENT/Ophthalmology When performing a Weber test on a patient with impacted cerumen in the right canal, the sound should be A. referred to the right ear. B. referred to the left ear. C. equal in both ears. D. louder with air conduction.

Explanations (c) A. In unilateral conductive hearing loss, the sound is referred to the impaired ear. (u) B. See A for explanation. (u) C. See A for explanation. (u) D. Bone conduction as noted with the Rinne test is louder than air with conductive hearing loss.

Clinical Therapeutics/ENT/Ophthalmology Which of the following may precipitate acute angle-closure glaucoma? A. metoclopramide B. timolol C. glyburide D. acetazolamide

Explanations (c) A. Metoclopramide and other drugs with high anticholinergic effects may precipitate acute angle-closure glaucoma from pupillary dilation. (u) B. Timolol, a beta-antagonist, is used in the treatment of acute angle-closure glaucoma. (u) C. Glyburide has no relationship to glaucoma. (u) D. Acetazolamide, a carbonic anhydrase inhibitor, may suppress the production of aqueous humor by 40-60% and is used in the emergency treatment of glaucoma.

Clinical Intervention/ENT/Ophthalmology In patents with diabetic retinopathy, what clinical intervention is most successful in preserving vision? A. Panretinal laser photocoagulation B. Iridectomy C. Radial keratotomy D. Vitrectomy

Explanations (c) A. Panretinal laser photocoagulation is indicated for preservation of vision in patients with diabetic retinopathy. (h) B. Iridectomy is of no value in preserving the retina and iridectomy is harmful in this situation due to the trauma it causes to the eye. (h) C. Radial keratotomy is indicated to correct myopia. This surgery destroys normal eye architecture and has no benefit in diabetic retinopathy. (h) D. Vitrectomy is indicated for treatment of retinal tears and not to preserve an intact retina.

History & Physical/ENT/Ophthalmology Whispered voice test on a patient reveals decreased hearing in the left ear. Which of the following would be most consistent with conductive hearing loss in the left ear? A. Sounds best heard in the left ear on Weber test. B. Air conduction longer than bone conduction in the left ear on Rinne test. C. Sound best heard in the right ear on Weber test. D. Bone conduction longer than air conduction in the right ear.

Explanations (c) A. Sound best heard in the ear with decreased hearing on Weber test (in this case, the left ear) is indicative of conductive hearing loss. (u) B. With conductive hearing loss, bone conduction should be heard as long as or longer than air conduction of sound in the effected ear. Air conduction lasting longer than bone conduction of sound would indicate sensorineural hearing loss. (u) C. Sound best heard in the ear with unaffected hearing on Weber test (in this case, the right ear) is indicative of sensorineural hearing loss. (u) D. With conductive hearing loss, bone conduction should be heard as long as or longer than air conduction of sound in the effected ear. The right ear showed normal hearing on physical exam.

Scientific Concepts/ENT/Ophthalmology Dental caries are caused by which of the following organisms? A. Streptococcus mutans B. Streptococcus pyogenes C. Staphylococcus epidermidis D. Staphylococcus aureus

Explanations (c) A. Streptococcus mutans is the principle organism that helps to demineralize the enamel. (u) B. See A for explanation. (u) C. See A for explanation. (u) D. See A for explanation.

Diagnosis/ENT/Ophthalmology A 45 year-old smoker presents with a sore mouth and increasing difficulty eating for two weeks. Physical examination reveals a 1 cm white lesion on the buccal mucosa that cannot be rubbed off. Which of the following is the most likely diagnosis? A. Oral cancer B. Oral candidiasis C. Aphthous ulcer D. Necrotizing ulcerative gingivitis

Explanations (c) A. The presence of leukoplakia in a smoker over the age of 40 should be biopsied to rule out the presence of oral cancer. (u) B. Oral candidiasis presents with white patches. Unlike leukoplakia, the patches easily rub off. (u) C. While aphthous ulcers are commonly found on the buccal mucosa, they are usually 1 to 2 mm round ulcerative lesions. (u) D. Necrotizing ulcerative gingivitis is common in young adults under stress. Clinically, it presents with painful acute gingival inflammation and necrosis.

Clinical Therapeutics/ENT/Ophthalmology A 23 year-old sexually active female presents with a 4 day history of painless bilateral eye exudates which she describes as copious. Visual acuity is 20/20, generalized conjunctival inflammation with sparing of the cornea is noted on physical examination. Gram stain of the exudate reveals gram negative diplococci. Appropriate management of this case is A. ceftriaxone (Rocephin). B. polymyxin ophthalmic drops (Aerosporin). C. ciprofloxacin (Cipro). D. doxycycline (Doryx).

Explanations (c) A. With sparing of the cornea, as in this case, a single 1 gram IM dose of ceftriaxone is sufficient treatment for ophthalmic gonorrhea. If the cornea is involved, 5 days of IM ceftriaxone would be required. (u) B. Polymixin is ineffective against gonococcus. (u) C. Oral ciprofloxacin is not used in cases of gonococcal conjunctivitis. (u) D. Doxycycline is ineffective against gonococcus.

Diagnostic Studies/ENT/Ophthalmology A 32 year-old carpenter complains of right eye irritation all day after driving a metal stake into the ground with his hammer. He states that "something flew into my eye." Visual acuity is 20/20. Pupils are equal, round, reactive to light and accommodation. Extraocular movements are intact. There is minimal right corneal injection. No foreign body is noted with lid eversion. Fluorescein stain reveals a tiny pinpoint uptake in the area of the corneal injection. Which of the following is the most appropriate diagnostic test at this stage? Answers A. MRI B. X-ray orbits C. Applanation tonometry D. Fluorescein angiography

Explanations (h) A. MRI should never be used when there is suspicion of an iron-containing intraocular foreign body. (c) B. Orbital x-rays or CT scan will be most helpful in identifying an intraocular metallic foreign body. (u) C. Tonometry is used to evaluate intraocular pressure, but not the presence of intraocular foreign bodies. (u) D. Fluorescein angiography is used to evaluate vessels of the eye, not intraocular foreign bodies.

Clinical Intervention/ENT/Ophthalmology A 16 year-old male involved in a fight sustained a laceration to his right upper eyelid. He is unable to open his eye, and a possible laceration of the globe is suspected. Which of the following is the next step? A. Use a slit lamp to determine the extent of the injury. B. Use fluorescein strips to determine the extent of injury. C. Apply a metal eye shield and refer to an ophthalmologist. D. Apply antibiotic ointment to the lid and recheck in 24 hours.

Explanations (h) A. See C for explanation. (h) B. See C for explanation. (c) C. Protect the eye from any pressure with a rigid metal eye shield and refer for immediate ophthalmologic consultation. Avoid unnecessary actions that would delay treatment or cause further injury. (h) D. See C for explanation.

Diagnostic Studies/ENT/Ophthalmology Which of the following is most helpful in the diagnosis of a retropharyngeal abscess? A. CBC with differential B. fever and a muffled voice on examination C. CT of the neck with contrast D. history of a recent throat infection

Explanations (u) A. A CBC with differential would identify an infection but not specifically a retropharyngeal abscess. (u) B. The presence of fever and a muffled voice on physical exam is not specific for a retropharyngeal abscess. (c) C. CT of the neck is considered the "gold standard" for the diagnosis of a retropharyngeal abscess. (u) D. A recent throat infection is not specific for a retropharyngeal abscess.

Diagnostic Studies/ENT/Ophthalmology Which of the following diagnostic studies would be most helpful in diagnosing a retropharyngeal abscess? A. CBC B. Neck CT scan C. Rapid strep screen D. Heterophile antibody

Explanations (u) A. A CBC would be an important test, but it is not definitive for peritonsillar abscess. (c) B. A neck CT scan would identify a peritonsillar abscess. (u) C. A rapid strep screen may have been performed prior to placement on antibiotics, but does not provide a definitive diagnosis for a peritonsillar abscess. (u) D. A heterophile antibody is not indicated in the diagnosis of a retropharyngeal abscess.

History & Physical/ENT/Ophthalmology Which of the following are normal findings in a Weber test? Answers A. The tympanic membrane is movable with pneumatic otoscopy. B. The tympanic membrane is pearly gray with a sharp cone of light with apex at the umbo. C. Sound is heard equally in both ears when a vibrating tuning fork is placed on the mid forehead. D. Air conduction is greater than bone conduction when a vibrating tuning fork is moved from the mastoid bone to close to the ear canal.

Explanations (u) A. A movable tympanic membrane indicates there is no effusion, and is not the Weber test. (u) B. The tympanic membrane is evaluated by direct observation with an otoscope, and is not the Weber test. (c) C. A normal Weber test means there is no lateralization of sound perception when a vibrating tuning fork is placed on the mid forehead. (u) D. A normal Rinne test means that tuning fork vibration is heard longer through the air than the bone.

Clinical Therapeutics/ENT/Ophthalmology A 13 year-old presents with pain in his right ear and loss of hearing since yesterday. He has never had an episode like this before. On exam vital signs are T- 38°C P- 70/minute R- 18/minute BP- 90/60 mmHg. Neck is supple, without lymphadenopathy. Right tympanic membrane is not visible; the canal is swollen, with small amount of exudate noted. There is tenderness of the external ear, especially with gentle traction of the tragus. Left tympanic membrane is normal, and the canal is clear. Oropharynx is normal. Which of the following is the most appropriate topical treatment for this patient? Answers A. Acetic acid solution B. Erythromycin solution C. Cortisporin otic suspension D. Gentamycin drops

Explanations (u) A. Acetic acid solution may be used, but the burning sensation associated with it reduces patient compliance. It is usually used in combination with a topical antimicrobial. (u) B. Erythromycin solution is not used to treat otitis externa. (c) C. Cortisporin otic suspension is a combination antimicrobial (Neomycin and Polymyxin B) and steroid (hydrocortisone) that is effective to use in the treatment of otitis externa. In addition, the suspension is safe to use in suspected cases of tympanic perforation. (u) D. Gentamycin drops are not used in the treatment of otitis externa.

Diagnosis/ENT/Ophthalmology A 52 year-old female presents with complaints of intermittent episodes of dizziness, tinnitus, and hearing loss in the right ear for 6 months. She describes the dizziness as the "room spinning around her," with the episodes typically lasting for 2 to 4 hours. Physical examination reveals horizontal nystagmus and right ear hearing loss, but the remainder of the examination is unremarkable. Which of the following is the most likely diagnosis? A. Acute labyrinthitis B. Positional vertigo C. Acoustic neuroma D. Ménière's syndrome

Explanations (u) A. Acute labyrinthitis typically presents with an acute onset of continuous vertigo that lasts several days to a week and is associated with nausea and vomiting. It does not have any associated auditory or neurologic symptoms. (u) B. Positional vertigo occurs following changes in head positioning with very brief, less than 1 minute, episodes. Nystagmus occurs following the position change. (u) C. Acoustic neuroma typically presents with hearing loss and tinnitus. The neuroma grows slowly and central compensatory mechanisms can prevent or minimize the vertigo. Vertigo, when present, is continuous and not episodic. (c) D. Ménière's syndrome usually presents with episodes of vertigo that last from 1 to 8 hours, sensorineural hearing loss and tinnitus.

Clinical Therapeutics/ENT/Ophthalmology The best course of action for a patient with a bothersome inflamed pingueculae (pingueculitis) is A. antibiotic drops. B. excision. C. Visine drops. D. no treatment.

Explanations (u) A. Antibiotic drops have no benefit with pingueculitis. (h) B. Excision is indicated for a pterygium that is threatening vision. (u) C. Visine drops will not do anything, but artificial tears may be beneficial. (c) D. With pingueculitis, no treatment is necessary; a short course of NSAID drops or steroids may help.

Clinical Therapeutics/ENT/Ophthalmology A 19 year-old college student complains of a sore throat for over a week, with fever and general malaise. On exam T-38°C P-70/minute R-20/minute BP-110/76 mmHg. The patient is alert and oriented x 3. The skin is warm, dry, and without rash. The TMs have a normal light reflex and the canals are clear. The oropharynx is inflamed, with bilaterally enlarged tonsils, and a small amount of exudate. The neck is supple, with anterior cervical adenopathy. The lungs are clear. The heart has a regular rhythm without murmurs. The abdomen is soft, nontender and a spleen tip is palpable. The labs reveal a negative rapid strep screen and positive Monospot. The WBC count is 9,000/microliter with a differential of 40% atypical lymphocytes, 35% lymphocytes, 5% monocytes, 10% eosinophils, and 10% neutrophils. Which of the following is the most appropriate treatment? Answers A. Penicillin B. Erythromycin C. Acetaminophen D. Acyclovir

Explanations (u) A. Antibiotics are not indicated in the treatment infectious mononucleosis, or Ebstein-Barr virus infections. (u) B. See A for explanation. (c) C. Aspirin or acetaminophen may be used to treat fever and pain associated with infectious mononucleosis, or EBV infection. (u) D. Acyclovir is not approved for use in treatment of EBV, although it is active against the EBV in vitro and in vivo. It may be used in certain patients with AIDS, but has not been shown to affect the outcome of EBV in these patients.

Clinical Therapeutics/ENT/Ophthalmology A 19 year-old college student complains of a sore throat for over a week, with fever and general malaise. On exam T- 38 degrees C, P-70/minute, RR-20/minute, BP-110/76 mmHg. The skin is warm, dry, and without rash. The TMs have a normal light reflex and the canals are clear. The oropharynx is inflamed, with bilaterally enlarged tonsils, and a small amount of exudate. The neck is supple, with anterior cervical adenopathy. The abdomen is soft, nontender and a spleen tip is palpable. The labs reveal a negative rapid strep screen and positive heterophil antibody. The WBC count is 9,000/microliter with a differential of 40% atypical lymphocytes, 35% lymphocytes, 5% monocytes, 10% eosinophils, and 10% neutrophils. Which of the following is the most appropriate treatment? A. Penicillin B. Erythromycin C. Acetaminophen D. Acyclovir

Explanations (u) A. Antibiotics are not indicated in the treatment infectous mononucleosis, or Ebstein-Barr virus (EBV) infections. (u) B. See A for explanation. (c) C. Acetaminophen may be used to treat fever and pain associated with infectious mononucleosis, or EBV infection. (u) D. Acyclovir is not approved for use in treatment of EBV, although it is active against the EBV in vitro and in vivo. It may be used in certain patients with AIDS, but has not been shown to affect the outcome of EBV in these patients.

Clinical Therapeutics/ENT/Ophthalmology A 2 year-old female presents with purulent nasal discharge bilaterally with fever and cough for several days. Her 80 mom had taken her out of daycare for a similar occurrence 2 months ago, that was treated with Amoxicillin. Exam further reveals halitosis and periorbital edema. Treatment should be initiated with which of the following? A. Antihistamines B. Ribavirin (Rebetol) C. Intranasal corticosteroids D. Amoxicillin-clavulanate (Augmentin)

Explanations (u) A. Antihistamines and intranasal corticosteroids have not been adequately studied in children to prove they make a difference in treating recurrent sinusitis. (u) B. Ribavirin is approved for the treatment of RSV infection. (u) C. See A for explanation. (c) D. High dose amoxicillin-clavulanate is the treatment of choice for resistant bacterial sinusitis, especially in children presenting with risk factors (daycare attendance, previous antibiotic treatment 1-3 months prior, age younger than 2 years).

Clinical Therapeutics/ENT/Ophthalmology A 12 year-old presents with complaint of both eyes "watering." He also complains of sinus congestion and sneezing for two weeks. On exam vital signs are T-38°C, P- 80/minute, and RR-20/minute. The eyes reveal mild conjunctival injection bilaterally, clear watery discharge, and no matting. Pupils are equal, round, and reactive to light and accommodation. The extraocular movements are intact. The funduscopic exam shows normal disc and vessels. The TMs are normal and the canals are clear. The nasal mucosa is boggy, with clear rhinorrhea. Which of the following is the most helpful pharmacologic agent? Answers A. Artificial tears B. Tobramycin drops C. Erythromycin ointment D. Naphazoline (Naphcon-A) drops

Explanations (u) A. Artificial tears are used for dry eyes. (u) B. Tobramycin drops and erythromycin ointment are used to treat bacterial infections. (u) C. See B for explanation. (c) D. Naphazoline is a topical antihistamine that relieves symptoms of allergic conjunctivitis.

Clinical Therapeutics/ENT/Ophthalmology A 20 year-old woman presents with a 3-day history of sneezing, watery nasal discharge, and a nonproductive cough. Her throat was sore for the first 2 days, and she now complains of fatigue and difficulty breathing because of her "stuffy nose." Which of the following is most likely to improve this patient's status? Answers A. Ascorbic acid B. Amoxicillin C. Pseudoephedrine D. Chlorpheniramine

Explanations (u) A. Ascorbic acid is of no proven benefit in the treatment of viral URI. (u) B. The use of antibiotics is inappropriate for viral infections. (c) C. A decongestant will relieve the nasal congestion and stuffy nose symptoms. (a) D. An antihistamine may relieve the sneezing, but it may thicken secretions, making them difficult to clear.

Clinical Therapeutics/ENT/Ophthalmology A 22 year-old woman presents with sneezing, runny nose, postnasal drip, and nasal congestion for the last week. She says this happens every spring. She is not allergic to any medications. Which of the following is the most appropriate pharmacologic treatment for this patient? A. Azithromycin (Zithromax) B. Phenylephrine (Neo-synephrine) C. Nedocromil D. Pseudoephedrine

Explanations (u) A. Azithromycin is used to treat bacterial infections not allergic disorders. (u) B. Neo-synephrine is not indicated in the treatment of allergic rhinitis. (c) C. Nedocromil inhibits mast cell degranulation and is an effective treatment for allergic rhinitis. It may take 2-6 weeks for full therapeutic effect. (u) D. Pseudoephedrine is a decongestant that may relieve the nasal congestion, but has no effect on the allergic response.

Diagnosis/ENT/Ophthalmology A patient is evaluated in the office with a red eye. The patient awoke with redness and a watery discharge from the eye. The eyelids were not matted together. Examination reveals a palpable preauricular node. Which of the following is the most likely diagnosis? A. bacterial conjunctivitis B. viral conjunctivitis C. allergic conjunctivitis D. gonococcal conjunctivitis

Explanations (u) A. Bacterial conjunctivitis is associated with purulent, not watery eye discharge. (c) B. Viral conjunctivitis is associated with copious watery discharge and preauricular adenopathy. (u) C. Allergic conjunctivitis is associated with symptoms limited to the conjunctiva with hyperemia and edema. (u) D. Gonococcal conjunctivitis is associated with copious purulent discharge and no preauricular adenopatthy.

Health Maintenance/ENT/Ophthalmology Which of the following is the leading cause of permanent visual loss in a patient over the age of 75? A. Blepharitis B. Cataracts C. Central retinal artery occlusion D. Macular degeneration

Explanations (u) A. Blepharitis is a chronic bilateral inflammatory condition of the lid margins. (u) B. Cataracts are the clouding of the lens sufficient to reduce vision. Most develop slowly as a result of aging, leading to gradual impairment of vision. (u) C. Central retinal artery occlusion presents as a rare cause of sudden profound monocular visual loss. (c) D. Age-related macular degeneration is the leading cause of permanent visual loss in the older population. The exact cause is unknown, but the prevalence increases with each decade over age 50 years.

Clinical Intervention/ENT/Ophthalmology A 13 year-old boy with leukemia presents with epistaxis for 2 hours. The bleeding site appears to be from Kiesselbach's area. The most appropriate intervention is A. electrocautery of the bleeding site. B. silver nitrate application. C. posterior nasal packing. D. intranasal petrolatum gauze.

Explanations (u) A. Cautery is not used because the edges of the cauterized area may begin to bleed. (u) B. Silver nitrate is not used in children because it increases the risk for nasal septal perforation. (u) C. Posterior nasal packing is indicated for posterior bleeds in the inferior meatus. (c) D. Petrolatum gauze will provide pressure to the bleeding point while the cause of bleeding is corrected.

Diagnosis/ENT/Ophthalmology A 59 year-old male complains of "flashing lights behind my eye" followed by sudden loss of vision, stating that it was "like a curtain across my eye." He denies trauma. He takes Glucophage for his diabetes mellitus and atenolol for his hypertension. He has no other complaints. On funduscopic exam, the retina appears to be out of focus. Which of the following is the most likely diagnosis? Answers A. Central retinal vein occlusion B. Retinal artery occlusion C. Retinal detachment D. Hyphema

Explanations (u) A. Central retinal vein occlusion causes painless, variable loss of vision. Exam shows retinal hemorrhages in all quadrants and edema of the optic disk. (u) B. Retinal artery occlusion presents with sudden, painless loss of vision. Exam shows pale retina with normal macula, seen as a cherry-red spot. (c) C. Patients with retinal detachment frequently complain of flashes of light or floaters that occur during traction on the retina as it detaches. This is followed by loss of vision. In small detachments, the retina may appear out of focus, but with larger detachments, a retinal fold may be identified. (u) D. Hyphema is usually associated with trauma, and is a collection of blood in the anterior chamber.

History & Physical/ENT/Ophthalmology Which of the following findings is most consistent with cataracts? A. conjunctival injection B. poorly visualized optic disc C. central visual field loss D. arcus senilis

Explanations (u) A. Conjunctival injection is associated with conjunctivitis and other inflammatory conditions of the eye. (c) B. Cataracts are caused by opacification of the crystalline lens, and this decreases the amount of light that enters the eye. It is difficult to see through the lens from either direction, and thus, the optic disc is poorly visualized on examination. (u) C. Central field loss is associated with macular degeneration. (u) D. While arcus senilis may be seen in geriatric patients and is not associated with cataracts.

History & Physical/ENT/Ophthalmology Which of the following is described as a harmless triangular nodule in the bulbar conjunctiva on either side of the iris? A. Corneal arcus B. Hordeolum C. Pinguecula D. Xanthelasma

Explanations (u) A. Corneal arcus is an extremely common, bilateral, benign peripheral corneal degeneration, associated hyperlipidemia. (u) B. Hordeolum is characterized by a localized red, swollen, acutely tender area on the upper or lower lid. (c) C. Pinguecula is a yellow elevated conjunctival nodule, more commonly on the nasal side, in the area of the palpebral fissure. (u) D. Xanthelasma appear as yellow plaques that occur on the anterior surface of the eyelid, usually bilaterally near the inner angle of the eye.

Scientific Concepts/ENT/Ophthalmology Closure of the eyelids is mediated by which cranial nerve? A. cranial nerve III B. cranial nerve V C. cranial nerve VII D. cranial nerve IX

Explanations (u) A. Cranial nerve III is involved in extraocular movement and controls opening of the eyelids but has no control over closing the eyelids. (u) B. Cranial nerve V controls the motor function of the temporal and masseter muscles and facial sensation. It has no control over eyelid closure. (c) C. Cranial nerve VII controls the motor function of the facial muscles not controlled by cranial nerve V (forehead, eyebrows, mouth, and lips) including closing of the eyelids. (u) D. Cranial nerve IX controls swallowing and the voice.

Clinical Intervention/ENT/Ophthalmology An 18 year-old college student took her goggles off in the chemistry lab while she was washing her glassware from an experiment she had just completed. She thinks the beaker had contained sodium hydroxide, and it splashed into her right eye. She rinsed her eye out for about five minutes in the lab. Which of the following is the most appropriate first step? Answers A. Check visual acuity B. Flush the eye with two liters saline C. Check for corneal damage with fluorescein D. Instill a topical anesthetic to facilitate examination

Explanations (u) A. Delaying irrigation of the eye to assess vision or corneal damage is inappropriate as damage to the eye may occur during the delay. (c) B. Any chemical injury to the eye may result in severe injury and loss of sight, and is a true emergency. The eye should be irrigated copiously with at least two liters of saline, and pH checked to determine when the chemical has been cleared. (u) C. See A for explanation. (u) D. Topical anesthetic may be used to facilitate irrigation with a Morgan lens, but see A for explanation.

Clinical Therapeutics/ENT/Ophthalmology A 34 year-old female presents with recurrent bouts of dizziness, tinnitus, and hearing loss. She states that the episodes are incapacitating and cause her to become nauseous and vomit. The attacks last about one hour and the symptoms disappear after a few days. The last two episodes were treated with meclizine (Antivert) and prochlorperazine (Compazine) at the emergency room. Audiologic testing reveals low-tone frequency hearing loss. Which of the following is the most appropriate long-term management for this patient? A. Epley maneuver B. Diuretics and low-sodium diet C. Broad-spectrum antibiotics and Ibuprofen D. Scopolamine transdermal patch

Explanations (u) A. Epley maneuver is used to treat benign paroxysmal positional vertigo. (c) B. Diuretics and a low sodium diet will decreases the endolymphatic pressure in the semicircular canals, which is believed to be elevated in Meniere's disease, and help relieve symptoms. (u) C. Broad-spectrum antibiotics and ibuprofen are used to treat otitis media, not Meniere's disease. (u) D. A scopolamine patch is useful for treatment of a single episode, but not long-term management.

Scientific Concepts/ENT/Ophthalmology Small grayish vesicles and punched-out ulcers in the posterior pharynx in a child with pharyngitis is representative of which organism? A. Epstein-Barr virus B. Group C Streptococcus C. Coxsackievirus D. Gonorrhea

Explanations (u) A. Epstein-Barr virus presents with enlarged tonsils with exudates and petechiae of the palate. (u) B. Group C Streptococcus presents with a red pharynx and enlarged tonsils with a yellow, blood tinged exudates (c) C. Coxsackievirus presents with small grayish vesicles and punched-out ulcers in the posterior pharynx. (u) D. Neisseria gonorrhea of the pharynx may be asymptomatic

History & Physical/ENT/Ophthalmology Which of the following would indicate an optic nerve lesion? A. Excessive conjunctival edema B. Ptosis C. Inability to gaze laterally D. Afferent pupillary defect

Explanations (u) A. Excessive edema of the conjunctiva is a feature of chemosis. (u) B. Ptosis is not indicative of an optic nerve lesion. (u) C. Inability to gaze laterally would be due to paralysis of the lateral rectus muscle controlled by cranial nerve VI. (c) D. Pupil size, controlled centrally by the Edinger-Westphal nucleus in the midbrain, is primarily based on the afferent light stimulus transmitted via the optic nerve.

Diagnostic Studies/ENT/Ophthalmology During a baseball game, a 22 year-old college student is hit in the right eye by a baseball. He complains of blurry vision in that eye. On physical exam, the physician assistant notes proptosis of the right eye, and limitation of movement in all directions. On CT scan, which of the following is most likely to be seen? A. Fracture of the medial orbital wall B. Prolapse of orbital soft tissue C. Hematoma of the orbit D. Orbital emphysema

Explanations (u) A. Fracture of the medial orbital wall is associated with diplopia from medial rectus impingement, orbital emphysema and epistaxis. (u) B. Prolapse of orbital soft tissue, including inferior rectus muscle, inferior oblique muscle, orbital fat, and connective tissue results in enophthalmos, ptosis, diplopia, anesthesia of the ipsilateral cheek and upper lip, and limitation of upward gaze and is seen with fractures of the orbital floor. (c) C. Orbital hemorrhage into the space surrounding the globe following blunt trauma and rupture of the orbital vessels results in increased ocular pressure, proptosis, visual loss, and limitation of movement in all directions. CT reveals a hematoma. (u) D. Orbital emphysema is seen with fractures of the medial orbital wall or floor of the orbit into the maxillary and ethmoid sinuses respectively. It will not lead to proptosis.

Clinical Intervention/ENT/Ophthalmology A 66 year-old male presents with epistaxis of two hours duration. He has been unable to stop the bleeding at home. He has a history of hypertension and cardiovascular disease. On exam the vital signs are T-38°C, P-74/minute, RR-20/minute, and BP-180/110 mmHg. Bleeding is noted from the right nare; the left is essentially clear with normal nasal mucosa. After applying pressure for 15 minutes, a bilateral anterior pack is placed. You note persistent bleeding, with blood in the posterior oropharynx. Of the following, the most appropriate treatment is to remove the packing and Answers A. replace with Gelfoam packing. B. cauterize Kiesselbach's plexus. C. insert a posterior balloon pack. D. apply topical solution of lidocaine and epinephrine

Explanations (u) A. Gelfoam packing, cauterization of Kiesselbach's plexus, and topical lidocaine and epinephrine are helpful in treating an anterior bleed, but not in a posterior bleed. (u) B. See A for explanation. (c) C. A posterior balloon packing is used to treat posterior epistaxis. These patients must be admitted to the hospital and prompt consultation with an otolaryngologist is indicated. (u) D. See A for explanation.

Clinical Therapeutics/ENT/Ophthalmology An 18 year-old patient is diagnosed with bacterial conjunctivitis. Gram stain reveals gram-negative intracellular diplococci. Which of the following is the most appropriate treatment of this infection? A. Gentamicin B. Penicillin C. Bactrim D. Ceftriaxone

Explanations (u) A. Gentamicin is used in the treatment of gram negative rods and does not provide coverage of Neisseria gonorrhoeae. (u) B. Penicillin does provide coverage for some Neisseria species but is not considered the treatment of choice for Neisseria gonorrhoeae because of increased resistance. (u) C. Bactrim is used to treat Chlamydia trachoma not Neisseria gonorrhoeae. (c) D. Neisseria gonorrhoeae is confirmed by the Gram stain findings of intracellular gram-negative diplococci. The treatment of choice for Neisseria gonorrhoeae is ceftriaxone.

Health Maintenance/ENT/Ophthalmology A patient with type 2 diabetes mellitus presents for a yearly eye exam. Ophthalmoscopic exam reveals neovascularization. Which of the following is the most likely complication related to this finding? A. Glaucoma B. Cataracts C. Vitreous hemorrhage D. Optic neuritis

Explanations (u) A. Glaucoma occurs in about 6% of diabetics. Neovascularization of the iris can cause closed angle glaucoma. (u) B. Cataracts can occur secondary to diabetes, but are not caused by proliferative retinopathy. (c) C. Proliferative retinopathy, as evidenced by neovascularization, is associated with an increased risk of vitreous hemorrhage. (u) D. Optic neuritis is strongly associated with demyelinating disease, like multiple sclerosis.

Clinical Therapeutics/ENT/Ophthalmology A 35 year-old patient has recurrent seasonal rhinitis and a history of mild asthma. Which of the following should be included for first-line management? A. Immunotherapy B. Decongestants C. Corticosteroid inhalers D. Cromolyn sodium (Intal)

Explanations (u) A. Immunotherapy (desensitization) is indicated as a last resort in patients who fail to either respond to pharmaceutical management or face prolonged exposure to known allergens. (u) B. Decongestants have a limited role in helping to decrease edema, and are generally ineffective in relieving allergic symptoms. (c) C. Regular use of corticosteroid nasal spray and oral inhalers prior to the allergy season is among the best means of preventing allergies. (u) D. Cromolyn sodium has been found to be moderately effective for some patients with allergic symptoms, but it is not usually first-line management.

Scientific Concepts/ENT/Ophthalmology A 53 year-old woman complains that she has not been able to smell for several weeks. Prior to an upper respiratory infection 3 weeks ago, her sense of smell was "just fine." The most likely cause of the anosmia is Answers A. intranasal obstruction. B. destruction of the olfactory neuroepithelium. C. thickened mucus covering the olfactory cilia. D. depletion of the G-protein in the ciliary membrane.

Explanations (u) A. Intranasal obstruction is common with acute bacterial or viral infections, but anosmia resolves when the obstruction resolves. (c) B. The olfactory epithelium is destroyed by viral infections and chronic rhinitis. (u) C. Thickened mucus does not inhibit odorants from reaching the neuroepithelium. (u) D. G-protein is not depleted in viral URIs.

Diagnosis/ENT/Ophthalmology A 23 year-old graduate student presents with sudden onset of severe dizziness, with nausea and vomiting for the past couple of hours. She denies hearing loss or tinnitus. She has had a recent cold. Which of the following is the most likely diagnosis? A. Meniere's disease B. vestibular neuronitis C. benign positional vertigo D. vertebrobasilar insufficiency

Explanations (u) A. Meniere's disease is associated with hearing loss, tinnitus, and vertigo that lasts from seconds to hours. (c) B. Vestibular neuronitis or labyrinthitis presents with vertigo, nausea, and vomiting, but not hearing loss or tinnitus. It is related to viral URIs, and develops over several hours, with symptoms worse in the first day, with gradual recovery over several days. (u) C. Benign positional vertigo occurs with changes in position, especially rapid movements of the head. Nausea may occur, but vomiting is not significant. (u) D. Vertebrobasilar insufficiency is usually accompanied by brain stem findings, such as diplopia, dysarthria, or dysphagia, and is not common in this age group.

Diagnosis/ENT/Ophthalmology A 64 year-old woman complains of headache and left eye pain for about a day. She says it started yesterday as a dull ache and now is throbbing. She also complains of nausea and vomiting, which she attributes to the popcorn she ate at the movie theater yesterday afternoon. On exam, the left pupil is mid-dilated and nonreactive. The cornea is hazy. A ciliary flush is noted. Which of the following is the most likely diagnosis? Answers A. Migraine headache B. Temporal arteritis C. Acute glaucoma D. Retinal artery occlusion

Explanations (u) A. Migraine headache does not present with eye findings. (u) B. Temporal arteritis presents with headache and systemic symptoms of fever, myalgias, anorexia, and tenderness over the temporal artery. (c) C. Acute glaucoma often presents with abdominal complaints that may delay diagnosis. Findings of ciliary flush, mid-dilated and nonreactive pupil, and hazy cornea in a patient with severe eye pain are consistent with acute angle closure glaucoma. (u) D. Retinal artery occlusion presents with sudden, painless, severe loss of vision. There are no systemic symptoms.

Diagnosis/ENT/Ophthalmology A 23 year-old graduate student presents with sudden onset of severe dizziness, with nausea and vomiting for the past couple of hours. She denies hearing loss or tinnitus. She has had a recent cold. Which of the following is the most likely diagnosis? Answers A. Ménière's disease B. Vestibular neuronitis C. Benign positional vertigo D. Vertebrobasilar insufficiency

Explanations (u) A. Ménière's disease is associated with hearing loss, tinnitus, and vertigo that lasts from seconds to hours. (c) B. Vestibular neuronitis or labyrinthitis presents with vertigo, nausea, and vomiting, but not hearing loss or tinnitus. It is related to viral URIs, and develops over several hours, with symptoms worse in the first day, with gradual recovery over several days. (u) C. Benign positional vertigo occurs with changes in position, especially rapid movements of the head. Nausea may occur, but vomiting is not significant. (u) D. Vertebrobasilar insufficiency is usually accompanied by brain stem findings, such as diplopia, dysarthria, or dysphagia, and is not common in this age group.

Clinical Intervention/ENT/Ophthalmology A 57 year-old male was working on his farm, when some manure was slung hitting his left eye. He presents several 61 days after with a red, tearing, painful eye. Fluorescein stain reveals uptake over the cornea looking like a shallow crater. Which of the following interventions would be harmful? A. Ophthalmic antibiotics B. Pressure patch C. Examination for visual acuity D. Copious irrigation

Explanations (u) A. Ophthalmic antibiotics and copious irrigation are indicated when treating a patient with a suspected corneal ulcer due to an infectious cause. (c) B. Patching of the eye after abrasion associated with organic material contamination is contraindicated due to increased risk of fungal infection. (u) C. Examination for assessment of visual acuity should be performed. (u) D. See A for explanation.

Diagnostic Studies/ENT/Ophthalmology In a patient with amaurosis fugax what is the most appropriate initial diagnostic study? A. Ophthalmoscopy B. Schiotz tonometry C. MR angiography D. Carotid ultrasound

Explanations (u) A. Ophthalmoscopy provides visualization of retina but does not help elucidate the source of the plaque. (u) B. Schiotz tonometry is used to measure intraocular pressure. (u) C. MR angiography is a useful test to identify retinal vascular anatomy but is not the first choice in the search for causes of amaurosis fugax. (c) D. The most common cause of amaurosis fugax is an atherosclerotic plaque in the carotid artery which can be identified with ultrasound.

Clinical Intervention/ENT/Ophthalmology A 20 year-old presents 30 minutes after being struck by a hockey puck in the mouth. On physical examination a central incisor is missing from its socket. The patient has the tooth wrapped in tissue paper and the root appears intact. Which of the following is the most appropriate next step in the treatment of this patient? A. Administration of IM penicillin B. No treatment is warranted C. Place tooth in saline and refer to plastic surgery for reimplantation D. Immediately reimplant the tooth and refer to an oral surgeon

Explanations (u) A. Penicillin is not indicated for treatment of an avulsed tooth. (h) B. Reimplantation is warranted as this is a permanent tooth with root intact. Primary teeth are never reimplanted. (u) C. See D for explanation. (c) D. Avulsed permanent teeth should be cleansed, transported in Hanks solution or saline and reimplanted by an oral surgeon within one hour.

Diagnosis/ENT/Ophthalmology A 2 year-old child is brought to the office because of a cough and a fever of 102 degrees F for 2 days. The physician assistant notes the presence of hoarseness, a barking cough, and stridor. The ears and nose exam are unremarkable. Auscultation of the chest reveals decreased breath sounds without crackles or expiratory wheezes. Which of the following would be the initial diagnostic impression? A. pneumonia B. bronchiolitis C. croup D. asthma

Explanations (u) A. Pneumonia would not produce the stridor noted on physical exam. Pneumonia would more than likely produce crackles, also not noted in this patient. (u) B. Bronchiolitis would produce inspiratory wheezes which are absent in this patient. (c) C. Hoarseness, inspiratory stridor, and a barking cough are classic signs of croup, all of which are noted in this patient. (u) D. Asthma would not normally present with fever and stridor.

Diagnostic Studies/ENT/Ophthalmology A 2 month-old infant presents for a routine health maintenance visit. The mother has been concerned about the infant's hearing since birth. Physical examination reveals no apparent response to a sudden loud sound. Which of the following is the most appropriate diagnostic evaluation? A. audiometry B. tympanometry C. acoustic reflectometry D. auditory-evoked potentials

Explanations (u) A. Pure tone audiometry can be used to screen for hearing deficits in children over the age of 3 years. (u) B. Tympanometry is used to identify an effusion as the cause of hearing loss, but in infants over the age of months. (u) C. Acoustic reflectometry measures the spectral gradient of the tympanic membrane, but is not used clinically due to concerns about its reliability. (c) D. Brainstem auditory-evoked potentials evaluate the sensory pathway and identify the site of any anatomical disruption. The test does not require any active response from the patient and is useful in the evaluation of suspected hearing loss in an infant.

Clinical Intervention/ENT/Ophthalmology Which of the following is a potential complication of a traumatic hyphema? A. retinal detachment B. glaucoma C. cataract formation D. chronic conjunctivitis

Explanations (u) A. Retinal detachment may occur from trauma but not from a hyphema. (c) B. If the trabecular network becomes obstructed from the hyphema then glaucoma may occur. (u) C. Cataracts may be caused by increasing age (most commonly), toxins, systemic disease, smoking, and hereditary, but not by hyphemas. (u) D. Chronic conjunctivitis is not known to be associated with hyphemas.

Health Maintenance/ENT/Ophthalmology Which of the following is the most common cause of adult blindness in the USA? Answers A. Hypertension B. Diabetes mellitus C. Macular degeneration D. Retinal artery occlusion

Explanations (u) A. See B for explanation. (c) B. Diabetes is the leading cause of blindness in the US, and is usually due to diabetic retinopathy. Up to 15% of type 1 diabetics and up to 7% of type 2 diabetics become legally blind. (u) C. See B for explanation. (u) D. See B for explanation.

Health Maintenance/ENT/Ophthalmology Which of the following is the most common cause of blindness in adults under the age of 70 years in the USA? A. hypertension B. diabetes mellitus C. macular degeneration D. retinal artery occlusion

Explanations (u) A. See B for explanation. (c) B. Diabetic retinopathy is the leading cause of blindness in adults under the age of 70 in the US. (u) C. See B for explanation. (u) D. See B for explanation.

Diagnostic Studies/ENT/Ophthalmology A 45 year-old male complains of loss of hearing in his left ear. He also complains of ringing in the ear, and has had occasional dizziness. On exam, there is unilateral left- sided sensorineural hearing loss and a diminished corneal reflex. Neuro exam is otherwise normal. TMs are normal, and canals are clear. Neck is supple, without adenopathy. Oropharynx is normal. Of the following, the best diagnostic study to identify the cause of this patient's complaints is Answers A. auditory brainstem evoked response. B. gadolinium-enhanced MRI. C. acoustic reflex testing. D. vestibular testing.

Explanations (u) A. See B for explanation. (c) B. MRI has replaced auditory brainstem evoked response and acoustic reflex testing in the evaluation of patients for acoustic neuromas. (u) C. See B for explanation. (u) D. Vestibular testing is not a useful screening test for acoustic neuromas.

Clinical Intervention/ENT/Ophthalmology A 28 year-old HIV positive male presents complaining of fever, dysphagia, odynophagia, and trismus for the past 2 days. Physical examination reveals an ill-appearing male with a temperature of 101.3 degrees F, poor dentition, "woody" edema in the sublingual area and neck, tongue displaced posteriorly, and drooling. The immediate managment of this patient includes which of the following? A. CT of the neck B. intubation C. start IV penicillin D. incision and drainage of the abscess

Explanations (u) A. See B for explanation. (c) B. This person presents with the classic signs and symptoms of Ludwig's angina. He is unable to handle his secretions and the displacement of his tongue suggests impending airway obstruction. Intubation to secure his airway is the most immediate concern. CT of the neck for diagnosis, starting IV antibiotics, and incision and drainage are actions that need to be taken after securing the airway. (u) C. See B for explanation. (u) D. See B for explanation.

Clinical Intervention/ENT/Ophthalmology A 17 year-old male is accidentally struck in the right eye while playing football and is immediately transported to the hospital. In the emergency room, he complains of severe pain behind the eye as well as double vision. On examination, he has exophthalmos, cannot move his right eye upward and blood is noted in the anterior chamber. Which of the following is the most appropriate course of action at this time? A. Apply ice packs and cold compresses B. Immediately refer the patient to an ophthalmologist C. Attempt to keep the patient calm and order a skull x-ray D. Administer a dose of intramuscular broad-spectrum antibiotic

Explanations (u) A. See B for explanation. (c) B. This scenario describes a "blow-out" fracture of the orbit with hyphema and, because of the signs and symptoms presented, warrants an immediate consult by an ophthalmologist. (u) C. See B for explanation. (u) D. See B for explanation.

History & Physical/ENT/Ophthalmology In infants, the eyes should move in parallel without deviation by the age of A. 2 weeks. B. 3 months. C. 6 months. D. 1 year.

Explanations (u) A. See C for explanation. (u) B. See C for explanation. (c) C. Intermittent alternating convergent strabismus is frequently noted for the first 6 months of life, but referral is indicated if it persists beyond 6 months. (u) D. See C for explanation.

History & Physical/ENT/Ophthalmology Hairy leukoplakia has the greatest prevalence of distribution on the 51 A. palate. B. floor of the mouth. C. lateral tongue. D. gingiva.

Explanations (u) A. See C for explanation. (u) B. See C for explanation. (c) C. The lateral border of the tongue is where hairy leukoplakia is commonly seen. (u) D. See C for explanation.

Scientific Concepts/ENT/Ophthalmology What is the most common location of anterior nasal epistaxis? A. Middle turbinate B. Posterior ethmoid artery C. Kiesselbach's triangle D. Inferior turbinate

Explanations (u) A. See D for explanation. (u) B. See D for explanation. (c) C. 90% of all cases of anterior epistaxis originate from Kiesselbach's triangle. (u) D. The inferior turbinate is the most common location for posterior epistaxis.

Diagnosis/ENT/Ophthalmology An 18 year-old sexually active female was seen in the student health clinic 1 week ago for a sore throat. A streptococcal antigen test was positive, and she was given a prescription for oral penicillin. After 3 days, she stopped her medication because she felt better. She now presents with a severe sore throat. On physical examination, she has a temperature of 102.6° F (39.2° C), marked pharyngeal erythema, medial deviation of the soft palate on the left, tender left anterior cervical adenopathy, and a "hot potato" voice. The rest of her history and physical examination are unremarkable. Which of the following is the most likely diagnosis? A. Recurrent streptococcal pharyngitis B. Infectious mononucleosis C. Gonococcal pharyngitis D. Peritonsillar abscess

Explanations (u) A. This presentation suggests a complication of an incompletely treated streptococcal pharyngitis rather than recurrent disease. (u) B. Infectious mononucleosis may present with severe sore throat, fever, and cervical adenopathy in this age group, but would not cause deviation of the soft palate or the muffled voice. (u) C. Gonococcal pharyngitis usually follows a more indolent course than this patient's presentation. (c) D. The soft palate deviation and a muffled voice are classic signs of peritonsillar abscess.

Diagnosis/ENT/Ophthalmology A 4 year-old boy presents with purulent, foul-smelling nasal discharge for three days. He has not had any other symptoms of respiratory illness, cough, wheeze, or fever. His activity level and appetite has been normal. On exam, he is afebrile. TM's have normal light reflex, canals are clear. Left nare is clear; there is considerable amount of purulent exudate from the right nare, and a bright reflection of light is noticed. Oropharynx is without inflammation or exudate. Neck is supple, without lymphadenopathy. Which of the following is the most likely diagnosis? A. viral URI B. acute sinusitis C. allergic rhinitis D. nasal foreign body

Explanations (u) A. Viral URI does not present with foul-smelling nasal discharge. (u) B. Acute sinusitis may present with purulent nasal discharge, but the observation of a bright light reflection suggests a foreign body. (u) C. Allergic rhinitis is seasonal, associated with sneezing and other allergy-related symptoms. (c) D. Nasal foreign body is suggested by unilateral nasal obstruction or discharge.

Diagnosis/ENT/Ophthalmology A 4 year-old boy presents with purulent, foul-smelling nasal discharge for three days. He has not had any other symptoms of respiratory illness, cough, wheeze, or fever. His activity level and appetite has been normal. On exam, he is afebrile. TM's have normal light reflex, canals are clear. Left nare is clear; there is considerable amount of purulent exudate from the right nare, and a bright reflection of light is noticed. Oropharynx is without inflammation or exudate. Neck is supple, without lymphadenopathy. Lungs are clear, with equal breath sounds and no wheezing. Heart has regular rhythm without murmurs. Which of the following is the most likely diagnosis? Answers A. Viral URI B. Acute sinusitis C. Allergic rhinitis D. Nasal foreign body

Explanations (u) A. Viral URI does not present with foul-smelling nasal discharge. (u) B. Acute sinusitis may present with purulent nasal discharge, but the observation of a bright light reflection suggests a foreign body. (u) C. Allergic rhinitis is seasonal, associated with sneezing and other allergy-related symptoms. (c) D. Nasal foreign body is suggested by unilateral nasal obstruction or discharge .

Health Maintenance/ENT/Ophthalmology Which of the following is considered a risk factor for retinopathy of prematurity? A. maternal rubella infection B. maternal alcohol abuse C. perinatal oxygen therapy D. family history of retinal detachment

Explanations (u) A. While maternal rubella infection is a risk factor for ocular disease in the newborn, it is not a specific risk for retinopathy of prematurity. (u) B. Maternal alcohol abuse is associated with the development of fetal alcohol syndrome, which includes craniofacial abnormalities, but does not include increased risk for retinopathy of prematurity. (c) C. Risk factors for retinopathy of prematurity include perinatal oxygen therapy, low birth weight, prematurity, and sepsis. (u) D. If retinopathy of prematurity is not treated, retinal detachment may occur causing blindness. A family history of retinal detachment is not considered a risk factor for the development of retinopathy of prematurity.

14

Extra-ocular movements

What're the s/sx of angle-closure glaucoma?

Extreme pain, blurred vision, N/V, HA. elevated intraocular pressure w tonometry (>22mmHg)

Pinguecula

Eye condition caused by chronic actinic exposure, repeated trauma, and dry and windy conditions. May present with elevated, yellowish, fleshy, conjunctiva mass found on the sclera adjacent to the cornea. Painless inflammation may occur. Tx - none necessary, may be respected if cosmetically undesirable or chronically inflamed.

Bacterial Conjunctivitis

Eye condition characterized by acute onset of copious, prudent discharge from BOTH eyes. May present with mild decrease in visual acuity and mild discomfort. Eyes may be "glued" shut upon waking. Common pathogens are strep pneumo, Staph aureus, Haemophilus aegyptius, and Mirabella. Transmitted via direct contact or famines, autoinoculation from one eye to other is typical. Infection is usually self limited. Rare pathogens via sexual contact or vaginal delivery and may result in permanent visual impairment if not treated. Tx is ABX for specific pathogen topically, for rare pathogens TX topical plus systemic ABX.

Blowout Fracture

Eye condition commonly caused by trauma from fist or ball, effecting the floor of the orbit, trapping orbital structures. May present with swelling and misalignment of the eyes, movement restriction, specifically the inability to look up due to entrapment of the infraorbital nerve and musculature. Double vision is common and subcutaneous emphysema and exophthalmos are present. Prompt referral to ophthalmologist, keep pt from sneezing or coughing, use of nasal decongestants, ice packs and cold compresses as well as ABX during transport to ophthalmology.

Amblyopia

Eye condition of reduced visual acuity not correctable by refractive means. Most commonly caused by Strabismus, but may caused by Uremia, or toxins such as alcohol, tobacco, lead, or other toxic substances.

Presence of the Centor criteria are suggestive of group A strep pharyngitis. List the criteria.

Fever - Absence of cough - Tonsillar exudates - Tender cervical lymphadenopathy

A 5-year-old child with no known drug allergies is diagnosed in your clinic with bilateral acute otitis media. Which of the following is the drug of choice?

First choice antibiotic treatment for acute otitis media includes a 10-day course of amoxicillin (80 to 90 mg/kg/day in two divided doses) or a combination of erythromycin (50 mg/kg/day) and a sulfonamide (150 mg/kg/day). Reasons for amoxicillin therapy include spectrum of activity including both susceptible and intermediate resistant S pneumoniae, safety, cost, and tolerability.

If an abrasion is suspected, what diagnostic test will aid?

Fluorescein stain.

Corneal Abrasion

Fluorescein staining

Which topical antibiotics are not ototoxic and may be used with a tympanic membrane perforation?

Fluoroquinolones

dysfunction of eustachian tube

Follows URI -sx- ear fullness, popping -dx- fluid behind TM -tx- decongestants pseudophedrine, phenylepherine, oxymetazoline nasal spray

A patient presents with a cerumen impaction. Which of the following is true when performing the Rinne- Weber test?

For conductive hearing loss, the Weber will lateralize to the affected ear and bone conduction will be greater than air conduction. Conductive hear loss prevents sound from entering the inner ear due to obstruction in the external auditor canal and middle ear. Examples would be cerumen impaction and otitis media. Sensory neural hearing loss affects the inner ear and cranial nerve VIII. Weber will lateralize to the unaffected ear (normal ear) and Rinne will reveal Air conduction > Bone conduction.

What PE is essential for suspected foreign body?

Full inspection of lids, conjunctiva and cornea. Slit lamp

16 - TURN OFF LIGHTS

Fundoscopic exam - have them focus on a spot on the wall in front of them - their Right eye, my right hand, my right eye - their Left eye, my left hand, my left eye

Fever, tender anterior cervical adenopathy, pharyngotonsillar exudate

GABHS Criteria

Tender temporal artery Fever, malaise Elevated ESR EMERGENCY

Giant Cell Arteritis clinical features

Causes of monocular vision loss

Giant cell arteritis Retinal artery occlusion Multiple sclerosis

In a patient whom you suspect has giant cell arteritis, which of the following medication and dosage combinations is indicated?

Giant cell arteritis (GCA) or temporal arteritis is a large vessel vasculitis that can cause ischemic optic neuropathy and blindness. The first line treatment of high dose prednisone 60 mg/day for one month is sight saving. The lower dose of prednisone 20mg per day is effective in treating polymyalgia rheumatic (PMR). Methylprednisolone dose-pack is a very low dose of steroid and would be ineffective in treating GCA. Infliximab is a tumor necrosis factor inhibitor used for treating ankylosing spondylitis, rheumatoid arthritis and crohn's disease.

What diagnostic tool may be used to aid in diagnosing acute angle closure glaucoma

Gonioscopy - measures iridocorneal angle

dx of closed angle glaucoma

Gonioscopy is the gold standard

Diptheria

Gradual onset of sore throat and fever with a grey, white patchy psuedomembrane development in the throat which may obstruct the airway or cause a "barking" cough similar to that in croup. May include markedly swollen cervical lymph nodes that may impede swallowing or breathing, "BULLS NECK". May involve cutaneous eruptions. Caused by Corynebacterium. Fatal in 5-10% of cases, up to 20% in young children and those over 40 years old. Antibiotic treatment is not shown to effect course of disease, but is indicated to reduce transmission to others. Metronidazole, Penicillin G, or Erythromycin

A 10-year-old male who plays soccer presentswith annularly configured dermal papules that are skin colored andshiny, and are located on his shins. His mother states that the lesionsstarted as nodules, and have since enlarged. There is no scale presentand they are asymptomatic. No one else in his home has similar findings.What is the most likely diagnosis?

Granuloma annulare (GA) is a self-limited condition that mayappear as solitary lesions or in a more generalized distribution.They begin as small, shiny skin colored dermal papules that enlargeover time with central clearing. There are no epidermal manifestations.GA commonly occurs over bony surfaces, such as the shins or dorsaof the hands. It is frequently misdiagnosed as tinea corporis; however,the absence of epidermal scaling helps rule out this diagnosis.

griseofulvin

Griseofulvin is used to treat skin infections such as jock itch, athlete's foot, and ringworm; and fungal infections of the scalp, fingernails, and toenails.

apthous ulcer

HPV6 -sx-several ulcers round, yellow greyish centers and red halos -tx- topical corticosteroids then predisone taper 1 week, prevention= cimetidine -mouth wash- chlohirixidne

herpetic gingivostomatitis

HSV-1 lesions on gingiva & mucocutaneous border of lip; seen in children 6mo-5 years causes burning followed by small vesicles that rupture & form scabs; treated with acyclovir & magic mouthwash

herpetic pharyngotonsillitis

HSV-1 vesicles--> ulcerative lesions on the posterior pharyngeal mucosa; seen in adults

Most common organism causing epiglottitis in children?

Haemophilus influenzae

Closed angle

Halo, headache, vomiting, fixed pupil

How long can a patient with an external hematoma of the ear wait to see the ENT?

He should be seen ASAP, goal < 7 days

Head and Face Inspection

Head is norm-o-cephalic, there are no lesions or areas of erythema, ecchymosis or swelling

tinnitus

Hearing ones own heartbeat is characteristic of the pulsatile version of this ear disorder. Perception of abnormal ear or head noises. May indicate sensory hearing loss. Intermittent, mild, high pitched episodes common in normal-hearing individuals. May present with sensory hearing loss with Weber and Rinne. Avoid exposure to excessive noise or ototoxic agents (aminoglycosides). Oral antidepressants may help (nortriptyline).

First step in removing foreign body?

Heavy irrigation

Common cause of Cauliflower ear (recurrent damage to cartilage)?

Hematoma of the external ear

A 10 year old male presents with bright red, well-demarcated petechiae and palpable purpura located on bilateral lower extremities. He also complains of abdominal pain and mild joint pain. His mother report the child had an upper respiratory infection about a week ago. Punch biopsy shows IgA immunoreactivity around post-capillary venules. What is the most likely diagnosis?

Henoch Schonlein purpura (HSP) is commonly described as palpable purpura. It is precipitated by an upper respiratory infection and can also be associated with abdominal pain and joint pain.

A 45-year-old female presents with a rashthat started on her ankles two days ago and is spreading up herleg. The lesions are asymptomatic; however, she has abdominal painand joint pain. The lesions are palpable and non-blanchable whencompressed. What is the most likely diagnosis?

Henoch-Schonlein Purpura (HSP) is a hypersensitivity vasculitisthat occurs most frequently after an infection with group A streptococcus.This rash consists of the classic palpable purpura, and can be accompaniedby abdominal pain that is worse after meals secondary, bowel ischemia,bloody diarrhea, and arthritis.

A patient presents complaining of a painful rash on his lips. What is the causative pathogen for the rash shown in the image?

Herpes simplex type 1 causes an orolabial and gingival vesicular rash. Herpes simplex type 2 causes genital lesions and is sexually transmitted. Herpes zoster or shingles causes a dermatomal, unilateral, and painful vesicular rash. Staphylococcus aureus and beta hemolytic streptococci cause bacterial skin infections.

An 18-year-old female presents with a history of draining abscesses in the axilla and groin, with large, open comedones. Currently, she has multiple scars in the axillae. What is the most likely diagnosis?

Hidradenitis suppurtiva affects females more than males, and may show a family history of nodulocystic acne and/or hidradenitis suppurtiva. Skin lesions are usually tender nodules and abscesses that may spontaneously drain. Open comedones, including double comedones, are common. Eventually, sinus tracts may form. Nodulocystic acne consists of nodules and cysts, ranging in size from 1 to 4 cm in diameter. These lesions are distributed on the face, back, and chest. Acanthosis nigricans is described as a velvety, hyperpigmented plaque distributed around the neck, in the axillae, and in the groin. MRSA can have several different presentations, ranging from erythrasma to the presence of papules and pustules. Comedones are not associated with MRSA.

Viral Conjunctivitis

Highly contagious eye condition, transmitted via direct contact with contralateral eye or with other persons. Can be transmitted in swimming pools and is most common in midsummer to early fall. Characterized by acute onset of unilateral or bilateral erythema of the conjunctiva, copious water discharge, and ipsilateral tender preauricular lymphadenopathy. TX is eye lavage with normal saline BID for 7-14 days. Vasoconstrictor-antihistamines may help. Warm compress to reduce discomfort.

A 66-year-old male patient complains of pain and swelling in his left foot intermittently over the past year. He denies any current symptoms. On examination you note the following findings. His left foot is unremarkable. Laboratory finding include an elevated uric acid. What is the most likely diagnosis?

His uric acid level is elevated, which further supports a diagnosis of gout. It is suspected that the intermittent left-foot swelling is related to acute flares of gout, which usually affects the first MCP joint. Choices (A), (C), (D), and (E) are not associated with elevated uric acid level or tophi. The patient may have osteoarthritis, but this does not explain the high uric acid level and the tophi on exam.

What is the name of the sign of a Shingles lesion on the tip of the nose?

Hutchinson Sign If you see this, make sure you do a through exam of the eye!!

Risk factors for thyroid cancer?

Hx of radiation exposure - HX of goiter - female gender - Asian

Hemorrhage into the anterior chamber of the eye is AKA. . .

Hyphema

tx of Peritonsillar abscess

I&D AB + steroids needle aspiration = gold standard, dx and tx tonsillectomy (10%)--if airway obstructed or marked asx recurrent strep

salter harris fracture

I- epiphysis. S= Seperation II- epiphsis + metaphyseal plate. A=Across. fx through metaphysis. MC!!! III- fracture through epipysis into articular surface.Lower= epipysis. IV- fracture through distal metaphysis, epiphysis, epipysiseal plate.T= THrough V- impaction of epiphyseal plate. ER= Everythings ruined

What is the tx for Strep pharyngitis?

IM pencillin in patient compliance is in doubt. Othewise an oral penicillin or cefoxime can be used. macrolides in the case of penicillin allergy

Retinal vein occlusion

INSIDIOUS ONSET 2 types: branch retinal vein occlusion (BRVO) central retinal vein occlusion (CRVO)

tx of mastoiditis

IV ABs (ceftriaxone) -if doesnt respond: myringotomy, a small incision in the tympanic membrane (eardrum), or the insertion of a tympanostomy tube into the eardrum -if still doesn't respond: mastoidectomy

What's the tx for G/C Bacterial Conjunctivitis?

IV Rocefin (ceftriaxoe) Topical erythromycin

You have hospitalized your patient who had presented with a peritonsillar abscess. What antibiotics would be appropriate choices for empiric therapy?

IV ampicillin/sulbactam or clindamycin

ciprofloxacin

IV antibiotic given to treat malignant otitis externa

azithromycin

IV antibiotic treatment for neonatal chlamydia conjunctivitis

ceftriaxone

IV antibiotic treatment for neonatal gonorrhea conjunctivitis

cefazolin

IV antibiotic used to treat mastoiditis

mastoiditis is a complication OM and can present with spiking fever and post auricular pain, erythema and fluctuant painful mass. What is the treatment?

IV antibiotics and myringotomy, followed by full course of oral antibiotic if ineffective, mastoidectomy

what is the treatment for mastoiditis?

IV antibiotics, possible mastoidectomy

treatment for closed angle glaucoma

IV carbonic anhydrase inhibitor, topical beta blocker, and osmotic diuresis. this is an emergency

GC conjunctivitis

IV ceftriaxone topical erythromycin

dexamethasone

IV corticosteroid used to treat epiglottitis

methylprednisone

IV corticosteroid used to treat optic neuritis caused by MS

Tx Orbital Cellulitis?

IV- Vancomycin + Ceftriaxone, Cefotaxime, Ampicillin-Sulbactam or Piperacillin-tazobactam So, Vanco + Anti-pseudomonas beta-lactam.

What is one of the main concerns when dealing with a hyphema?

If trauma was so intense as to cause a hyphema, be concerned about trauma elsewhere (Cervical, skull fracture, blow out fracture, etc)

A 33 year old male presents with chronic sinusitis. Cultures find fungal pathology. What underlying disorders should be considered?

Immunocompromise - Uncontrolled diabetes - Neutropenia - Glucocorticosteroid use - Nosocomial infection

If retinopathy is due to Toxoplasmosis of CMV retinitis, what must you also consider?

Immunocompromised state, HIV

What is the appropriate first line treatment of lyme disease in a non-pregnant adult female with erythema migrans and no other symptoms of lyme disease and no known drug allergies?

In patients over the age of 9 exhibiting skin or joint manifestations of lyme disease, the first line treatment is Doxycycline 100 mg bid. Patients less than 9 or those who are allergic to Doxycycline should be treated with amoxicillin. Erythromycin is fourth line treatment for all age groups. Ceftriaxone is first line for patients with nervous system involvement. (Wolff & Johnson, pg 691)

A Blowout fracture may result in entrapment of which muscle? Damage to the ______ nerve would cause paresthesia where?

Inferior rectus muscle - child cannot look upward. Infraorbital verve cause paresthesias in gums, upper lip and cheek.

retinitis pigmentosa

Inherited autosomal dominant Present with nightblindness in childhood May note pigmentation on retina Progressive visual loss begins in 2nd decade (Tunnel vision) Blindness often by age 40-50.

FB in eye

Injury to the eye which requires eyelids to be carefully exerted, stained with fluorescein, and observed with a Wood's lamp. Removal attempt with moistened, cotton-tipped swab. Patching may be beneficial if large, but limited to 24 hours with next day reexamination. Rust ring on cornea may be seen with metal objects, remove with rotating burr, or refer pt to ophthalmologist.

Open Angle Glaucoma

Insidious progressive bilateral loss of peripheral vision (Tunnel Vision). Gradual progressive "CUPPING" and pallor of optic disc with loss of vision progressing from decreased peripheral fields to complete blindness. IOP elevation is due to reduced or obstructed drainage of aqueous fluid. If left untreated may result in complete blindness by at 60-65. Primary - Bilateral presentation , with increased prevalence in 1st degree relatives and blacks. Secondary - Results from uveitis, ocular trauma, and all forms of CORTICOSTEROID USE. Cup:Disc radio > 0.5, or 0.2 or more asymmetry between both eyes. Initially paracentral visual field constriction. Central vision remains intact until late. Reproducible abnormalities in at least two of the following: optic disc, visual field, IOP. Tx with laser trabeculoplasty is often primary tx, consider surgical trabeculectomy if trabeculoplasty fails. Prostaglandin analogs are first line. BB, topical and/or oralcarbonic anhydrase inhibitors.

11

Inspect - external eyelids (eyes closed)

10 - EYES

Inspect - external eyelids (eyes open)

23 - NECK

Inspect - neck

21 - New speculum on otoscope

Inspect - nose and nares

1 - HEAD

Inspect - scalp and head

22 - MOUTH

Inspect (with a light) - lips - buccal mucosa (ulcers) - posterior pharynx (uvula) - under the tongue*****

Inflammation and infection of the meibomian gland

Internal Hordeolum

Throbbing pain, small pupil, normal pressure and perilimbic injection(Redness around iris), think...?

Iritis

A 16-year-old female has tried topical clindamycin and tretinoin. In addition, she recently finished a six-month course of doxycycline 100 mg bid. She has not noticed much improvement in her acne. The patient continues to complain of large, painful lesions, as well as numerous comedonal lesions. What is the next appropriate step in treatment?

Isotretinoin is indicated for nodulocystic acne, as well as acne that is resistant to topical treatments and oral antibiotics. Clotrimazole is an antifungal medication that is indicated for the treatment of cutaneous candidiasis and tinea. Keflex is a first generation Cephalosporin. It is not a first line treatment for acne. Elidel is indicated as a second line treatment for atopic dermatitis in patients over two years of age. It does not treat acne.

What're the s/sx of Bacterial Conjunctivitis?

Itching Purulent d/c Mild loss of visual acuity erythema, tearing Pt may wake up w a "crust" over the eye in the AM.

Epistaxis

Keisselbach plexus

What anatomical location is the most common site of anterior epistaxis?

Kiesslebach's Plexus or Little's Area is the most common site of anterior nosebleeds

Location of most anterior nose bleeds?

Kiesslebach's plexus

What should be done to remove a live insect from an ear?

Kill insect first with mineral oil or viscous lidocaine before attempting removal

cataract

LEADING CAUSE OF BLINDNESS IN THE WORLD causes: trauma, sun, steroids, congenital in kids

Patient presents with acute SEVERE vertigo that last days to weeks and hearing loss. The vertigo progessively improves over a few weeks but the hearing loss may or may not resolve. Likely dx?

Labrynithitis

Vertigo and hearing loss after acute otitis media with a normal otoscopic exam is the most likely what diagnosis?

Labyrinthitis

Acute vertigo + hearing loss hearing loss for several days

Labyrinthitis clinical features

A 12-year-old female presents with a complaint of dry flaking skin that becomes fissured and painful. Her skin has always had dark plate-like scales. What is the most likely finding in the patient's history?

Lamellar ichthyosis is a condition in which a baby is born with a collodion membrane. Within a few weeks, this membrane is shed and replaced by large gray scales. These plate-like scales persist with no improvement over time. Painful fissures on the hands and feet are common.

Hoarseness of voice, cough Follows URI

Laryngitis clinical features

Tx for Open-angle Glaucoma: Name Prostaglandins used

Latanoprost Bimatoprost

prostaglandin analogs

Latanoprost (Xalatan) Bimatoprost (Lumigan) Travoprost - CHANGES IRIS COLOR PERMANENTLY

Macular Degeneration

Leading cause of permanent visual loss in developed countries. Eye disorder that may be age related or secondary to toxic effects of medications (chloroquine or phenothiazine). Related to increased age, white race, female gender, family hx, and smoking, but exact cause unknown. Drusen deposits (discrete yellow deposits) are found on Burch's membrane, leading to degenerative changes, loss of nutritional supply, atrophy and neovascularization. Insidious onset, with chief clinical feature of gradual loss of central vision without pain or ocular erythema. WAVY OR DISTORTED VISION can be measured with Ambler Grid. Mottling, serious leaks, hemorrhages commonly develop on retina. Tx - no effective Tx. Laser therapy (reduces drusen) or intravitreal injections of monoclonal antibody drugs may slow progression of age related variant, but will not stop it.

Lesion of the RIGHT optic tract will cause what visual symptoms?

Left homonymous hemianopsia

Oral Herpes Simplex

Lesions most commonly occur on the gingival and mucocutaneous junction of the lip. Condition is common, mild, and self-resolving in most adults. May be severe in the immunocompramised. May present initially with burning around the lips, followed by small vehicles that rupture and scab. Lesions are found on the gingival, mucocutaneous junction of the lip, tongue, buccal mucosa and soft palate. Tx with antiviral therapy may shorten course and reduce pain afterwards.

Menieres disease

Less commonly known as endolymphic hydrops. Distention of endolymphatic compartment of the inner ear. Precise cause is unknown, however, two known sources are SYPHILIS and head trauma. May present with episodic vertigo (spells last 20 minutes to several hours), with fluctuating low-frequency sensorineural hearing loss, tinnitus, and unilateral aural pressure. Tx with low sodium diet and diuretics, congenital anomalies, otosclerosis, neoplasms, vasculitis.

A mother presents with her 6-year-old childwho has a rash on his arm. The rash has remained unchanged despitethe use of topical steroids for one month. The lesion is asymptomatic.What is the most likely diagnosis?

Lichen striatus is a benign rash consisting of linearly configured,shiny, and flat lesions that occur on any skin surface. This rashoccurs suddenly and resolves on its own in several weeks. The etiologyis unknown.

Ectropion

Lid everts, often secondary to age, trauma, infection or palsy of the facial nerve. TX - surgical repair if condition causes trauma, excessive tearing, exposure keratitis, or cosmetic distress.

Entropion

Lid inverts, often secondary to scar tissue or spasm of the orbicularis oculi muscle. Also frequently in older individuals due to degeneration of lid fascia. TX - surgical repair if condition causes trauma, excessive tearing, exposure keratitis, or cosmetic distress.

What is the Dx? A patient with pharyngitis is given amoxicillin and develops a macular-papular rash.

Likely mononucleosis (not bacterial)

What is the Dx? Bacterial infection/cellulitis of the floor of the mouth (stems from root of the teeth) potentially life threatening.

Ludwig's angina

epistaxis

MC = anterior. kiesslebach's plexus posterior = woodruff's plexus. sphenopalatine artery.

herniated disc

MC herniated disk L4-L5, L5-S1 L4- weak ankle dorsiflexion, loss of knee jerk L5- lateral, weak big toe extension S1- weak plantar flexion, loss of ankle jerk -dx- straight leg raise, mri

supracondylar fractures

MC in children 5-10 yr -sx- swelling tenderness @ elbow, + anterior fat pad in children -tx- non displaced splint. displaced- ORIF

Pantellofemoral syndrome

MC in runners -sx- knee pain behind or around patella -dx- apprehension sign- examiners appliers pressure medial- lateral patella with pain -tx- PT, stregnthening

macular degeneration (dry vs wet)

MC of vision loss in elderly sx- central vision loss. rapid vision loss dry- yellow deposits -dx-dry- YELLOW DEPOSITS- DURSEN wet- flourescin angiogrpahy -tx- dry- amsler grid -wet- bevacizumab

Reactive arthritis****

MC- chlaymydia -sx- cant see, cant pee, cant climb a tree. conjunctivitis, urethritis, arthritis. Keratoderma blennorrhagica (hyperkeratotic lesions on palms/soles) -dx- HLA-27 (Alkalosing spondylitis, bamboo spine too), synovial fluid- neg bacterial culture***, WBC 1,000-8,000 -tx- NSAIDS

Sialadenitis

MC- staph aureus. MC viral- MUMPS -sx- pus can be massaged by opening of duct -dx- failure to improve-U/S or CT -tx- sialogues....naficillin 405 g Q IV and measures to salivary flow if worsening sialogues

retinal detachment

MC= rhegmatogenous sx- extreme mytopia (far away objects appear blurry), curtain over vision*** Flashers and floaters -dx- Schaffer sign- clumping of pigment afferent pupillary defect--- contrlateral pupil doesnt constrict when light shown in fundscopy- regous retina- flapping**** -tx- EMERGENCY! remain supine, detach from temporal... turn head towards attachment side. optha

retinopathy

MCC = Diabetic retinopathy other causes: HIV, CMV, toxoplasma

acute otitis media

MCC = STREP PNEUMO H. flu, M. cat, strep pyogenes, staph aureus RSV = viral Dx: INSUFFLATION

pharyngitis

MCC = group A Beta hemolytic strep = strep pyogenes Dx: throat culture, ASO TITERS consider C&G infection Tx: PCN

Retinopathy

MCC is Diabetes, with others being HTN, pre-eclampsia/eclampsia, blood dyscriasis, HIV Leading cause of blindness in the US. Proliferative - neovascularization, vitreous hemorrhage. Nonproliferative - venous dilation, retinal hemorrhage, retinal edema, hard exudates, microanuerysms. Treat underlying condition, control BP, glucose. Laser photocoagulation or vitrectomy may be used. Severe dz is permanent.

thyroglossal duct cyst

MIDLINE moves with protrusion of tongue

femoral condyle fractures

MOI- axial loading peroneal nerves injuries (check 1st web space)-popliteal injury -tx- immediate ortho consult

What is the best imaging modality for acoustic neuroma?

MRI

dx of mastoiditis

MRI or CT clinical based on sx

dx of labyrinthitis

MRI to r/o other cause LP if meningitis suspected audiogram

Monteggia/ Galeazzi fracture

MUGR monteggia- proximal ulnar shaft fx with anterior radial head dislocation , usually direct blow to forearm -sx- radial nerve injury- paralysis to -tx- ORIF galeazzi- mid distal radial head dislocation -tx- unstable! needs ORIF

Tx of Bacterial conjunctivitis?

Macrolides (Erythromycin) Sulfa (Sulfacetamide) Fluoro (Cipro, Oflaxacin) Polymixin (Cortisportin, Polytrim)

What is the disease condition that involves gradual central vision loss?

Macular Degeneration

Irreversible central vision loss

Macular Degeneration is the leading cause of...

This disorder of the eye may be age related or secondary to the toxic effects of drugs such as chloroquine or phenothiazine.* It is the leading cause of irreversible central visual loss.*

Macular degeneration

What is the concern with Dacryoadenitis?

May lead to Dacryocistitis and periorbital cellulitis

Which of the following conditions is a cause for central vertigo?

Meniere syndrome, labyrinthitis, vestibular neuronitis, and perilymphatic fistula are causes of peripheral vertigo. Acoustic neuroma, or eight cranial nerve schwannomas, are among the most common intracranial tumors, and a cause for central vertigo.

What is the most common cause of head and neck malignancy?

Metastasis to the lymph nodes of the neck

Tx of Optic Neuritis?

Methylprednisolone (Solu-medrol) IV, Neurology evaluation

A 6-year-old female presents with small erythematous papules grouped around the mouth. The mother reports that she tried to treat with over-the-counter hydrocortisone 1% cream. The condition has worsened. What is the best step in management of this condition?

Metronidazole 0.75% gel bid is a first line treatment for perioral dermatitis. Topical steroids, such as hydrocortisone valerate cream, will actually worsen perioral dermatitis and create a granulomatous condition. Oral antibiotics, like Keflex 500 mg bid, are frequently used to treat perioral dermatitis. However, Keflex is not indicated for perioral dermatitis. Clotrimazole is an antifungal cream that is not used in perioral dermatitis.

What is the Dx? Pharyngitis with diffuse cervical lymphadenopathy and fever.

Mononucleosis

Orbital Cellulitis

More common in children than adults with the median age at 7-12 years. Has several possible causes, including MCly sinusitis, dental infections, facial infections, infection of the glove or eyelid, and infection of the lacrimal system, less often due to trauma. Common pathogens are Staph pneumoniae, Staph aureus, Hflu, and gram negative bacteria. In adults it occurs secondary to acute or chronic sinusitis and has many possible causative agents. A MRSA increase has been noted with this condition. May present with Fever, proptosis, eyelid edema, exophthalmos, prudent discharge, restricted EOMs and conjunctivitis. Exam will reveal fever, decreased ROM in eye and sluggish papillary response.. CBC, blood cultures, drainage cultures required. Sinus CT to determine extent. Condition requires emergent treatment with broad spectrum IV ABX until pathogen is identified. Surgical drainage for abscess formation.

A 60-year-old male complains of right scapularpain that is sharp for the last two days. He reports that the painwas preceded by tenderness and a tingling sensation. What is themost likely diagnosis?

More than 2/3 of cases of the herpes zoster virus occurs in patients over the age of 50. A herpes zoster virus flare occurs unilaterally in a dermatomal distribution. The prodromal stage can consist of neuritic pain or paresthesias prior to eruption of the rash in two to three weeks. The rash consists of vesicles that eventually crust and heal. The pain of herpes zoster can continue for months or years after the rash has resolved.

Parotitis

Most common cause is Staph aureus and is strongly linked to Mumps as a defining characteristic. Is also a complication of Sjogrens disease and anorexia nervosa. May be infectious, Autoimmune, or due to blockage. Asymmetrical swelling of the face.

Adenovirus type 3, 8, or 19

Most common cause of Viral Conjunctivitis

Laryngitis

Most common cause of hoarseness. Most common cause is viral, Moraxella and Hflu may be isolated. May present with hoarseness that is persistent after resolution of a URI. Avoid vigorous use of voice to prevent formation of vocal fold hemorrhage, polyps or cysts. Tx with erythromycin, Cefuroxime, augmentin reduce severity. Oral or intramuscular corticosteroids may be used to speed recovery if vocal hemorrhage is absent.

Presbycusis

Most common cause of sensorineural hearing loss

Thyroid Cancer

Most common form is Papillary (Best, 80%), Follicular (14%), Medullar (3%), Anaplastic (worst, 2%). Woman are more commonly affected than men and the usual presentation is a single nodule, however only 5% of nodules are malignant. Associated with childhood neck or head irradiation or exposure to readioactive isotopes or iodine with peak occurence 20-25 years later. 1/3 of medullary type are associated with multiple endocrine neoplasia type 2 (MEN2), 1/3 familial, 1/3 sporadic. May present with a painless neck swelling and have a single, palpable, nontender, firm nodule. May also have hoarseness, neck discomfort, or dysphagia. Nodule may enlarge, gland may be stony and hard. Medullary causes flushing, diarrhea, fatigue, cushings syndrome; anaplastic has signs of pressure or invasion (recurrent laryngeal nerve plasy) Dx with TSH, serum thyroglobulin is often elevated, Bx, US, MRI, PET, CT, CXR Tx with surgical resection an near total thyroidectomy are indicated, followed by radiation for anaplastic disease. Monitor for postoperative hypocalcemia. T4 HRT, Radioactive iodine ablation for residual disease.

Oral Candidiasis

Most common in individuals with local or systemic immunosuppression (corticosteroids, chemotherapy, antibiotics). May present with fluctuating throat or mouth discomfort. Erythema of the oral cavity or oropharynx with fluffy, whiter, CURD-LIKE patches overlying erythematous mucosa. White areas are EASILY SCRAPED OFF and bleed. Dx clinically, may use wet prep KOH. Biopsy. HIV testing recommended if no predisposing factors present. Tx with antifungal therapy (topical or systemic). Half-strength hydrogen peroxide or .12% chlorhexidine mouth rinses. Insure oral hygiene in denture wearers, diabetics, and the immunosuppressed.

S Pneumoniae

Most common pathogen of Acute Otitis Media

HSV 6

Most common pathogen of Apthous Ulcers

S pneumoniae

Most common pathogen of Bacterial Conjunctivitis

Seborrhea

Most common pathogen of Blepharitis

P aeruginosa

Most common pathogen of Chronic Otitis Media

S aureus

Most common pathogen of Dacryocystitis

S aureus

Most common pathogen of Hordeolum

Viral

Most common pathogen of Laryngitis

Epistaxis

Most common site is anterior, kiesslebach's plexus. Posterior site at woodruff's plexus. sphenopalatine artery. Predisposing factors include trauma, rhinitis, supplemental O2 use, septum deviation, HTN, hereditary telangiectasia (Oiler-Weber-Rend), nasal cocaine, anticoagulants, or coagulopathies. Bleeding is most common in the anterior septum where a confluence of veins forms a superficial venous plexus. Posterior bleeds are associated with atherosclerotic disease and HTN. Most cases respond to direct pressure within 15 minutes. Pt should sit, leaning forward. May need nasal tampon, cauterization or surgicel patch. May use topical vasoconstrictors (phenylephrine, 4% cocaine, oxymetazoline).

Mononucleosis

Most commonly caused by Epstein-Barr Virus (HHV4) and transmitted via saliva. KISSING DISEASE, associated with burkitt's lymphoma, nasopharyngeal carcinoma, pediatric leiomyomas, collagen vascular disease, and other disorders. Presents with fever and sore throat, may have exudative pharyngitis, tonsilitis, gingivitis, and soft palate petechiae. Severe infections exhibit malaise, anorexia and myalgias. Typically have painless posterior cervical node lymphadenopathy. May include splenomegaly (50%), a maculopapular or petechial rash (15%). Administration of amoxicillin incrases incidence of rash to 90%. Hemolytic anemia and thrombocytopenia may develop. Dx with heterophile antibody and screen tests. Will show FALSE POSITIVE SYPHILIS TEST (VDRL) or RPR in 10% of patients. Tx is based on symptoms, use nonaspirin NSAIDs and antipyretics. Antivirals will decrease viral shedding but will not affect course of illness.

Jane, a 21-year-old female, was seen in the office 10 days ago and was diagnosed with perennial allergic rhinitis and sent home with instructions for increased fluids, decongestants, and nasal steroids. She returns today with worsened symptoms of malaise, low-grade fever, nasal discharge, cough that is worse at night, mouth breathing, early morning unilateral pain over sinuses, and congestion. Physical examination reveals thick purulent nasal discharge, postnasal discharge visible in the posterior pharynx, periorbital swelling, and tenderness of sinuses upon palpation. She is 36-weeks pregnant and allergic to penicillin. Of the following, what is the most appropriate antibiotic?

Most patients with a diagnosis of acute rhinosinusitis based on clinical grounds improve without antibiotic therapy. The preferred initial approach in patients with mild to moderate symptoms of short duration is therapy aimed at facilitating sinus drainage, such as oral and topical decongestants, nasal saline lavage, and—in patients with a history of chronic sinusitis or allergies—nasal glucocorticoids. Adult patients who do not improve after seven days, children who do not improve after 10 to 14 days, and patients with more severe symptoms (regardless of duration) should be treated with antibiotics. Empirical therapy should consist of the narrowest-spectrum agent active against the most common bacterial pathogens, including S. pneumoniae and H. influenzae—e.g., amoxicillin. But amoxicillin is contraindicated in patients with urticarial reactions to penicillins, and quinolones are similarly contraindicated in pregnancy. trimethoprim-sulfamethoxazole is contraindicated in the third trimester of pregnancy. The best choice is clindamycin.

What is the tx for TM perforation?

Most will resolve on their own, however, surgical repair of TM as well as the ossicular chain may be necessary water/moisture to the ear should be avoided to prevent a secondary infection

What should you think of when you hear Optic Neuritis?

Multiple Sclerosis

Optic Neuritis

Multiple sclerosis will develop within 15 yars of the first episode in 50% of patients with this eye condition. Strongly associated with demyelnating diseases (MS, Acute disseminated encephalomyelitis), sarcoidosis, viral infections, and autoimmune disorders. Presents with unilateral vision loss developing over a few days. Pain is exacerbated by eye movements. Visual field loss is usually central scotoma. Marked color vision loss. Optic disc swelling (papillitis) with flame-shaped peri-papillary hemorrhages. Visual acuity usually improves by 95% in 2-3 weeks. Tx with IV methylprednisalone will accelerate visual recovery if cause is due to demyelinating disorder. Poor prognosis when associated with sarcoidosis, zoster, lupus, and may require prolonged corticosteroid therapy.

What is the Dx? Immigrant presents with fever and bilateral parotid swelling.

Mumps

What is the most common cause of acute non-suppurative sialadenitis in childhood?

Mumps

Treatment for chronic otitis media?

Myringotomy with T- Tube insertion

sx of pharyngitis

NEVER have a cough, lymphadenopahty, pharyngeal exudate

Barotrauma:

NO GTTS in RUPTURED EAR DRUM Tx: anti-inflammatories, decongestants

Pale, boggy masses on the nasal mucosa Chronic congestion Decreased sense of smell

Nasal Polyps clinical features

Topical corticosteroids Surgery

Nasal Polyps treatment

A child presents with foul smelling unilateral purulent nasal discharge. Most likely Dx?

Nasal foreign body

This bacteria causes bacterial conjunctivitis and patients will usu. present with unilateral copius purulent dc.

Neisseria

Copious purulent discharge, unilateral (bacterial conjunctivitis)

Neisseria gonorrhea bacterial conjunctivitis

A patient with persistent pharyngitis, negative throat culture and a history of multiple sexual encounters is likely to have infection with what organism?

Neisseria gonorrhoeae

Bilateral acoustic neuroma only occurs with which disorder?

Neurofibromatosis type II

Auricles, Tragus, Mastoid inspection and palpation

No lesions or tenderness of the auricles, tragus are non-tender, no discoloration, swelling or tenderness of the mastoid

Hot Thyroid Nodule

Not associated with thyroid cancer

A pt with a cup to disc ratio > 0.5 is diagnostic for..?

Nothing. Cup:Disc ratio > 0.5 is suspect of Open-angle glaucoma

Commonly associated with water exposure, trauma (ear cleaning or scratching) or exfoliative skin conditions (psoriasis, eczema)

OE

Patient presents with ear pain esp w/ movement of the tragus or auricle. They have redness and swelling of the ear canal and purulent exudate.What is the likely dx?

OE

A patient with chronic sinusitis has large nasal growths which resemble peeled seedless grapes. What is the most likely cause of her chronic sinusitis?

Obstruction from nasal polyps

Neural Hearing Loss

Occurs with lesions of the eighth cranial nerve, auditory nuclei, ascending tracts, or auditory cortex due to acoustic neuroma, multiple sclerosis, or auditory neuropathy. Corticosteroid therapy my be helpful.

Cataracts

Opacity of lens that may be partial or complete. LEADING CAUSE OF BLINDNESS IN THE WORLD May develop secondary to age, trauma, sun, steroids, and may be congenital in kids. Insidious onset of vision loss in persons generally >60yo May present with complaints of progressive vision loss, fixed spots, decreased color vision, or double vision. On exam, translucent yellow discoloration to the lens, on fundoscopic exam appearance of black spot on red background. Surgical extraction and lens replacement.

Cataract

Opacity of lens. UV light. Absent red reflex

This is a chronic, asymptomatic and potentially blinding dz that effects 2% of the population. causes defects in the peripheral visual field and increased cup to disc ratios. Patients typically aysmptomatic until late in disease.

Open angle glaucoma

Asymptomatic, potentially blinding Loss of peripheral vision Intraocular pressure elevated

Open-Angle Glaucoma clinical features

A patient presents to your office with a sudden onset of headache, right eye pain, decrease visual acuity, nausea and vomiting. His intraocular pressure is 47. Which of the following classes of medications are indicated for treatment of this condition? A. Alpha agonists and antihistamines B. Alpha agonists and Beta blockers C. Mydriatics D. Cycloplegics E. Angiotensin converting enzyme inhibitors

Ophthalmic alpha-agonists (brimonidine) and beta blockers (timolol) decrease aqueous humor production, and decrease intraocular pressure. They facilitate aqueous flow through outflow tract and the canal of Schlemm. Other acute treatments include prostaglandin analogs(latanoprost) and carbonic anhydrase inhibitors(acetazolamide). Mydriatics, cycloplegics(tropicamide) and antihistamines( diphenhydramine) can precipitate angle closure glaucoma in patients at risk. Angiotensin converting enzyme inhibitors are used for treating systemic hypertension.

A 45-year-old male presents to your office complaining of severe unilateral eye pain with some photophobia for one day. He denies any history of trauma. On examination and with staining, you notice a dendritic lesion to the cornea, and an otherwise normal examination. Which of the following medications would be contraindicated in this patient?

Ophthalmic corticosteroids in cases of suspected herpes simplex keratitis are contraindicated.

Papilledema

Optic disc swelling due to raised IOP. Usually bilateral, causing enlargement of the blind spot without visual acuity loss. Acute cases will result in profound loss of visual acuity. Tx with cerebrospinal fluid shunt or optic nerve sheath fenestration for those with progressive visual loss not controlled by medical therapy. Recommend weight loss when appropriate and acetazolamide.

characterized by painful visual loss and a swollen optic disc.

Optic neuritis

Acute monocular vision loss may be due to (3)?

Optic neuritis Retinal artery occlusion Giant cell arteritis

Glaucoma

Optic neuritis. Increased ocular pressure may or may not occur. No screening.

A 60-year-old male presents with complaints of irritation and a white plaque on his tongue. He denies pain. During physical exam you are unable to remove the white plaque from the mucosa with a tongue depressor. What is the most likely diagnosis, represented as follows?

Oral leukoplakia cannot be removed from the mucosa using a tongue depressor like oral thrush can. Lichen planus can mimic candidiasis, squamous cell carcinoma, or hyperkeratosis, and requires a biopsy to diagnose. Glossitis is a generalized inflammation, and loss of papillae of the tongue is caused by vitamin deficiencies, medication reactions, auto immune reactions, or psoriasis. Geographic tongue is an asymptomatic serpiginous area of atrophy and erythema of the anterior tongue. The condition is self-limiting.

Hospitalization Broad spectrum antibiotics Nafcillin, metronidazole

Orbital Cellulitis treatment

What is infected in Orbital Cellulitis?

Orbital muscles and fat (Periorbital cellulitis would involve only skin around the eye, but Orbital cellulitis involved muscles and fat behind the eye. It is life-threatening)

A 63-year-old male presents with an asymptomaticlesion in his mouth that was discovered by his dentist at a check-up.It is ill marginated with pigment ranging from medium brown to black.Parts of the lesion are raised. What is the next appropriate stepin management?

Oropharyngeal melanoma is characterized by varying pigment occurringin an irregularly shaped lesion. Although this is a rarely occurringmelanoma, a biopsy should be done and any pigmented oral lesionshould be excised. Areas which are raised within the lesion usuallyindicate sites of invasion.

Acute Otitis Media

Otalgia in the setting of an upper respiratory tract infection. Bacterial infection of mucosa-lined air spaces of the temporal bone. Often results from eustachian tube obstruction caused by viral URI. Fluid accumulation becomes infected by bacteria (strep pneumo, Hflu, Strep pyogenes). May present with otalgia, aural pressure, decreased hearing, mastoid tenderness, fever, erythema and decreased TM mobility with occasional bullae. Rupture more likely with outward bulging of TM due to severe empyema. Decrease in pain after rupture and onset of otorrhea. Swelling over mastoid or associated cranial neuropathy requires urgent care. Tx with tympanocentesis for recurrent cases after multiple courses of ABX. Myringotomy for severe otalgia or complicated cases. Insertion of tubes if prophylaxis fails. Amoxicillin is 1st line. Erythromycin plus sulfonamide may be used. Augmentin for resistant cases. Prophylaxis for recurrent cases is sulfa or amoxicillin daily for 1-3 months.

7

Otoscopic exam - pull external ear back - tilt head away - look in both ears

What causes retinopathy of prematurity?

Oxygen hitting the eyes (from O2 mask)

hordeolum

PAINFUL nodule or pustule w/in a gland in upper/lower eyelid tx: warm compress, topical AB if needed, I&D if not resolved

clinical features of retinal vessel occlusion

PAINLESS blindness or visual fiel defect usually UNILATERALLY

clinical features of open angle glaucoma

PAINLESS gradual history of occasional tunnel vision that is bilateral

what is a chalazion

PAINLESS indurated lesion deep from palpebral margin.

What's the tx for Shingles?

PO acyclovir Topical Steroids (if posterior uveitis is present)

herpes simplex eye infection tx

PO acyclovir, famicyclovir, valacyclovir topical antivirals (trifluridine) ophthalmology consult

chalmydial conjunctivitis

PO tetracycline, doxycycline, or azithromycin

Which PO ABX would be appropriate for Blepharitis? Which topical ABX?

PO- Tetracycline Topical- E-mycin or Bacitracin

13

PUPILS 1. Symmetry 2. Reactive to light

Epiglottitis

Painful swallowing out of proportion with oropharyngeal findings. Most common cause is viral due to Hflu vaccination. More common now in adults, was more common previously in children. May present with rapidly developing sore throat and odynophagia, swollen erythematous epiglottis and a muffled "HOT POTATO" voice. Xray shows THUMB PRINT sign on lateral view. Immediately protect airway in children, most adults do not require intubation. Indications for intubation include Dyspnea, rapidly advancing sore throat, endolaryngeal abscess on CT. Continuous pulse oximetry if patient is not incubated. Tx with IV ABX (ceftizoxime or Cefuroxime), dexamethasone, and airway observation. May eventually transition to oral medications.

Tx of Chalazion?

Painless Warm compress If persistent, refer to Opthalm

Cholesteatoma

Painless otorrhea. Retraction pockets. CT scan

Retinal Detachment

Painless vision loss. Curtain pulled down

4

Palpate - TMJ - have them open and close mouth

6

Palpate - external ears - mastoid process

3

Palpate - facial bones and sinuses - any tenderness/pain?

2

Palpate - scalp and head - any tenderness/pain?

25

Palpate lymph nodes - anterior cervical - posterior cervical - supraclavicular

20

Palpate nose

24

Palpate thyroid - stand behind patient - tilt head forward and down - palpate as they swallow

Most common type of thyroid cancer?

Papillary

This condition is defined as an increase in ICP. May be causes by malignant HTN, hemorrhagic strokes, acute subdrual hematoma and pseudotumor cerebri.

Papilledema

What virus causes mumps?

Paramyxovirus

Which salivary gland is most often affected with sialadenitis?

Parotid gland

19 - Nose

Patency - breathe IN through each nostril - with other one closed (hand)

A patient presents with sore throat. She is drooling, unable to speak and exam finds deviation of her uvula. Should you send her home with oral antibiotics?

Patient must be hospitalized with peri-tonsillar abscess - Needs airway monitored and I&D of abscess

Retinal artery occlusion

Patient presents with sudden monocular vision loss. Ophthalmologic emergency with poor prognosis, even if immediately treated. Commonly caused by embroil, thrombosis phenomenon, or vasculitides. Primarily effecting central or branch of retinal artery. Migraine, contraceptives, vasculitis, and thrombophilia should be considered in young patients. Presents with sudden, profound, painless, marked unilateral (monocular) vision loss. Explore carotid and cardiac sources of emboli Fundoscopy shows areteriolar narrowing (BOX CAR SEGMENTATION), separation of arterial flow, retinal edema, and perifoveal atrophy (CHERRY RED SPOT). Ganglionic death leads to optic atrophy and a pale retina. Emergent referral to ophthalmology with anterior chamber paracentesis and vessel dilation attempted. If seen within hours or onset, lay patient prone, ocular massage, high concentrations of inhaled O2, IV acetazolamide. Management of atherosclerotic disease to reduce risk.

What is the treatment for open angle glaucoma?

Patient should be referred to optho for close monitoring and chronic treatment Treatment consists of topical and/or systemic medications to decrease the IOP (BBs, acetazolamide, prostaglandin-like-medications, cholinergic mediactions, epinephrine, brimonidine) etc)

Patients who suffer with acne rosacea can relate a history of which outcome?

Patients who suffer with acne rosacea can relate a history of their condition worsening with exposure to hot temperatures, spicy foods, or alcoholic beverages. This is in response to increased reactivity of capillaries. Acne that worsens with the onset of menses is characteristic of acne vulgaris, not acne rosacea. Acne rosacea may resolve spontaneously; however, it is usually present in some form for a lifetime. Exposure to high temperatures, such as those in a steam room, can worsen acne rosacea.

A 54-year-old male patient presents to your office complaining of pain to the left eye with nausea, vomiting, and a headache after being brushed in the eye with his grandchild's stuffed animal. On examination the conjunctiva is not injected, and the cornea has a steamy appearance. You cannot visualize the retina. The pupil is fixed and 4 mm. When you stain the eye you are unable to see any lesions or scratches. You suspect:

Patients with acute glaucoma usually seek treatment immediately because of extreme pain and blurred vision, though there are subacute cases. The blurred vision is associated with halos around lights. Nausea and abdominal pain may occur. The eye is red, the cornea steamy, and the pupil moderately dilated and nonreactive to light. Intraocular pressure is usually over 50 mm Hg, producing a hard eye on palpation.

A 3-year-old male is brought to your office with a red, tearing right eye. The mother stated that the child was playing in another room with his 4-year-old brother. All she heard was the child beginning to cry. Upon physical exam, the child is intermittently crying, and his right eye is red and tearing. The child is continually rubbing the eye. The anterior chamber is clear and the pupil is equal and reactive. What is your next step in evaluating this patient?

Perform a fluorescein stain A corneal abrasion must be ruled out in a child with a red eye.

Sudden onset N/V Tinnitus, hearing loss, nystagmus

Peripheral Vertigo clinical features

Is Meniere's disease a peripheral or central cause of vertigo?

Peripheral vertigo

What is the visual loss progression in glaucoma? (Periphery to central or central to periphery?)

Periphery to central

Sore throat, pain with swallowing Deviation of soft palate/uvula Muffled 'hot potato' voice

Peritonsillar Abscess clinical features

What is the DX? Severe sore throat fever drooling muffled/hot potato voice uvula deviation.

Peritonsillar abscess

A patient presents with epistaxis from the right nares, along with direct pressure to the nares and elevation of the head. Which of the following is an appropriate initial treatment?

Phenylephrine is a topical decongestant, and acts as a vasoconstrictor to aid in stopping minor anterior septal epistaxis. Triamcinolone and momentasone are nasal steroids used for allergic rhinitis. Cromolyn sodium is a mast cell stabilizer, and azelastine is a topical H1 selective antihistamine, used for allergic rhinitis.

Medications that commonly cause gingival hyperplasia?

Phenytoin - Calcium Channel Blockers - Cyclosporine

Elevated, yellowish, fleshy conjunctival mass found on the sclera adjacent to the cornea

Pinguecula

Critical: Lymph nodes

Pre-auricular, Post-auricular, occipital, tonsillar, submandibular, submental, anterior cervical, deep cervical, posterior cervical, supraclavicular: No lymphadenopathy

Peritonsillar Abscess

Presentation includes a muffled "hot potato" voice. Caused by an infection that penetrates the tonsillar capsule and surrounding tissue. May present with sore throat, odynophagia, truisms, medial deviation of the soft palate and poeritonsillary fold, and a muffled voice. Dx with aspiration and culture of site. TX with needle aspiration, I&D, or tonsillectomy. Use of a single dose of IV amoxicillin of clindamycin in ER cases, may use oral ABX for less severe cases.

When should abrasion/ulcers NOT by covered with a pressure patching?

Pressure patching may cause further deformation of the cornea. Contact lens abrasions should NOT be patched, and should be treated with topical abx c/ pseudomonas coverage (Tobramycin, oflaxacin, cipro)

What pathogen is responsible for malignant otitis externa?

Pseudomonas

This is a slowly growing thickening of the bulbar conjunctiva that can be unilateral or bilateral. There will be a highly vascular triangular mass growing from the nasal side towards the cornea that enroaches on the cornea and interfers with vision.

Ptergyium

Highly vascular, triangular mass growing from the nasal side to the cornea

Pterygium

A 65-year-old male presents to you with a growth on the inner aspect of his left eye. He states that it has been getting slightly larger. On physical exam, you note a fleshy triangle shaped protrusion on the inner bulbar conjunctiva, touching the limbic border. Which of the following is the correct diagnosis?

Pterygium is a complication of exposure to ultraviolet light and wind. It consists of hyaline and elastin tissue. If it encroaches on the cornea, surgical removal is indicated.

12

Pull lower eyelids down - inspect conjuctivae/sclerae

How can you differentiate dacryoadentitis and dacryocystitis?

Purulent discharge, significant redness, pain, swelling, decreased feeding, failure to thrive would be seen in dacryocystitis.

How should you treat a corneal ulcer?

REFER TO OPTHALMOLOGIST stain lesion and culture

Tx for retinal vein occlusion?

Ranibizumab IV steroids Thrombolytics Surgical intervention

Tx for Wet Mac Degen?

Ranibizumab, laser tx

Acute Angle Closure Glaucoma

Rapid onset of severe eye pain with profound visual loss and "HALOS AROUND LIGHTS". Ophthalmic emergency presenting with painful eye with loss of vision, affecting people <40 years old and more commonly in African Americans or those with +family hx. Exam reveals circumlimbal injection (red eye), STEAMY CORNEA, fix mid-dilated pupil, and decreased visual acuity. IOP >50mmHg resulting in HARD EYE. Primary type occurs with closure of pre-existing narrow anterior chamber angle. Secondary type occurs in anterior uveitis, lens dislocation, or topiramate therapy. Narrow anterior chamber with acutely elevated intraocular pressure. May be accompanied by nausea, vomiting, and diaphoresis. Immediate referral to ophthalmology. Tx for primary is iridoplasty or anterior chamber paracentesis is effective if pharm therapies cause no response at initial presentation. Reduce IOP with single 500mg dose of IV acetazolamide then osmotic diuretics. Topical pilocarpine to reverse angle after IOP is decreased. Definitive Tx is iridectomy or iridotomy. Untreated dz will result in severe and permanent visual loss in 2-5 days.

A 1-day-old infant being examined in the newborn nursery is noted to have a central, 4 mm cataract affecting his right eye. Which of the following diagnostic studies should be performed as a result of this finding?

Rapid plasma reagin (RPR) Congenital cataracts may result from transmission of maternal infections such as herpes simplex virus, cytomegalovirus, toxoplasmosis, or syphilis and require further evaluation for potential systemic infection

17

Red reflex - B/L - "I see your red reflex."

What's the tx for angle-closure glaucoma?

Refer to an ophthalmologist! IV Acetazolamide (Diamox), once the pressure falls, topical pilocarpine 2%

What is the treatment for angle-closure glaucoma

Refer to optho immediately. Start IV acetazolamide, and topic timolol and osmototic diuresis (mannitol) - mydriatics should NOT be administered to patients optimal treatemnt is via laser or surgical iridotomy

Chalazion

Relatively painless, infuriated lesion deep from the palpebral margin, often secondary to chronic inflammation of meibomian gland or an internal hordeolum. Characterized by insidious onset with minimal irritation, may become pruritic and cause erythema of the involved lid. May present with complaints of a hard, non-tender, swelling of the upper or lower lid with adjacent conjunctival erythema and swelling. Large type may impress the cornea and cause distorted vision. TX with warm compress and referral to ophthalmology for elective excision if not resolved.

Tx for Allergic Conjunctivitis?

Remove offending agent Start with topicals: - Cromolyn, Olopatadine Oral Anti-histamines - Loratadine, Cetirizine, Fexofenadine, Diphenhydramine

Tympanic Membrane Perforation

Results from impact injury or explosive acoustic trauma. Persistent condition may result from secondary infections. May present with an audible whistle sound. May show prudent discharge if infection is present. Most cases heal spontaneously. Water exposure should be avoided (earplugs while swimming or bathing), until healed. Avoid ototoxic ear drops. Systemic antibiotics may be used if infection is present.

Inflammation of the nasal mucosa

Rhinitis

Lesion of RIGHT side of the RIGHT optic nerve will cause what visual symptoms?

Right nasal hemianopsia

9 - If hearing trouble

Rinne Test - Strike tuning fork - place on mastoid - after can't hear any more, place in front of ear - Do you still hear it? (Yes = nml) - It will be softer in the affected side **SENSORINEURAL hearing loss

Rotator cuff injury

SITS- Supraspinatus MC -sx- anterior deltoid pain, weakness, atrophy, Passive ROM greater than active ROM*** +hawkins, drop arm, neers test -tx- tendonitits- shoulder pendulum/ wall climbing exercises

EWING SARCOMA

SX- malignant bone tumors that present with pain and swelling. No improvement is noted with conservative therapy. MID SHAFT, SYSTEMIC -DX- Onion skin! -tx- local incision, chemo, radiation

Nasal Polyps

Samter's Triad is characterized by asthma, aspirin sensitivity, and this physical exam finding. Commonly seen in association with allergic rhinitis. Consider cystic fibrosis when seen in children. Presents with pale, edematous mucosa-covered masses. Diminished sense of smell and nasal obstruction. Surgical removal if medical management fails. Nasal corticosteroids for 1-3 months, short course of oral corticosteroids. Prevent with allergen avoidance, aspirin avoidance in those with this and asthma.

What is blepharitis associated with?

Seborrhea* Rosacea* Staph colonization

A 62 year old male presents with a concern regarding the numerous brown, warty looking papules on his back. They are asymptomatic but sometimes get caught on clothing which can cause irritation and bleeding. What is the most likely diagnosis?

Seborrheic Keratoses are described as brown, warty looking papules that have a "stuck on" appearance. They are more common in patients over 50 years old.

Blepharitis

Seborrheic, warm compress, baby shampoo

When would Neisseria Gonorrhea be considered?

Secretions, redness and pain are greater than expected *Newborns

Central Causes of Vertigo

Seizure Multiple sclerosis Wernicke Encephalopathy Chiaki malformation Cerebellar ataxia syndromes

What type of hearing loss is associated with Meniere's disease?

Sensorineural hearing loss

What type of hearing loss is associated with aging (presbycusis)?

Sensorineural hearing loss

Retinal Detachment

Separation of retina from pigmented epithelial layer, usually beginning at the superior temporal retina. May occur spontaneously or as a result of trauma. Central vision remains intact until macular involvement. May present with "CURTAIN DRAWN OVER EYES" from top to bottom. May complain of flashes of light or floaters early on. Fundoscopic exam may reveal rugous retina flapping in vitreous humor. Emergency consult with ophthalmology. Patient should remain supine with head turned towards effected side. 80% will not recur, 15% will need TX, 5% will no reattach. Methods used are laser, silicone, expansional gas, and vitrectomy, determined by underlying cause.

A 42-year-old male with a past medical history of renal failure and diabetes type II presents with facial swelling and pain. He states that it has been getting worse since it started five days ago. He also states that the side of his cheek became acutely swollen and painful five days ago when he was eating. His physical exam reveals a markedly swollen left submandibular space, with a firm and tender 1.5 cm nodule, palpable near the mandible, on the left side. When pressed, pus is seen coming out of the submandibular salivary duct. What is the most likely diagnosis?

Sialadenitis

A 45 year old male presents with progressive enlargement of his right submandibular gland over the last week. What is the most likely diagnosis?

Sialolithiasis

Pterygium

Slow growing, thickened part of the bulbar conjunctiva. May be unilateral or bilateral. Presents as highly vascularized triangular mass growing from the nasal side towards the cornea, occasionally encroaching on the cornea and interferes with vision. Tx with excision if vision is impaired. Recurrence is common.

Taking vitamin C, E, zinc, and beta carotene, and stopping smoking, have a preventative effect on the progression of which of the following diseases?

Smoking cessation and taking supplements, including vitamin C, E, zinc, and beta carotene, have shown an eight percent decrease in progression of late stage macular degeneration. Smokers or previous smokers should not take beta carotene, due to its link with lung cancer in smokers.

Taking vitamin C, E, zinc, and beta carotene, and stopping smoking, have a preventative effect on the progression of which of the following diseases? A. Macular degeneration B. Retinal detachment C. Central retinal artery occlusion D. Diabetic retinopathy E. Central retinal vein occlusion

Smoking cessation and taking supplements, including vitamin C, E, zinc, and beta carotene, have shown an eight percent decrease in progression of late stage macular degeneration. Smokers or previous smokers should not take beta carotene, due to its link with lung cancer in smokers.

A 25 year old female presents with multiple irregular brown macules on her upper back. They are asymptomatic. She has worked as a lifeguard for the past 7 years. She reports a couple of blistering sunburns and admits to maintaining a "healthy tan". What is the most likely diagnosis?

Solar lentigines are a result of skin damage from the sun. Unlike ephiledes, they do not fade once exposure to the sun has stopped. There is no risk of malignancy associated with solar lentigines.

Most common type of oral cancer?

Squamous cell (90%)

A 68 year old male drinker presents with 3 months of hoarseness. What condition must be considered?

Squamous cell carcinoma

Orbital Cellulitis

Staph & Strep. Pain with movement. EOM paralysis. CT scan.

The 4 most common pathogens responsible for Bacterial Conjunctivitis?

Staph A Strep pneu H flu M Catarrhalis

Most common organism causing painful eyelid nodule?

Staph Aureus

Most common pathogen causing sialadenitis?

Staph aureus

What're the common causes of Blepharitis?

Staph spp Viral INFX Seborrheic

What is the most common pathogen associated with sialadenitis? Which empiric antibiotic class should be used?

Staph. Aureus - Cephalosporins are first line

What most commonly causes orbital cellulitis?

Staph/Strep

Treatment for nasal polyps?

Steroids or surgical removal

3 most common bacterial organisms causing sinusitis?

Strep Pneumo #1 - H. Influenzae #2 - M. Catarrhalis #3

3 most common organisms causing AOM (Acute Otitis Media)?

Strep Pneumo #1 - H. Influenzae #2 - M. Catarrhalis #3

What are the common chronic pathogens causing dacryocystitis?

Strep pneumo H flu Pseudomonas Strep Viridans

Otitis media

Strep, moraxella, haemophilus. Tugging of ear. Immobility of TM. Bulging TM. Amoxicillin

In what area does a retinal tear most commonly begin?

Superior temporal retinal area

Tx for Dry Mac Degen?

Supportive, supplements

What's the tx for Cataracts?

Surgery!

How do you treat an entropion or an ectropion?

Surgical repair if the condition causes trauma (triachiasis), excessiv tearings, exposure keratitis or cosmetic distress

Treatment for cholesteatoma?

Surgical/possible stapedectomy with prosthesis placement

Second step in removing foreign body?

Swab attempt After that, opthalmologist.

What is papilledema?

Swelling of optic disk, may represent ICP. (Tumor, edema)

Otitis Externa

Swimming and Qtips. Otorrhea. Pain with tragus/pinna movement

Cranial Nerve 5 and 7 motor

Symmetric motor function of cranial nerve 5 and 7

Any impediment to the flow of aqeuous humor through the trabecular meshwork and canal of schlemm will increase pressure in the anterior chamber in Glaucoma. T or F

T

The prognosis for retinal detachment is good. 80% will recover w/o recurrence, 15% will require treatment and 5% will never reattach. T or F

T

Critical: Tympanic membrane

TM is pearly gray and translucent, light reflex is in anterior inferior quadrant, landmarks are undistorted

Which of the following concerning changes in nevi can be associated with melanoma?

The ABCDE's of moles are asymmetry, border irregularity, color change or irregularity, diameter >5mm, and evolving (changing in some way). These are all signs that the mole should be evaluated for possible dysplastic or malignant changes.

A 12 year old female found a tick on her leg after a camping trip. The tick was removed without incident. According to CDC recommendations, what is the appropriate testing for Lyme disease?

The CDC recommends first using ELISA to test for Lyme disease. If this is positive, then a Western Blot should be performed as confirmation. Acute and convalescent titers should be tested as only 20-30% of patients have a positive response in the acute phase. That percentage rises to 70-80% in the convalescent phase.

Since the introduction of the Haemophilus influenza B vaccine, which of the following diseases is in decline for the pediatric population, but is being seen more in adults?

The H. influenza B vaccine has significantly decreased the incidence of epiglotitis in children, leaving the adult population with a higher incidence of epiglotitis. The course of the illness in adults is not as severe as in the pediatric population. Influenza B is a seasonal virus, and requires yearly immunizations.

Which extra ocular muscles may become entrapped in a blowout fx? Which s/sx will the pt have in this case?

The Inferior Rectus and/or Inferior Oblique muscles This may cause an inability to gaze upwards, as well as double-vision, and possible subQ emphysema and/or exophthalmos

Viral Conjunctivitis caused by H. zoster is indicative of which nerve?

The Ophthalmic branch of the Trigeminal nerve. Shingles in the eye can be vision-threatening!!

A 1-day-old infant being examined in the newborn nursery is noted to have a central, 4 mm cataract affecting his right eye. Which of the following is the most appropriate management for this patient? A. Cataract surgery within the next 6 weeks B. Cataract surgery within the next year C. Observation every 3 months D. Observation every 6 months

The answer is A. EXPLANATION: Congenital cataracts that are large and affect visual acuity (e.g., central) must be surgically corrected within the first two months of life (A) to avoid the development of deprivation amblyopia. Observation (C, D, and E) or delayed surgery (B) may result in permanent deprivation amblyopia.

A patient presents with epistaxis from the right nares, along with direct pressure to the nares and elevation of the head. Which of the following is an appropriate initial treatment? A. Phenylephrine spray and anterior packing B. Triamcinolone spray and anterior packing C. Azelastine spray and anterior packing D. Momentasone spray and anterior packing

The answer is A. EXPLANATION: Phenylephrine is a topical decongestant, and acts as a vasoconstrictor to aid in stopping minor anterior septal epistaxis. Triamcinolone and momentasone are nasal steroids used for allergic rhinitis. Cromolyn sodium is a mast cell stabilizer, and azelastine is a topical H1 selective antihistamine, used for allergic rhinitis.

A 23-year-old male presents to the clinic complaining of left anterior neck pain that developed over the past week following recovery from an acute upper respiratory infection. On physical exam a tender mass is felt anterior to the left sternocleidomastoid muscle from the mandible inferiorly to the level of the cricoid cartilage. Which of the following is the most likely diagnosis? A. Branchial cleft cyst B. Dermoid cyst C. Peritonsillar abcess D. Salivary gland tumor

The answer is A. EXPLANATION: The development of a neck mass in a young adult following URI is consistent with branchial cleft cyst (A) and thyroglossal duct cyst (E). The location of this mass away from the midline and anterior to the SCM is most consistent with branchial cleft cyst (A). The location of the mass and history are inconsistent with dermoid cysts (B), which are typically midline, peritonsillar abcesses (C), which would be located in the retropharyngeal space, and salivary gland tumors (D), which would be located in the parotid, submandibular, or submental salivary regions.

A 43-year-old female presents to the outpatient clinic complaining of itching and irritation of her right eye. She denies decreased vision or photophobia. On physical exam the patient's eye has the following appearance: Which of the following is the most appropriate management for this patient's condition? A. Daily lid cleansing and application of bacitracin ophthalmic ointment 500 units/g B. Doxycycline 100 mg by mouth once daily C. Incision and drainage D. Referral to an ophthalmologist

The answer is A. EXPLANATION: The patient's symptoms of itching and irritation of the lid margin with an inflamed eyelid and eyelash scaling is consistent with mild anterior blepharitis, which is initially treated with cleansing and the potential addition of a topical antistaphlococcal antibiotic (A). Answers (C) and (E) are appropriate treatments for a hordeolum. Doxycycline once daily (B) can be used as a long-term treatment for posterior blepharitis. The condition doesn't warrant referral (D).

As a diver descends for a deep water dive, at about 10 feet of depth he begins to feel nausea, severe ear pain, and develops vertigo and vomiting. What is the most likely cause of his symptoms? A. Decompression sickness B. Decreasing pressure in the middle ear C. Benign paroxysmal positional vertigo D. Increasing pressure in the middle ear E. Equalization of pressure between the middle ear and eustachian tube

The answer is B. EXPLANATION: Boyle's law states that as a diver descends, the increasing external pressure causes an equal decrease in pressure in the middle ear, which must be equalized during the descent. If the middle ear pressure is not equalized, the tympanic membrane becomes severely retracted, due to the negative middle ear pressure. This can result in hemotympanum, hemorrhage, or tympanic membrane perforation. Ascent causes increased pressure in the middle ear as the external pressure is decreased. Equalization techniques must also be used to prevent a tympanic membrane perforation. Decompression sickness occurs on ascent, when nitrogen gas bubbles are forced into the middle ear, and vascular and lymphatic spaces.

A 60-year-old male presents with complaints of irritation and a white plaque on his tongue. He denies pain. During physical exam you are unable to remove the white plaque from the mucosa with a tongue depressor. What is the most likely diagnosis, represented as follows? A. Oral thrush B. Leukoplakia C. Geographic tongue D. Glossitis E. Lichen planus

The answer is B. EXPLANATION: Oral leukoplakia cannot be removed from the mucosa using a tongue depressor like oral thrush can. Lichen planus can mimic candidiasis, squamous cell carcinoma, or hyperkeratosis, and requires a biopsy to diagnose. Glossitis is a generalized inflammation, and loss of papillae of the tongue is caused by vitamin deficiencies, medication reactions, auto immune reactions, or psoriasis. Geographic tongue is an asymptomatic serpiginous area of atrophy and erythema of the anterior tongue. The condition is self-limiting.

A 65-year-old male presents to you with complaints of decreasing hearing, along with difficulty discerning words when in conversations in noisy environments, such as restaurants. His only medication is simvastatin for hyperlipidemia. The following is his audiogram. He has bilateral decreased high frequency hearing loss, and decreased speech recognition. What is the most likely diagnosis? A. Vestibular schwannoma B. Presbycusis C. Presbystasis D. Cerumen impactions E. Vestibulobasilar insufficiency

The answer is B. EXPLANATION: Presbycusis is age related bilateral loss of high frequency hearing, and decreased word recognition. Presbystasis is age related balance disorder. Vestibular schwannoma (acoustic neuroma) causes unilateral hearing loss. Vestibulobasilar insufficiency results from atherosclerosis of the vertebral arteries, and can cause many symptoms including double vision, speech defects, vertigo, ataxia, and drop attacks.

A 45-year-old male presents with purulent discharge from his right ear for three weeks. He states that despite being treated by his family doctor for an ear infection one month ago, the problem continues to get worse. Upon exam, you note purulent discharge in the ear canal, an erythemic tympanic membrane, and a possible perforation. What are the pathogens most likely to culture positive? A. Strep pneumoniae B. Pseudomonas aeroginosa C. Escherichia coli D. Candida albicans E. Mycoplasma pheumoniae

The answer is B. EXPLANATION: The clinical vignette describes a chronic otitis media. Usually, this refers to a complication of acute otits media with perforation. Pathogens that culture from these infections are usually pseudomonas, proteus, or staphylococcus aureus. Strep pneumoniae is often seen in acute otitis media. E.coli is a urinary tract pathogen. Candida albicans is a cause of vaginitis, and mycoplasma is a respiratory pathogen.

A 43-year-old woman presents to the outpatient clinic complaining of right eye redness, photophobia, and pain. She notes some blurred vision and denies the presence of discharge. On physical exam her visual acuity is 20/20 left eye, and 20/60 right eye. Her right eye has circumcorneal injections and the pupil is 3 mm and responds poorly to light. Her left pupil is 5 mm and responds well. Fluorescein staining of the eye is unremarkable and intraocular pressures are normal. Which of the following treatment regimens should be prescribed? A. Homatropine 5% solution four times daily B. Homatropine 5% solution four times daily and prednisolone 1% solution every 1 or 2 hours while awake C. Prednisolone 1% solution every 1 or 2 hours while awake

The answer is B. EXPLANATION: The patient's presentation of acute uveitis is best treated with topical corticosteroids and cycloplegics (B) once infectious causes (e.g., HSV) have been ruled out. The addition of a cycloplegic helps reduce pain. Antibiotic drops (D) aren't indicated for acute uveitis.

You are evaluating a patient who is complaining of facial drooping , and inability to close his eye. During the cranial nerve exam you notice he is unable to wrinkle his forehead. Based on this information what is the most likely diagnosis? A. Cerebrovascular accident B. Transient ischemic attack C. Bell's palsy

The answer is C. EXPLANATION: Bell's palsy affects cranial nerve VII, the facial paralysis conforms to the all branches of the peripheral nerve including the side of the face, eyelid and forehead muscles. An acute cerebrovascular accident would present only with a facial droop, the ability to close the eye and wrinkle the forehead would be preserved and there would likely be other focal weakness on physical exam. Horner's syndrome is miosis, ptosis and facial flushing and anhydrosis caused by abnormalities of the supercervical ganglion along the internal carotid artery.

A 37-year-old male presents to your office with a history of vision loss in his right eye. He denies any pain, and states that the vision loss occurred suddenly. He noted there was a wavy, "curtain-like" visual disturbance preceding the vision loss. Upon physical exam you notice a cherry red spot over the macula and retinal pallor. What is the most likely diagnosis? A. Macular degeneration B. Retinal detachment C. Central retinal artery occlusion D. Cerebrovascular accident

The answer is C. EXPLANATION: Central retinal artery occlusion is characterized by a sudden, painless vision loss. A cherry red spot is characteristic on the macula, along with pallor to the retina.

Jane, a 21-year-old female, was seen in the office 10 days ago and was diagnosed with perennial allergic rhinitis and sent home with instructions for increased fluids, decongestants, and nasal steroids. She returns today with worsened symptoms of malaise, low-grade fever, nasal discharge, cough that is worse at night, mouth breathing, early morning unilateral pain over sinuses, and congestion. Physical examination reveals thick purulent nasal discharge, postnasal discharge visible in the posterior pharynx, periorbital swelling, and tenderness of sinuses upon palpation. She is 36-weeks pregnant and allergic to penicillin. Of the following, what is the most appropriate antibiotic? A. amoxicillin B. trimethoprim-sulfamethoxazole C. clindamycin D. levofloxacin

The answer is C. EXPLANATION: Most patients with a diagnosis of acute rhinosinusitis based on clinical grounds improve without antibiotic therapy. The preferred initial approach in patients with mild to moderate symptoms of short duration is therapy aimed at facilitating sinus drainage, such as oral and topical decongestants, nasal saline lavage, and—in patients with a history of chronic sinusitis or allergies—nasal glucocorticoids. Adult patients who do not improve after seven days, children who do not improve after 10 to 14 days, and patients with more severe symptoms (regardless of duration) should be treated with antibiotics. Empirical therapy should consist of the narrowest-spectrum agent active against the most common bacterial pathogens, including S. pneumoniae and H. influenzae—e.g., amoxicillin. But amoxicillin is contraindicated in patients with urticarial reactions to penicillins, and quinolones are similarly contraindicated in pregnancy. trimethoprim-sulfamethoxazole is contraindicated in the third trimester of pregnancy. The best choice is clindamycin.

A 55-year-old male presents with severe swelling to his left eye. He denies injury or allergies. He states that he has had a severe sinus infection for the past two weeks. What is the most likely diagnosis for the following physical finding? A. Allergic reaction B. Eyelid abcess C. Orbital cellulitis D. Erysipelas

The answer is C. EXPLANATION: The image shows a severe orbital cellulitis. These infections often spread from paranasal sinus infections. Multiple pathogens may be involved, such as s.aureus, s. pheumoniae, and anaerobes. An eyelid abscess would be more localized. Allergic reactions that cause angioedema around the eye are usually bilateral, and also pruritic.

A 68-year-old man presents to the outpatient clinic complaining of decreased hearing in his left ear. The following is seen on otoscopic evaluation. Which of the following is the most likely diagnosis? A. Acute otitis media B. Cerumen impaction C. Cholesteatoma D. Chronic otitis media

The answer is C. EXPLANATION: The photo depicts a classic cholesteatoma (C) effecting the pars flaccida. The localized nature of the findings and lack of inflammation make otitis media (A and D) unlikely. Any perforation in the TM (E) would be secondary to the cholesteatoma and the visible TM is inconsistent with a diagnosis of impacted cerumen (B).

An 18-year-old female presents with two weeks of severe sore throat and fatigue. Her exam shows an exudative tonsillitis. A mono-spot test is positive, and a rapid strep test is positive. Which of the following medications should be avoided? A. Erythromycin B. Clindamycin C. Cephalexin D. Ampicillin E. Prednisone

The answer is D. EXPLANATION: Ampicillin should be avoided, because a high percentage of mononucleosis patients develop a fine, non-allergic maculopapular rash when given ampicillin class drugs. The remaining antibiotics are appropriate for treating group A strep. Prednisone is used to reduce the pain and inflammation associated with severe tonsillitis.

A 20-year-old male presents with cough, nasal congestion, and a low grade fever for one week. His cough seems to be getting worse, which is the reason for his visit. His past medical history includes asthma and nasal polyps. On physical exam, his temperature is 101°F, his pharynx is erythemic, and there is grey nasal discharge with a few nasal polyps seen using a nasal speculum. His lungs have a few expiratory wheezes bilaterally. What medication is to be avoided in this patient? A. Penicillin B. Acetaminophen C. Erythomycin D. Aspirin E. Ciprofloxacin

The answer is D. EXPLANATION: Aspirin should be avoided in patients with asthma and nasal polyps. Aspirin can precipitate bronchospasm in these patients, due to immunologic salicylate sensitivity.

A 5-year-old boy presents to urgent care complaining of painful lesions in his mouth that have made eating difficult the past 2 days. The mother confirms he has been unable to eat for 48 hours, but has been able to sip water. On physical exam he has a temperature of 102.6 F; numerous small vesicles and ulcers on the buccal mucosa and tongue, inflamed gingiva; and tender anterior cervical adenopathy. Which of the following is the most likely diagnosis? A. Aphthous ulcers B. Hand, foot, and mouth disease C. Herpangina D. Herpes simplex gingivostomatitis

The answer is D. EXPLANATION: The classic presentation of initial herpes simplex infection (D) includes multiple small, painful vesicles or ulcers on the mucousa with gingival involvement, fever, and adenopathy. (A), (B), and (C) all present with ulcers, but typically involve an isolated area (A), or the tonsils and posterior pharynx (B and C). Reference:

A 63-year-old female complains of a 5-day history of a persistent left-sided headache, which she has not experienced before. She also notes a tender swollen area around her left temple, which appeared around the same time. On examination you note tenderness and prominence of the left temporal artery. You order an ESR, which is 75 mm/h. What is your best course of action at this time? A. Repeat the ESR in 72 hours. B. Begin prednisone 20 mg/d and increase if symptoms persist. C. Refer to a rheumatologist for appointment next month, with a trial of nonsteroidal anti-inflammatory drugs (NSAIDs). D. Begin prednisone 60 mg/d immediately. E. Refer for a temporal artery biopsy next week, with a trial of hydrocodone for analgesia.

The answer is D. EXPLANATION: The correct answer is (D). The patient's history and physical examination findings point to giant cell arteritis (temporal arteritis) as the most likely cause, prompting immediate treatment with high-dose prednisone to prevent visual loss. The patient meets the criteria for clinical diagnosis of giant cell arteritis without a temporal artery biopsy, but it is recommended for definitive diagnosis due to the complications associated with long-term corticosteroid treatment. Treatment with prednisone should not be withheld while waiting for a temporal artery biopsy. NSAIDs and hydrocodone do not prevent the complications of temporal arteritis.

A 23-year-old man presents to the outpatient clinic for follow-up from a recent urgent care visit. He complains of sore throat, fever, fatigue, myalgias, and a rash that started 5 days ago, and have worsened since he was seen in the urgent care 3 days ago. The patient appears non-toxic with a temperature of 39.4 degrees Celsius. Physical exam reveals pharyngeal and tonsillar erythema without exudates, generalized lymphadenopathy, a morbilliform rash on his trunk, and no hepatosplenomegaly. A rapid strep screen and Monospot performed at the local urgent care were reportedly negative. Which of the following laboratory tests is most likely to confirm the expected diagnosis? A. Complete blood count B. Cytomegalovirus titer C. Group A beta-hemolytic strep culture D. HIV viral load

The answer is D. EXPLANATION: The patient presentation is consistent with acute retroviral syndrome, which is best confirmed during this initial presentation phase through direct testing for the HIV virus, such as an HIV viral load (D). The lack of tonsillar exudates, a negative monospot, and presence of generalized adenopathy make infectious mononucleosis (B and E) less likely. A CBC (A) may show lymphopenia and support the diagnosis, but it doesn't confirm the diagnosis.

A 3-year-old girl presents to the otolaryngologist for evaluation of a persistent left ear infection and drainage that have failed to respond to multiple antibiotic regimens. Which of the following is the most likely causative organism for this patient's condition? A. Aspergillus B. Chlamydia pneumoniae C. E. coli D. Streptococcus pneumoniae E. Staphylococcus aureus

The answer is E. EXPLANATION: Chronic otitis media is typically caused by P. aeruginosa, H. influenzae, S. aureus (D), Proteus species, Klebsiella pneumoniae, or Moraxella catarrhalis. Aspergillus (A) and E. coli (C) are associated with otitis externa and streptococcus pneumoniae (D) is the most common bacterial cause of otitis media. Reference:

Chronic Otitis Media

The clinical Hallmark of this disease is purulent aural discharge. Results from recurrent acute variant, certain diseases, and trauma. Pathogens include P aeruginosa, Proteus species, S aureus, and mixed anaerobic infections. May present with TM perforation, mucosal changes, osseous changes, purulent aural discharge with or without otalgia, and conductive hearing loss. Tx with regular removal of infected debris, protection against water exposure. Definitive management is TM reconstruction. Topic ABX drops, oral Cipro.

A 50 year old female presents multiple episodes of sudden onset of vertigo over the past few months. She states this began after turning her head suddenly. The events last for about 30 seconds. She complained of nausea during the spells. She denies headaches, hearing problems, focal weakness or recent illnesses. What is the most appropriate clinical intervention.

The clinical history suggests benign paroxysmal positional vertigo (BPPV). The Dix-Halpike test and Nylen Barany test are physical exam techniques to diagnose positional vertigo. The Epley maneuver is a therapeutic canalith repositioning maneuver for debris in the posterior semicircular canal, which is the cause of BPPV. The maneuver works in 80% of the patients with BPPV. A head CT would be indicated for any patient with neurologic symptoms or a new onset of headache.

A 60-year-old male presents with scaling feet for several months. The nails are spared. The patient has tried over-the-counter hydrocortisone cream with no help. KOH shows branching hyphae and spores. Which of the following should be part of an appropriate treatment regimen?

The clinical presentation and KOH results are consistent with tinea pedis or athlete's foot. Since the nails are unaffected, topical treatment with clotrimazole is appropriate. Lotrisone contains an antifungal and a steroid; this combination medication is not appropriate for this patient. Fluticasone is a class 3 topical steroid that would worsen this patient's condition. Mupirocin is a topical antibiotic and will not help resolve a fungal infection.

An 18 month old female is brought to the pediatricians office with a history of cough, fever of 102, and decreased fluid intake. Her immunizations are not up to date as the family just moved to the United States from out of the country. On physical exam she is drooling and sitting up in a "tripod position" with mild stridor. What is the most appropriate treatment indicated for this condition?

The clinical presentation suggests epiglotitis. This is an emergent airway condition. The anesthesiologist , or the pediatric otolaryngologist must be called to stand by to intubate or insert a tracheostomy if the patients airway closes. Racemic epinephrine via nebulizer relieves much of the edema to the upper airway in a patient with epiglotitis. It is a stabilizing measure until definitive care can be arranged. Oxygen and antibiotics should administered emergently also. No x-rays are indicated when the presentation is classic. Albuterol is a beta-agonist used for treatment of asthma. Budesonide,a steroid and ipratropium, an anticholinergic agent are most often used in combination with albuterol for treatment emphysema and asthma.

An 18 month old female is brought to the pediatricians office with a history of cough, fever of 102, and decreased fluid intake. Her immunizations are not up to date as the family just moved to the United States from out of the country. On physical exam she is drooling and sitting up in a "tripod position" with mild stridor. What is the most appropriate treatment indicated for this condition? A. Humidified air B. Albuterol nebulizer C. Budesonide nebulizer D. Recemic epinephrine nebulizer E. Ipratropium nebulizer

The clinical presentation suggests epiglotitis. This is an emergent airway condition. The anesthesiologist , or the pediatric otolaryngologist must be called to stand by to intubate or insert a tracheostomy if the patients airway closes. Racemic epinephrine via nebulizer relieves much of the edema to the upper airway in a patient with epiglotitis. It is a stabilizing measure until definitive care can be arranged. Oxygen and antibiotics should administered emergently also. No x-rays are indicated when the presentation is classic. Albuterol is a beta-agonist used for treatment of asthma. Budesonide,a steroid and ipratropium, an anticholinergic agent are most often used in combination with albuterol for treatment emphysema and asthma.

A 71 year old female presents with complaint of a severe headache for 2 days. The patient denies a history of headaches in the past. She complains of a 2 week periods of morning shoulder and pelvic stiffness. There also a history of jaw pain when she chews her food. The past medical history is remarkable for well controlled hypertension, hyperlipidemia. On physical exam there is a markedly tender scalp and left temporal artery. The neurologic exam is normal. Besides a temporal artery biopsy what other diagnostic test is most indicated?

The clinical scenario suggests giant cell arteritis (GCA) or temporal arteritis. An erythrocyte sedimentation rate( ESR) or C-reactive protein( CRP) can aid in the diagnosis of GCA when evaluating an inflammatory vasculitis such as GCA. Treatment is based on history and physical and not the result of the ESR. Temporal artery biopsy is the gold standard for diagnosis.

A 12-month-old female presents with bilateral lichenification, scaling, and excoriations in the antecubital fossae and popliteal fossae. Which of the following should treatment include?

The first line treatment for atopic dermatitis is a topical steroid, such as triamcinolone ointment. Oral hydroxyzine is an antihistamine that is used to control pruritis in atopic dermatitis. Clotrimazole/betamethasone diproprionate cream is a combination antifungal and topical steroid. There are no clinical indications for this medication. An antifungal such as ketoconazole is not indicated. The oral hydroxyzine is an antihistamine that is used to help control pruritis. Petroleum jelly is an excellent emollient; however, augmentin is not indicated unless there is a secondary infection. No infection is present in this patient.

The patient below comes to your office for a routine physical exam. She has no complaints. You note the following physical examination findings. There is an absence of tenderness with palpation. What is the most likely diagnosis?

The following finding represents xanthelasma, which is commonly associated with hyperlipidemia. In some familial hyperlipidemias, skin eruptions or nodules may form (xanthomas). Gout and rheumatoid arthritis are types of inflammatory arthritis and manifestations are more likely to be found around joints. Gouty tophi may also be found around the helix of the ear and are usually whitish in color in contrast to the yellow xanthelasma. There is no evidence of erythema consistent with cellulitis and the patient is asymptomatic. Lacrimal duct obstruction would likely be symptomatic and there would be tender swelling over the lacrimal duct.

You are evaluating an 80-year-old female for the first time. She has a history of mild Alzheimer's disease, for which she takes Aricept. She states that she feels fine but her daughter feels she is depressed and has been complaining of not feeling well. Her daughter admits that the patient has a history of primary hyperparathyroidism. What laboratory results would be most consistent with her diagnosis of hyperparathyroidism?

The hallmark of primary hyperparathyroidism is a high serum calcium and high intact PTH. A low intact PTH is consistent with hypoparathyroidism. The urine serum calcium is usually high in primary hyperparathyroidism. Cortisol is related to endocrine conditions affecting the adrenal cortex.

A 50-year-old male states that his eye is bothering him since yesterday. He complains of pain and redness. He states that he mowed his lawn yesterday and that it was windy outside. He attempted to irrigate the eye but still has significant irritation. He notes that it hurts to blink his eyes. What is the correct sequence of steps to treat this condition?

The history suggests a retained foreign body to the upper eyelid. A fluorescein stain will reveal significant superficial vertical scratches on the cornea. An upper eyelid eversion must be done, to inspect for and remove the foreign body. If the practitioner is successful in removing the foreign body, relief of the irritation will be immediate.

A 54-year-old female presents complaining of decreased visual acuity to her right eye over the past few hours. She denies pain, and describes having wavy vision and seeing flashes of light. Her visual acuity in the affected eye is 20/200. What condition best describes the following physical finding?

The image demonstrates a detached retina. The superior aspect of the retina appears wavy and flowing. A central retinal artery occlusion is characterized by a pale retina, as well as a cherry red spot on the macula. Open angle glaucoma does not cause acute vision loss. Angle closure glaucoma causes painful acute vision loss. Optic neuritis is characterized by painful visual loss and a swollen optic disc.

A 55-year-old male presents with severe swelling to his left eye. He denies injury or allergies. He states that he has had a severe sinus infection for the past two weeks. What is the most likely diagnosis for the following physical finding?

The image shows a severe orbital cellulitis. These infections often spread from paranasal sinus infections. Multiple pathogens may be involved, such as s.aureus, s. pheumoniae, and anaerobes. An eyelid abscess would be more localized. Allergic reactions that cause angioedema around the eye are usually bilateral, and also pruritic.

What is the most appropriate management for the lesion shown, which is noticed on a 50-year-old female?

The lesion is asymmetric with irregular margins. The optimal treatment of this lesion would be excision with 1 cm margins. A punch biopsy would only be performed if excision cannot be performed. Cryotherapy would destroy the lesion and prohibit a diagnosis and staging.

These lesions are visible on a 14-year-old female's forehead. What medication is this disorder best treated with?

The lesions are comedones (open and closed). Optimal treatment should be with topical retinoids such as tretinoin and adapalene, as these are comedolytic. Topical erythromycin is indicated in inflammatory acne, not comedonal acne as pictured. Benzoyl peroxide only has mild comedolytic activity and erythromycin has none. This combination medication would be more appropriate for inflammatory acne. Doxycycline has no comedolytic activity.

A 21-year-old man presents to the emergency department complaining of a swollen left ear (cauliflower) after he experienced blunt trauma in a collegiate wrestling match. Which of the following is the most appropriate management for this condition?

The patient has an auricular hematoma that must be evacuated to prevent cartilage necrosis or infection. answer: *Evacuation, antibiotics, and splinting* -remove the hematoma, minimize the risk of infection, and help prevent the re-accumulation of blood.

A 24-year-old female presents with hyperpigmented macules on her cheeks, nose, and upper lip. They have been present for a couple of months. Her current medications include oral LoEstrin 24 Fe, cetirizine, and a multivitamin daily. What is the most likely diagnosis?

The patient is experiencing melasma secondary to the use of oral contraceptives. This is a frequent cause of melasma. Melasma can also be precipitated by hormonal changes that occur during pregnancy. The condition will resolve upon discontinuation of the oral contraceptive. A congenital nevus is a nevus that presents within the first year of life. It is monitored in the same way as acquired nevi. They can be larger than acquired nevi, with only a slight increase in chance of malignant change over time. Post-inflammatory hyperpigmentation includes darker areas of pigmentation that can result after inflammation on the skin. Common causes include acne and atopic dermatitis. The hyperpigmentation will resolve over time. A Café-au-lait macule is a type of birthmark. It is usually light tan to light brown in appearance, and can vary greatly in size. They are usually benign, but can be associated with neurofibromatosis when more than six, with a diameter greater than 1.5 cm, are present.

A 25-year-old female presents with a complaint of dry, cracking hands for two months. She has never had any rashes or similar problems. She does not work outside the home. She is the mother of a 6-month-old healthy female. Over-the-counter lotions have been tried, but they sting with application. What is the next appropriate step?

The patient most likely has an irritant dermatitis, secondary to increased water exposure, from having an infant. Appropriate treatment includes a topical steroid, like triamcinolone. Ointments are better vehicles than creams, as they penetrate better and moisturize. Using petroleum based moisturizers are more effective than oil based in repairing the epidermis. There is no evidence at this point of an allergen causing the outbreak. If the disorder is resistant to treatment, then patch testing to determine the allergen may be an appropriate step. A punch biopsy is not indicated, unless there is no response to treatment. A KOH test would be indicated if the rash was suspicious for a fungal infection. If the patient does not respond to treatment with a topical steroid, KOH testing may be indicated.

A 45-year-old male presents with a history of thick, adherent yellow scaling in his scalp, and red scaling patches with fissuring in post auricular areas bilaterally. Which of the following are other areas of potential involvement?

The patient presents with seborrheic dermatitis. Other areas of potential involvement include the eyebrows, eyelashes, and beard area. Antecubital and popliteal fossae are common areas of involvement in atopic dermatitis; not seborrheic dermatitis. The palms and soles are not involved in seborrheic dermatitis. The trunk and neck are not usually affected in seborrheic dermatitis.

A 63-year-old female complains of a 5-day history of a persistent left-sided headache, which she has not experienced before. She also notes a tender swollen area around her left temple, which appeared around the same time. On examination you note tenderness and prominence of the left temporal artery. You order an ESR, which is 75 mm/h. What is your best course of action at this time?

The patient's history and physical examination findings point to giant cell arteritis (temporal arteritis) as the most likely cause, prompting immediate treatment with high-dose prednisone to prevent visual loss. The patient meets the criteria for clinical diagnosis of giant cell arteritis without a temporal artery biopsy, but it is recommended for definitive diagnosis due to the complications associated with long-term corticosteroid treatment. Treatment with prednisone should not be withheld while waiting for a temporal artery biopsy. NSAIDs and hydrocodone do not prevent the complications of temporal arteritis.

You are examining n 42-year-old male with the following physical examination findings (see picture below). What symptoms would most likely correspond to his diagnosis?

The patient's physical examination findings suggest acanthosis nigricans, a condition associated with diabetes mellitus. This patient would most likely present with polydipsia and polyuria at diagnosis. He may have weight loss and fatigue. Anxiety and palpitaions are not as likely to present symptoms of diabetes.

A 38-year-old male complains of increasing fatigue, weakness, weight loss, and intermittent nausea, vomiting, and diarrhea over the past few months. He has noted some agitation at times. When this first started he thought that this was related to a virus but the GI symptoms have reoccurred on multiple occasions. Labs show a complete blood count (CBC) within the normal reference range. He is noted to have hyponatremia. On examination you note the multiple areas of hyperpigmentation as seen below. His blood pressure in the office is 100/50, P = 66, T = 97.1˚F. What test would you order to confirm your suspected diagnosis?

The patient's symptoms and examination findings are consistent with a diagnosis of Addison's disease, which is most likely due to an autoimmune process that destroys the adrenal glands resulting in a chronic adrenal insufficiency. The cosyntropin (ACTH) stimulation test should reveal a low am cortisol level and an elevated ACTH level if he has Addison's disease. The dexamethasone suppression test, choice (A), is a laboratory test for Cushing's syndrome. The vasopressin challenge test, choice (B), is a laboratory test for diagnosis of diabetes insipidus. A radioactive iodine uptake scan, choice (C), is used in the diagnosis of thyroid disease (hyperthyroidism and thyroid nodules). A follicular stimulation test, choice (E), is a factitious test.

You are conducting a physical exam on a female, who was referred to you from an optometrist. She sought a visual screening due to progressive loss of visual acuity. She has not been seen by a physician in 10 years due to lack of insurance. She admits to a 15 lb weight gain in the past three years, and also complains of parasthesias in her feet. During an ophthalmoscopic exam you notice deep retinal microvascular hemorrhages, and cotton wool spots. What is the most likely cause of her visual disorder?

The patient's symptoms suggest a likelihood of diabetes. Retinal findings can include microaneurysms, deep hemorrhages, a flame-shaped hemorrhage, exudates, and cotton wool spots.

A 24-year-old female comes into the clinic complaining of a severe sore throat. She was seen three days ago at an urgent care facility, and was given amoxicillin. She states that the pain is worse, she is unable to drink fluids, and is now having difficulty swallowing. She talks with a muffled voice. A physical exam reveals a markedly swollen and erythemic right tonsil and tonsillar pillar, with the uvula deviating to the left. The patient has extreme difficulty opening her mouth. What is the most likely diagnosis?

The physical exam is highly suspicious for peritonsillar abcess, which must be considered first. Tonisllar cellulitis, or phlegmon, is swelling and enlargement of the tonsil and peritonsillar tissue, without the presence of fluctuant abcess. Uvulitis can exist with a peritonsillar abcess or tonsillitis, but isolated uvulitis usually includes symmetric swelling and erythema as a result of irritation (snoring), allergy (angioedema), or infection from upper respiratory pathogens. Diphtheria is a tonsillitis, with a characteristic gray pseudomembrane on the tonsils and upper airway, caused by corneybacterium diphtheriae.

An 8-year-old male with hair loss, pruritus, and posterior cervical lymphadenopathy has a + culture on dermatophyte test medium (DTM). Which of the following is the most appropriate treatment?

The positive DTM confirms the diagnosis of tinea capitis. The only approved treatment for tinea capitis is griseofulvin. In tinea capitis, the dermatophyte invades the hair shaft and topical treatment is not effective. An antifungal shampoo is often used as an adjunct to treatment. Keflex is an oral antibiotic, which will not help treat the fungal infection. Desonide is a topical steroid that will worsen the fungal infection.

What will fundoscopic exam reveal of the pt w/ retinal detachment?

There will be relative afferent pupillary defect. Fundoscopic exam may reveal the ridges (rugae) of the displace retina flapping in the vitreous humor

A 72-year-old farmer presents with multiple rough adherent scaly lesions on his scalp. They are better felt than seen. He reports mild pain when he inadvertently scratches them. What would the most effective management include for this patient?

These lesions are consistent with actinic keratoses. Treatment is indicated, as 10% of these lesions progress to squamous cell carcinoma. First line treatment is 5-Fu cream 5%. Excision is not indicated for actinic keratoses. Doxycycline is an antibiotic used to treat acne and various bacterial infections, and it is not an effective treatment for actinic Keratosis.

A two-year-old male presents with a noduleon the side of his index finger. His mother states that he has hadthis nodule on one prior occurrence during infancy, and it resolved onits own. What is the most likely diagnosis?

This child has a recurrent digital fibroma. It is a smooth, firm,pink nodule that occurs on the fingers and toes up through earlychildhood. Surgical excision is recommended so that the functionof the digit is not impaired.

A 5-year-old boy presents with a tense,fluid filled blister on his fingertip. What organism is most likelyto be found when the lesion is cultured?

This child has blistering distal dactylitis. This is a form ofimpetigo, and is caused by either streptococcuspyogenes or staphylococcus aureus.

A 12-year-old female presents with linearlydistributed light brown papules on her arm. They are asymptomaticand have been present for several years. The mother states that theyappear to grow as the child grows. What treatment is necessary?

This condition is consistent with a linear epidermal nevus. Theycan appear at any age, but are usually present at or shortly afterbirth. The pigmented papules are arranged linearly and can occuron any skin surface. They are not symptomatic and will grow withthe child. There is no treatment necessary.

A six-month-old infant presents to the primary care provider with complaints of a spreading rash. The physical exam shows multiple yellow-brown macules and plaques that urticate when stroked. What would an appropriate treatment regimen include?

This condition is consistent with urticarial pigmentosa, and it will resolve over time. However, certain things such as NSAIDS, codeine, and scopolamine, as well as extreme temperatures, can cause such reactions as anaphylaxis. This condition is frequently mistaken for child abuse, as the lesions can look like small finger sized bruises. It is consistent, however, with urticaria pigmentosa, which is an accumulation of mast cells in the skin, as indicated by urtication of the lesion after gentle stroking. Urticaria pigmentosa will resolve; however, it will take longer than a week to resolve. Ketoconazole cream is an antifungal that is used to treat fungal infections.

Cholesteatoma

This condition presents with granulation debris or keratin over the tympanic membrane. Variety of chronic otitis media, due to prolonged eustachian tube dysfunction. Creation of squamous epitherlium-lined sac filled with keratin, which may become chronically infected. Will usually erode bone (penetration of mastoid). May eventually erode inner ear, facial nerve, and spread intracranially (rare). Presents with epitympanic retraction pocket or marginal TM perforation that exudes keratin or granulation debris. Tx with surgical marsupialization or removal of sac.

How will papilledema look on fundoscopic exam?

This disc appears swollen and the margins are blurred with an obliteration of the vessels. The patient may be asymptomatic or may cmplain of transient visual alterations that last for seconds.

A two-month-old bottle fed infant female presents with a sharply demarcated scaling red rash on the face and in the diaper area. The mother reports that her child has been irritable and has had diarrhea. The primary care provider has treated with hydrocortisone 2.5% ointment bid for two weeks with no improvement. What should be the next step in confirming the diagnosis?

This infant is displaying classic signs of zinc deficiency. This disorder can occur in infants who are bottle fed. Treatment with topical steroids will not improve the condition until the zinc level is corrected. A punch biopsy would not help in diagnosing the zinc deficiency that this patient is exhibiting. Bacterial and viral cultures will not be helpful in establishing the diagnosis. A KOH is used to diagnose fungal infections. The KOH and bacterial culture will not be helpful in establishing the diagnosis of zinc deficiency.

A 26-year-old female presents with a history of a rash around her neck, off and on for several years. It has been treated with mid potency prescription topical steroids, only to recur again. She reports that during treatment she will discontinue wearing jewelry until the rash resolves. The patient complains of pruritis but no other symptoms. What is the most likely diagnosis?

This is a classic contact dermatitis, secondary to nickel allergy. Nickel is a very common metal that is contained in metals, clothing, and jewelry. It is a delayed, cell mediated hypersensitivity reaction, so it takes multiple exposures before an allergic response is exhibited. Atopic dermatitis is usually manifested prior to the age of six. The classic distribution is the flexural surfaces of the extremities. Herpes zoster presents with prodromal neuralgic pain two to three weeks prior to outbreak. The rash has vesicles and erythema in a dermatomal distribution that crust over after three to five days. The pain may last after resolution of the lesions. Presentation of tinea corporis is pruritic, annular scaling patches that enlarge with central clearing.

An 8-year-old male presents to his primary care provider with the onset of a new rash, consisting of small, oval, discrete scaling plaques on his trunk, and a large red plaque with overlying thin, silvery scales in the gluteal cleft. Which of the following is a potentially important historical finding in this patient?

This is a classic guttate psoriasis. An acute strep infection is a known precipitating factor of guttate psoriasis. All patients need to be checked and treated for a strep infection. Atopy has no correlation with guttate psoriasis. It is not caused by contact with an allergen or irritant. It is also not caused by an allergic reaction.

A 30 year old male presents with bright red erythema over her cheeks and nose after spending a week at the beach. She has had no prior rashes. She states that the rash is pruritic and she has had a low grade fever accompanied by lethargy. Lab results show a (+) ANA and anti DS-DNA with an elevated ESR. What is the most likely diagnosis?

This is a classic presentation of systemic lupus erythematosus (SLE). This autoimmune disorder is more common in women who are in their 20s or 30s. Frequently it is precipitated or worsened by sun exposure.

A mother presents with her four-month-old infant for a well child check. While examining the child, you notice ill defined bluish macules on the back and lumbosacral regions. What is the appropriate next step?

This is a common presentation of hypermelanosis, sometimes commonly referred to as Mongolian spots. These usually occur in patients with more pigmented complexions. They usually spontaneously resolve prior to the child entering grade school. Hypermelanosis can sometimes be mistaken for child abuse by an inexperienced practitioner; however, hypermelanosis is a common benign condition that occurs in patients with pigmented skin. There is no increased incidence associated with hypermelanosis, and no treatment is required or available.

An 8-year-old female presents with numerous discrete, skin colored papules with a central umbilication. The lesions are mildly pruritic. What is the most likely diagnosis?

This is classic presentation of Molluscum contagiosum. Molluscum primarily affects younger children, from infancy through elementary age. Lesions appear as pearly, skin colored papules with a central umbilication. They can be pruritic. Herpes simplex virus is described as a vesicle that crusts after a few days. Neurogenic pain is associated with the outbreak. The varicella zoster virus is chicken pox. These lesions are described as "a dew drop on a rose petal," due to the characteristic vesicle on a red base. They occur in crops, with some being vesicular and some being crusted. There is no umbilication. Comedonal acne primarily affects teens. These lesions are located on the face, back, and chest.

A 62-year-old female diabetic patient complains of a pruritic rash under her breasts. A physical exam shows an eroded red plaque with satellite papules. What is the most likely cause?

This is consistent with a yeast infection of the skin, caused by Candida albicans. Diabetic patients are particularly susceptible to these infections. A staph infection of the skin will have either honey colored crusting or inflammatory papules and pustules with erythema of the skin. A strep infection of the skin will have inflammatory papules and pustules with erythema of the skin. A herpes simplex infection will have vesicles that crust over after a few days. There is often preceding neurogenic pain prior to the outbreak.

A 6-year-old male presents with multiple lesions on his shins. The lesions are annular dermal plaques with a central depression. There are no epidermal changes. He states the lesions are asymptomatic. The child has no other medical problems and is a normal active child. What is the most likely diagnosis?

This is the classic distribution of granuloma annulare. These lesions commonly occur over bony surfaces and are thought to be secondary to minor trauma (such as playing soccer, normal play activities, or insect bites). The lesions will spontaneously resolve and no treatment is indicated. The distribution in this patient is similar to that of classic necrobiosis lipoidica; however, the dermal changes are classic for granuloma annulare. Necrobiosis lipoidica starts as brown-red plaques that evolve to become waxy appearing. They are commonly misdiagnosed as tinea corporis; however, there are no epidermal changes such as scaling. The lesions are completely dermal. Atopic dermatitis in a 6-year-old child is most commonly distributed on the flexural surfaces and consists of red scaling plaques that are pruritic.

A 7-year-old presents with vesicles withsurrounding erythema, located on the sides of his fingers and toesand in his mouth. He has a low-grade fever and has difficulty eating,secondary to pain. What is the most likely cause of this rash?

This is the classic eruption of hand-foot-and-mouth (HFM) disease,which is caused by the Coxsackie A16 virus. HFM is highly contagious.Patients will present with painful ulcerative lesions, which maymake it difficult to eat. There are also vesicles on the hands and feetthat are relatively asymptomatic. HFM may be associated with a low-gradefever, malaise, abdominal pain, and respiratory symptoms.

A 30 year old female complains of a dome shaped slightly erythematous nodule on her right thigh. Upon clinical examination it exhibits the "dimple sign". The patient states the lesion is not changing and is asymptomatic. What is the most likely diagnosis?

This is the classic presentation of a Dermatofibroma. The dimple sign is when a depression forms after the lesion is laterally compressed between the fingers

A 22-year-old complains of a spreading rash. He states it began about a week ago, with one large spot on his abdomen. Very shortly thereafter, the rash rapidly spread on his torso. The lesions are small oval red plaques with a collarette of scale. He states they are only mildly pruritic. What is the most likely diagnosis?

This is the classic presentation of pityriasis rosea. The large initial lesion is the herald patch. The ensuing lesions are small, oval, and have a collarette of scale. The lesions are distributed along Blaschkoe's lines, which is the "Christmas tree" distribution. Presentation of tinea corporis is pruritic, annular scaling patches that enlarge with central clearing. Guttate psoriasis presents as salmon-pink papules or small plaques with overlying fine silvery scales. Atopic dermatitis is usually manifested prior to the age of six. The classic distribution is the flexural surfaces of the extremities.

A 25-year-old female presents for a skin exam. She has no family history of skin cancer, and has no moles that itch, bleed, or ulcerate. She is concerned about a mole on her arm that is surrounded by a hypopigmented area. She states that the mole appears to be decreasing in size. What would appropriate management of the lesion include?

This lesion is consistent with a halo nevus. The depigmented macule that surrounds the nevus is similar to vitiligo and may consume the nevus; therefore, reassurance that this will resolve is appropriate for this patient. There is no need for excision with margins as this is not indicative of malignant changes. Hydrocortisone 2.5% ointment is a low potency topical steroid. This treatment is not indicated for a halo nevus. Ketoconazole is an antifungal used to treat fungal infections. There are no indications of fungal infection in this halo nevus.

The parents of a four-month-old child present with concern regarding a birthmark on the child's scalp and left side of face. The lesion has been present since birth and is growing. It has an orange, pebbly appearance. What is the appropriate treatment?

This lesion is consistent with a nevus sebaceous. It will grow as the child grows. After puberty, the lesion becomes thicker with a warty appearance. There is a slight increase in incidence of basal cell carcinoma as the patient ages. It is recommended that the lesion be excised prior to puberty, when these changes occur in response to hormone secretion. It is confined entirely to the skin; therefore, a CT is not necessary. Laser therapy is not an effective treatment for a nevus sebaceous. It requires complete excision. It also has an increased risk of basal cell carcinoma. It will not spontaneously resolve.

A 28-year-old male presents with a tan-pink, well-demarcated waxy plaque, with raised firm borders located in the pretibial region of the left lower extremity. What is the next appropriate step to confirm the diagnosis?

This lesion is consistent with necrobiosis lipoidica, and is associated with diabetes mellitus. The patient should undergo glucose tolerance testing to be evaluated for diabetes.

An 8-year-old male with a history of atopicdermatitis presents with a widespread rash consisting of vesiclesand eroded lesions. What is the causative organism?

This patient has eczema herpeticum. This is caused by the herpessimplex virus. Transmission can occur innocuously via the parent.Atopic dermatitis is a risk factor for eczema herpeticum, secondaryto the impaired barrier function of the skin. This impaired barrier functionallows the virus to spread rapidly.

A 78-year-old Caucasian female has a 3-year history of stiffness and achiness of bilateral shoulders and hips. She has been tested for rheumatoid arthritis in the past and has been found negative. Multiple radiographs of her hips and shoulders are unremarkable. She admits that she was placed on prednisone for an allergic reaction and noted a temporary resolution of her symptoms. For the past two weeks she complains of increasing symptoms now involving her neck and pain in her jaw with chewing. Today she noticed that her scalp is sore when she brushed her hair on the right side. What is the most feared complication of this condition that may be prevented with prompt diagnosis and treatment?

This patient has long standing symptoms of PMR with current symptoms suggestive of giant cell (temporal) arteritis. Visual loss is the most feared complication of temporal arteritis, but it can be prevented by prompt initiation of high-dose prednisone. PMR often occurs with or prior to development of temporal arteritis and is not considered a complication. Large vessel involvement--which may result in choices (B), (C), and (D)--is less common than temporal artery involvement in GCA. The patient does not have symptoms of large vessel involvement.

A mother presents with a four-month-old male infant complaining of a dry, itchy rash that never seems to completely resolve. There are days when it appears to improve. She currently uses baby wash and baby lotion to care for his skin. She recently discontinued the lotion because he screams when it is applied. Which of the following is this condition exacerbated by?

This patient has the classic presentation of infantile atopic dermatitis. There are numerous factors that can irritate this condition including frequent (more than once a day) or long baths, soap based cleansers, cold dry environments, illness, stress, itchy clothing, and allergies. Lotions may sting, especially if the skin is dry and the skin barrier is broken. Atopic dermatitis usually improves in warm, humid environments. Petroleum based moisturizers are an important part of the treatment of atopic dermatitis. Soap free cleansers are recommended for patients with atopic dermatitis, as they are less irritating and drying. Topical steroids are the first line treatment of atopic dermatitis.

Parents present with their 8-month-old childwith complaints of a new rash. This rash appears as monomorphic,flat, and red papules, some of which coalesce to form plaques. Theyare present on the face, buttocks, and extremities. What is theappropriate management of this condition?

This patient has the rash of Gianotti-Crosti. It is also calledpapular acrodermatitis of childhood. This rash starts with classicmonomorphic, flat, and red papules that coalesce to form plaques.It is symmetric in its distribution and classically affects theface, elbows, and knees. It will resolve on its own; therefore,no treatment is necessary.

A 12-year-old male presents with hyperkeratoticpapules located on both hands. What is the causative organism?

This patient has verruca vulgaris orthe common wart. The causative organism is the human papillomavirus(HPV). They can affect patients of any age and can occur on anyskin surface. There is a predilection for the hands and fingers.

A 4 year old male presents with a fever for 3 days. His highest temperature was 39.4C. His mother brings him to the ED because she noticed this morning that his palms and soles were red. Now, there is blotchy erythema on the trunk with bulbar conjunctivitis and diffuse erythema on the tongue and prominent papillae. CBC shows leukocytosis. What is the appropriate management of this patient?

This patient is exhibiting classic signs and symptoms of Kawasaki Disease. Complications of Kawasaki Disease include coronary artery aneurysms, myocarditis, myocardial ischemia or infarction, and stroke. Recommended treatment is hospitalization to monitor for complications and administration of IvIg with aspirin.

An 18-year-old male presents with a rashconsisting of erythematous target-like lesions on his arms. Physicalexam shows a healing cold sore on his lips, but no other skin or mucosallesions are present. What is the most likely cause?

This patient is exhibiting the classic rash of erythema multiformeminor. Etiology of this disorder can be traced to herpes simplexoutbreaks or other viral or bacterial infections. Recurrent diseaseis most often associated with herpes simplex outbreaks.

A 14-year-old female presents with a 24-hour history of episodic outbreaks on her hands and feet. She describes the outbreaks as beginning on the sides of her fingers and toes, with small intensely pruritic vesicles. What should be the next step in treatment?

This patient is experiencing probable dyshidrotic eczema. It is necessary to rule out a secondary bacterial infection, so a bacterial culture is necessary. It is also necessary to rule out a fungal infection or parasitic by performing a KOH. This does not have any features of a viral infection, so a viral culture is not necessary. A punch biopsy is also not necessary because it can be diagnosed with a non-invasive procedure. A shave biopsy is not indicated because non-invasive techniques can be used to diagnose. Shave biopsies are not indicated when vesicles are present.

A 6-year-old male presents with crusted erythematous lesions on the nose, mouth, and chin. He has a history of atopic dermatitis. Which of the following should be part of an appropriate treatment regimen?

This patient is suffering from impetigo. Impetigo is easily treated with a topical antibiotic, such as mupirocin. Hydrocortisone is a topical steroid and may worsen the infection. Lotrisone is a combination medication that includes an antifungal and a topical steroid, neither of which is indicated in this patient. Ketoconazole cream is used to treat fungal infections, not bacterial infections.

A 15-year-old obese male presents with annularlyconfigured lesions in a generalized distribution. They are asymptomatic.What can these lesions be associated with?

This patient presents with a generalized form of granuloma annulare(GA). Generalized GA can be associated with diabetes mellitus.

A 15 year old male presents with lesions on his palms, dorsum of his hands and lower arm. They began as red macules that developed a central vesicle of a few days. The lesions are pruritic with no other symptoms. What is the most common etiology?

This patient presents with the classic iris or target lesion of erythema multiforme (EM). The most common cause of recurrent EM is herpes simplex outbreak which usually precedes EM by a few days.

A six-month-old male has recurrent diaper rashes, which are not responding to ketoconazole cream or zinc oxide diaper creams. Physical exam shows well demarcated perianal erythema, with scattered red papules on the buttocks. A KOH is negative. What would appropriate management include?

This rash is consistent with a perianal staph or strep infection. This bacterial infection is easily treated with topical mupirocin. If the rash does not resolve after two weeks of topical treatment, treatment with an oral antibiotic such as Keflex is usually successful. Treatment with a topical steroid may worsen the infection. Tretinoin is a topical retinoid indicated to treat acne, and is not appropriate for this patient. Lotrisone is a combination medication that includes an antifungal and a topical steroid, neither of which is indicated in this patient.

A 16-year-old male complains of a recurrent rash that is noticed each year during the summer. He states that the rash is asymptomatic but is spreading. A physical exam shows small hypopigmented macules with fine scale. A KOH exam shows budding yeast. What is the most likely diagnosis?

Tinea versicolor is a yeast infection that primarily affects teens and young adults. The area of infection is usually the upper back, upper chest, and lower face. It occurs in warm, humid environments and can recur yearly. The appearance of the infection can be hypo- or hyper-pigmented, slightly scaling macules. The characteristic microscopic appearance on a KOH is described as "spaghetti and meatballs," due to the shortened hyphae and spores of the yeast. Atopic dermatitis in a teen patient would normally be described as red scaling plaques that affect the flexural surface. Atopic dermatitis is also described as being pruritic. Post inflammatory hypopigmentation occurs following an inflammatory reaction on the skin. The hypopigmentation is not scaling and will have a negative KOH. Tinea corporis can be hypopigmented and scaling, but usually exhibits central clearing and pruritis. It does not recur seasonally and affects exposed surfaces. A KOH will show branching hyphae and spores.

Dysfunction of Eustachian Tube

Tube that connects the middle ear to the nasopharynx. When compramised, air becomes trapped in the middle ear causing negative pressure, usually results from viral URI or allergies. Presents with aural fullness, fluctuating hearing, discomfort with barometric pressure changes. Yawning or swallowing may elicit a pop or crackle. Retraction and decreased mobility of TM is present. Tx with forced autoinflation (unless active intranasal infection is present). Use of systemic or intranasal decongestants and oxymetazoline. Prevent with intranasal corticosteroids for allergies. Avoid air travel, altitude changes or driving during active disease.

Pain, decreased hearing and bloody drainage from the ear after a recent diving trip is the most likely what diagnosis?

Tympanic membrane perforation due to barotrauma

Bacterial Keratitis

Type of Corneal Ulcer commonly seen in people who wear contacts while sleeping, or following trauma or surgery. MC pathogens are pseudomonas, pneumococcus, Moraxella and staph. Has hazy cornea with central ulceration and hypopyon. Tx with high concentration topic antibiotics (typically quinolones) for 48 hours. Add aminoglycoside if quinolone resistance is prevalent in the area.

Herpes Zoster Ophthalmicus

Type of Corneal Ulcer that frequently involves ophthalmic division of the trigeminal nerve. Patients present with malaise, fever, HA, and periorbital burning and itching. Symptoms may precede eruption by a day or more. Rash is vesicular, then pustules develop which quickly crust. Lesions of the tip of the nose or lid margins predict eye involvement. Ocular symptoms include conjunctives, Keratitis, episcleritis, anterior uveitis, elevated intraocular pressure. HIV is a strong risk factor. Tx with high dose acyclovir, valacyclovir, or famciclovir started within 72 hours of rash presentation reduces ocular complications and postherpetic neuralgia.

Herpes Simplex Keratitis

Type of Corneal Ulcer that has recurrent cases that may be observed with colonization of trigeminal ganglion. Dendritic (branching) ulcer is characteristic. Tx with debridement, patching, topical antivirals. Consider long term oral antivirals in specific populations.

acute sinusitis

URI leads to edema which blocks drainage of the sinuses leading to fluid buildup & bacterial colonization; commonly caused by strep pneumo, GABHS, H. flu, M. catarrhalis

Bacterial conjunctivitis

Unilateral purulent discharge

Peritonsillar Abscess

Unilateral sore throat, hot potato voice, trismus, displaced uvula. I&D

A 14-year-old female patient presents to your family practice clinic having received a 1% total body surface area first and second degree burn to the left forearm. Of the following, what would you recommend for your patient?

Unless a critical surface (face, genitalia or hands) is involved, first and second degree burns may be treated in the outpatient setting. Blisters may be left intact as a physiologic dressing, and deroofed after they rupture. The patient requires tetanus prophylaxis and a topical antibiotic cream, usually either silver sulfadiazine or bacitracin. Sulfadiazine may permanently stain skin, so use it cautiously in potentially exposed skin areas for cosmetic reasons.

Which of the following is a result of untreated or partially treated otitis media, which presents with fever, ear pain, otorrhea, tenderness behind the ear, fluid collection, and destruction of air cells seen on head CT?

Untreated or partially otitis media can result in mastoiditis. Tenderness, redness, and fluctuance over the mastoid bone is characteristic. Peritonsillar abcess symptoms include severe sore throat, drooling, dysphonia, and outpouching of the tonsillar pillar on the affected side and trismus. Ethmoid sinusitis presents with nasal congestion, discharge, and headache. Suppurative otits media is contained in the middle ear, without spreading to adjacent structures.

Corneal Abrasion

Usually a result of minor trauma, such as AZ fingernail, contact lenses, eyelash or small FB. Presents with severe pain and sensation of FB and may be accompanied by photophobia, tearing, injection, and blepharospasm. Record visual acuity BEFORE examination or treatment. Slit lamp or fluorescein stain will reveal defect but a clear cornea. Must search for FB. Tx with topical anesthetics for immediate relief; however, only to assist in confirming Dx, DO NOT PRESCRIBE. ABX ointment, Acetaminophen or NSAIDS for analgesia. Patching for no more than 24 hours, only for large abrasions (>5-10mm), longer than 24hrs retards healing. 1-2 day follow ups are essential.

Patient presents with acute onset of unilateral or bilateral erythema of the conjunctiva, copius watery discharge and ipsilateral tender preauricular lymphadenopahy. It started yesterday. What are you suspicious of?

VIRAL conjunctivitis

What condition is most likely in a patient with acute vertigo who was recently diagnosed with toxoplasmosis?

Vestibular neuronitis

Peripheral Causes of Vertigo

Vestibular neurotics/labryinthitis Menieres disease Benign positional vertigo Ethanol intoxication Inner ear barotrauma Semicircular canal dehiscence

In which type of Conjunctivitis is tearing "profuse?"

Viral

Is the most common cause of pharyngitis viral or bacterial?

Viral

Is the most common cause of sinusitis in adults viral or bacterial?

Viral

Acute onset of erythema of the conjunctiva Copious watery discharge Ipsilateral tender preauricular lymphadenopathy Highly contagious

Viral Conjunctivitis clinical features

What is left supraclavicular adenopathy called?

Virchows node (often associated with lung/gastric malignancy)

A 12-year-old presents with sharply demarcated depigmented macules on bilateral knees. The parents report that this began about one year ago, after the child fell and skinned his knees. After healing, he was left with these markings. What is the most likely diagnosis?

Vitiligo is the most likely diagnosis. Vitiligo is an autoimmune disorder that affects the melanocytes. There is often a history of trauma that can precede an occurrence of vitiligo. The course for vitiligo is variable. Sometimes it will resolve spontaneously. Other times, it will continue to progress despite treatment. Post-inflammatory hypopigmentation are areas of lighter pigment, not complete depigmentation, which result from a resolved inflammatory process. The pigment will return over time. A hypertrophic scar is one which is enlarged but stays within the borders of the original injury. No pigment changes are associated with these scars. Pityriasis alba is a yeast infection of the skin, and usually appears on the face. It usually occurs in elementary school aged children, and is more apparent in late summer or early fall.

8 - If hearing trouble

Weber Test - strike tuning fork and place on midline of skull - where do you hear it loudest? - Sound will be louder on affected side **"CONDUCTIVE" hearing loss

ultraviolet keratitis

Welder's flash; severe pain 6-12 hrs after exposure; treated with cyclopentolate

Macular Degeneration can be described as either "wet" or "dry". Which happens quickly?

Wet happens quickly. Due to neovascularization

Topical steroids

What do you avoid with herpes of the eye?

DONT remove object Ophthalmology referral Analgesics Prophylactic antibiotics

What do you do with penetrating trauma to the globe?

Dendritic lesions on fluorescein stain

What do you see with herpes of the eye?

You are examining a 65-year-old male who complains of partial vision loss in his right eye. Besides obtaining visual acuity, what is the most important physical exam to perform in order to evaluate his condition?

When a patient complains of vision loss, all of the choices are important parts of the eye examination; the visual field by confrontation exam is a screen to detect visual field defects.

Blepharitis

When bacterial in origin, this inflammatory eyelid condition is usually caused by Staphylococci. Chronic inflammation of the eyelid margins. Caused by seborrhea, strep or Staph infection, dysfunction of the meibomian glands. Anterior involves the eyelid, lashes and nearby glands. May become ulcerative due to infection with sataph, or seborrheic due to seborrhea of the scalp, brows, and ears. Posterior results from meibomian gland inflammation. Staph may also cause infection in the posterior type. Glandular dysfunction in posterior type is strongly associated with acne rosacea. May present with irritation, burning, and itching of the eyelids. Anterior presents as "RED RIMMED" eyes with scales or granulations adhering to lashes. Posterior presents with hyperemic lid margins with telangiectasias, inflamed meibomian glands and ducts, abnormal secretions, FROTHY or GREASY tears, and mild entropion. TX with lid scrubs using baby shampoo on cotton tip swab and/or topical antibiotics if infection is suspected.

Leukoplakia

White lesion of the tongue that cannot be scraped off the mucosal surface. Hyperkeratosis in response to chronic irritation (dentures, tobacco, lichen planes). Up to 6% of lesions are dysplastic or early Squamous cell carcinoma. Presents with a white lesion that cannot be scraped off the mucosal surface. Dx with biopsy, reveals Hyperkeratosis. No approved treatment available.

Oral leukoplakia

White patched. Cant be scraped. Pre-cancerous. Biopsy

What anatomical location is the most common site of posterior epistaxis?

Woodruff's Plexus is the most common site for posterior nose bleeds

A 12-year-old male exhibits prominent brownscaling on his neck, trunk, and extremities with involvement offlexural regions. The palms and soles are spared. What is the most likelydiagnosis?

X-linked ichthyosis is a recessive condition affecting males.The patient will have large scales that appear brown in color. This condition spares the palms and soles, and begins between the ages of two to six weeks.

Do thyroid masses move with swallowing.

Yes

Does thyroglossal duct cyst move with tongue protrusion?

Yes

acidic

_____ burns of the eye cause coagulative necrosis

alkali

______ burns of the eye are the worst type of chemical burns; denatures proteins & collagen & causes thrombosis of vessels

posterior (blepharitis)

_______ blepharitis is caused by a dysfunction of the meibomian gland; treated with eyelid massage

posterior

_______ lymphadenopathy is seen with mono

anterior

_______ lymphadenopathy is seen with strep pharyngitis

A "Stye" is AKA . . .

a Hordeolum

What usually precedes otitis media?

a URI that leads to eustachian tube dysfunction or blockage

cavernous sinus thrombosis

a blood clot in an area at the base of the brain, in the cavernous sinus, that contains a vein, which carries blood from the brain to the heart

what is the number one cause of optic neuritis

a demyelinating disease i.e. mutiple sclerosis

blow out fracture

a fracture of one or more of the bones surrounding the eye and is commonly referred to as an orbital floor fracture

dental abscess

a localized collection of pus associated with a tooth

what are the findings of hypertensive retinopathy on fundoscopic exam

a/v nicking, copper/silver wiring, papilledema, flame hemorrhages, cotton wool spots

cholesteatoma

abnormal growth of squamous epithelium leading to mastoid bony erosion; causes epitympanic retraction pocket & TM perforation; treated with surgical excision

arterial narrowing

abnormal light reflexes on dilated tortuous arteriole shows up as colors- copper/silver; seen with hypertensive retinopathy

sx of vertigo

accelerated with movement nystagmus peripheral: sudden, N/V, tinnitus, hearing loss, horizontal nystagmus central: slower onset, nonfatigable nystagmus, motor or sensory defecitis accompany

acute angle closure glaucoma tx

acetazolamide to lower pressure and once pressure drops then start topical pilocarpine

vestibular schwannoma

acoustic neuroma; common benign intracranial tumor; causes central vertigo & unilateral hearing loss; treated with microsurgical excision or sterotactic radiotherapy

eye pain, blurred vision, halos, steamy cornea, hard globe. what dx

acute angle closure glaucoma

bacterial rhinosinusitis

acute sinus infection; caused by strep pneumoniae, H. influenzae, S aureus, M. catarrhalis; causes facial pain/pressure, fever, headache, fatigue, cough

sx of dacryoadenitis

acute: rapid, unilateral, severe pain, redness, and pressure in the supratemporal region chronic: MORE COMMON, uni/bilateral, painless enlargement of the lacrimal gland present for more than a month looks like swelling of the lateral third of the upper lid (S-shaped lid)

viral conjunctivitis

adenovirus 3, 8, 19 HIGHLY CONTAGIOUS, even by swimming pool

typical cause of viral conjunctivitis

adenovirus 3, 8, or 19

postseptal cellulitis tx

admit to hospital IV 2nd or 3rd gen ceph, ampicillin/sulactam, carbapenems, clinda

tx of orbital cellulitis

admit, broad spectrum AB (staph and strep most commonly)

cause of macular dengeration

age, toxic effects of drugs (chloroquin/phenothiazine)

results of a normal rinne test

air conduction is greater than bone conduction. the patient hears the fork when placed beside the ear

ocular antihistamines used for

allergic conjunctivis

IgE mediated reactivity to airborne antigens (pollen, molds, dust). commonly occurs in people who have other atopic dz and those with fam hx.

allergic rhinitis

tx of OM

amoxicilin or azithromycin recurrent (x3 in 6 months) --> tubes if untx risk of mastoiditis --> IV ABs

AOM tx

amoxicillin 2nd line: amoxi/clav, cefaclor, cefixime, erythromycin

In sinusitis, antibx can be recommended in extended duration of symptoms... >10days or severe symptoms such as facial pain. What is the first line drug?

amoxicillin usu. 7-10 days azithromycin, bactrim or doxy can be used in penicillin allergy. augmentin may be used if there is no improvement after 3 days of first line agents or w/ recent antibx use.

hordeolum

an acute, painful staphylococcal abscess of the eyelid; treated with warm compress &/or antibiotic ointment

Mc location of epistaxis

anterior

he source of most cases of epistaxis comes from what anatomic location?

anterior nasal septum-95 percent of epistaxis come from Kesselbach's plexus, which is a superficial, fragile group of arterioles and veins that are the most likely cause of nosebleeds. Five percent are posterior bleeds that originate along the sphenopalentine artery.

keisselbach's plexus

anterior nose bleeds come from __________; most common site of nose bleeds

periorbital cellulits

anterior to orbital septum (so limited to eyelids) -from trauma, skin infection

anterior/ posterior glenohumeral shoulder dislocation

anterior-sx- arm abducted, externally rotated -dx- axillary and Y view determine anterior vs. posterior -tx- reduction, must rule out axillary nerve injury (pinprick over deltoid) posterior- adducted, associated with siezure, shock -tx- reduction

uveitis

anterior: inflammation of iris (iritis) or ciliary body (cyclitis) posterior: choroid inflammation etiology: systemic inflammatory disease: may be associated with HLA-B27 spondyloarthropathies, sarcoid, Behcets disease infectious: CMV, toxoplasmosis, syphilis, TB trauma anterior: unilateral ocular pain/redness/photophobia; excessive tearing but no discharge - blunt trauma posterior: blurred/decreased vision, floaters, absent sx of anterior involvement, no pain PE: ciliary injection (limbic flush), consensual photophobia, possible visual changes, inflammatory cells and flare within aqeuous (WBCs and protein) - topical steroids for anterior, homatropine - systemic steroids for posterior

anterior vs posterior blepharitis

anterior: staph infection, viral or seborheic posterior: dysfunction of meibomian glands

tx of otitis externa

antibacterial drops, keep dry if immunocompromised or DM risk of malignant OE --> IV AB

ofloxacin

antibiotic ear drops used to treat otitis externa & chronic otitis media; 5 drops QD x7 days (6mo-13yr), 10 drops QD x7 days (13+)

treatment for corneal abrasion

antibiotic ointment like polymyxin/bacitracin. acetaminophen for analgesia. patching only for large abrasions greater than 5 - 10 mm.

amoxicillin

antibiotic used to treat acute otitis media & serous otitis media

nafcillin

antibiotic used to treat most orbital cellulitis

flagyl/clindamycin

antibiotic used to treat orbital cellulitis caused by an anaerobic bacteria

ceftriaxone/cefazolin

antibiotics used to treat orbital cellulitis caused by trauma

What is the treatment for labrynithitis?

antibx are indicated with assoc fever or signs of bacterial infection vestibular suppressants are helpful during acute sx

Your patient presents with redness and swelling of the ear canal, and pain with movement of the auricle. You diagnose OE. What is your treatment plan?

antibx otic drops (aminoglycoside or fluoroquinolone + steroids) and avoid further moisure or ear injury

tx for allergic conjunctivitis

antihistamines topical steroid mast cell degranulation inhibitor

valganciclovir/foscarnet

antivirals used to treat CMV retinitis

tx for epistaxis

apply pressure 10-15 minutes can cauterize or pack

What is the tx for PTA?

aspiration, I&D and antibx. tonsillectomy considered.

what drugs are associated with increased tinnitus

aspirin and NSAIDS

blepharitis

associated with seb derm and rosacea. crusting/flaking eyelid rims "red swollen lids" usually staph tx: warm compresses

sx of TMP

audible whistling sounds during sneezing and nose blowing, decreased hearing, increased ear infections if with infection: painful, purulent drainage

dx of meniere's ds

audiogram caloric stimulation (will be abnormal)

What is the treatment for allergic rhinits?

avoid any known allergens and use antihistamines, cromolyn sodium, nasal or systemic corticosteroids, nasal saline drops or washes and immunotherapy

tx of blepharitis

baby shampoo topical AB if infected

sialadenitis

bacterial (S. aureus) infection most commonly affecting submandibular & parotid glands; causes swollen, red & tender gland; treated with nafcillin

acute otitis media

bacterial infection of middle ear; caused by strep pneumoniae, H influenzae, or strep pyogenes; causes erythema & decreased mobility of tympanic membrane & bulging; treated with amoxicilin

periorbital cellulits

bacterial infection of the skin surrounding the orbital area; causes swelling, tenderness & erythema; normal eye exam; tx w/ oral antibiotics

Patient presents with purulent nasal dc, facial pain and pressure, nasal obstruction, congestion and fever. Likely dx?

bacterial sinusitis

bacterial vs viral vs chlamydial vs allergic

bacterial: minimal itchy, generalized bright red, moderate tearing, profuse discharge, no adenopathy, occassional sore throat and fever viral: minimal itchy, generalized hyperemia, profuse tearing, minimal discharge, common preauricular adenopathy, occasional sore throat and fever chlamydial: minimal itchy, generalized hyperemia, moderate tearing, profuse discharge, never sore throat or fever allergic: severe itching, generalized milky hyperemia, moderate tearing, minimal discharge, no adenopathy or sore throat

brachial cleft cyst

benign. SIDE OF NECK tx by excision

vertigo symptoms that are brought on by changing in position of the head. dx?

bernign paroxysmal positional vertigo

treatment for open angle glaucoma

beta blockers, prostaglandin analogs (latanoprost). carbonic anhydrase inhibitors, and alpha agonists

tim/carte-olol

beta-adrenergic blocking agents used to treat chronic glaucoma

raccoon eyes

bilaterial periorbital ecchymosis seen with basilar skull fracture; diagnosed fracture with CT

cause of cholesteatoma

birth defect chronic ear infection

what shows on fundoscopy for a cataract

black on a red background

sx of hyphema

bleeding eye pain photophobia vision abnormalities

blot & dot hemorrhages

bleeding into the deep retinal layer; seen with nonproliferative diabetic retinopathy

flame-shaped hemorrhages

bleeding into the nerve fiber layer; seen with nonproliferative diabetic retinopathy

oral candidiasis

bleeds after scraping off erythematous base DM or HIV? Tx: fluconazole

Patient presents with "red rimmed eyes" and adhered eyelashes with dandruff like deposits (scurg) and fibrous scales (collarettes) on exam. The conjuncitva is clear but there is thick cloudy dc visible. What is the likely dx?

blepharitis

scotomas

blind spots, shadows; seen with macular degeneration

retinal hemorrhages

blood & thunder appearance seen with retinal vein occlusion

subconjunctival hemorrhage

blood between conjunctiva & sclera; generally asymptomatic; no treatment is required

Hyphema

blood in anterior chamber trauma HOB 30 degrees

hyphema

blood in anterior chamber of eye (behind cornea, in front of iris)

whats a hyphema

blood in the anterior chamber following trauma

dx of dacryoadenitis

blood/discharge culture lacrimal gland biopsy (if doesn't subside after 2 weeks) r/o sarcoidosis, graves, sjogrens, syphilis, gonorrhea CT

Allergic shiners are...

bluish discoloration below the eyes

sx of retinal detachment

blurred/blackened vision, curtain falling, floaters, flashing lights

allergic rhinitis

boggy turbinates blue allergic shiners allergic salute IgE RAST test = radio allergosorbent test

results of an abnormal rinne test

bone conduction is greater than air conduction; patient does not hear the fork placed beside the ear

labyrinth

bony & membranous part of the inner ear, consists of cochlea & vesibular system

otosclerosis

bony overgrowth of the stapes bone leading to conductive hearing loss; presents with slowly progressive hearing loss & tinnitus; treated with a stapedectomy with prosthesis

complications of cholesteatoma

brain abscess, meningitis

In macular degeneration, drusten deposits are found in the ____ membrane, leading to degenerative changes, loss of nutritional supply, atrophy and neovascular degeneration.

bruchen

sx of cavernous sinus thrombosis

bulging eye HA loss of vision

bunion/ hammer toe

bunion- hallux valgus- deformity of bursa over 1st metatarsal - hammer toe- deformity of PIP joint with flexion of PIP joint and hyperextension of MTP and DIP joint

glossodynia

burning & pain of the tongue; caused by DM, meds, tobacco, candidiasis; treated with alpha-lipoic acid & clonazepam

sialolithiasis

calculus (stone) formation in salivary glands; more common in Wharton duct; causes post-eating pain & local swelling; teated with removal or lemon drops

bullous myringitis

caused by mycoplasma pneumoniae; causes erythematous & blistering TM; treated with azithromycin & augumentin

gingival hyperplasia

caused by: CCB, phenytoin, cyclosporine

orbital cellulitis

cellulitis involving tissues that surround the orbital area as well as structures of the eye; causes fever, redness, swelling/pain of periorbital tissues & decreased vision; treated with IVantibiotics

ludwig angina

cellulitis of sublingual & submaxillary spaces; usually secondary to dental infections; can be caused by strep, staph, etc.; causes edema & erythema of upper neck under chin & floor of mouth, tongue displaced; treated with securing airway, & IV antibiotics

CHERRY RED SPOT =

central retinal artery occlusion

Opthalmic emergency with poor prognosis even w/ immediate treatment.Patient presents with sudden painless and marked unilateral loss of vision. Fundoscopy reveal pallor of the retina, arteriolar narrowing, separation of arterial flow (box carring) , retinal edema, and periofoveal atrophy (cherry red spot). Dx?

central retinal artery occlusion

Your 66 year old female presents with sudden, unilateral painless blurred vision or complete visual loss. Exam reveal an afferent pupilarry defect, optic disc swelling and a *blood and thunder* retina. Likely dx? central retinal artery or central retinal vein occlusion?

central retinal vein occlusion

vascular compromise

central vertigo; common cause of vertigo in the elderly; triggered by change in posture or neck extension; treated with aspirin or vasodilators

pure vertical, pure horizontal, or pure rotary nystagmus almost always represents

central vestibular dysfunction

what abx can be used to prophylax in epistaxis

cephalexin, augmentin, clinda, bactrim

granulomatous inflammation of meibomian gland. what dx?

chalazion

painless, red hard, contender swelling on eye lid. what dx

chalazion

otitis externa

cheesey, painful must visualize TM before gtt abx NO GTTS IN RUPTURED TM use ear wick

what do you see on the fundoscopic exam for central retinal artery occlusion

cherry red spot, arteries appear bloodless, embolic obstruction

This bacteria causes bacterial conjunctivitis and patients will usu. present with mucopurulent dc with a marked follicular response on the inner lids. Nontender preauricular adenopathy is noted

chlamhydia

otalgia, pruritus, brown/yellow discharge with strong odor. what dx?

cholesteatoma

pilocarpine

cholinergic agonist used to treat acute angle-closure glaucoma

pinguecula

chronic actinic exposure, repeated trauma, dry/windy conditions yellowish on sclera ADJACENT TO CORNEA, painless

dx of acute sinusitis

clinical CT if chronic and resistant to tx

schaffer's sign

clumping of pigment cells in the anterior vitreous; seen with retinal detachment

viral conjunctivitis

cobblestoning MCC = ADENOVIRUS self limiting watery d/c

thyroid nodule

cold (CA) not (not CA) order FNA. MCC = radiation

pyogenic granuloma

common skin growth; shiny red mass in the mouth; treatment may or may not be necessary

bacterial pharyngitis

commonly presents with a fever, sore throat, petechiae on pharynx, sandpaper-like rash, strawberry tongue & tonsillar hypertrophy/exudate/erythema; diagnosed with a rapid strep test & culture

viral pharyngitis

commonly presents with a sore throat with additional URI symptoms without tonsillar exudate; treated with supportive care & salt water gargles

Chornic OM may result in what kind of hearing loss?

conductive

pterygium

conjunctiva grows in a triangular shape onto the nasal side of the cornea; treatment often not necessary, artificial tears, topical NSAIDs or low dose steriods

eustachian tube

connects middle ear to nasopharynx

allergic conjunct tx

cool compresses topical vasoconstrictors or antihistamines topical mast cell stabilizer

viral conjunct tx

cool compresses vasoconsticture, antihistamine artificial tears

sx of bacterial conjunctivitis

copious purulent discharge, decreased visual acuity, discomfort, glued shut on awakening

Slit lamp examination of the eye reveals an epithelial defect but a clear cornea. What is the likely dx?

corneal abrasion

Your 21 year old patient is complaining of a feeling of a "foreign-body sensation" in their eye, but nothing is there. She wears contacts. What is the likely dx? (chalazion, corneal abrasion, hordeolum)

corneal abrasion

avoid patching if

corneal abrasion d/t organic material or soft contact lens

what tests can be used to diagnose strabismus

corneal light relfex aka hirschburg test and the cover-uncover exam

what is the treatment for optic neuritis

corticosteriods, especially if multiple sclerosis is suspected

Bell's palsy affects What cranial nerve?

cranial nerveVII, the facial paralysis conforms to the all branches of the peripheral nerve including the side of the face, eyelid and forehead muscles.

treatment for orbital cellulitis

ct of orbits, IV antibiotics, ophthalmology consult

treatment for corneal abrasion

cycloplegics - dilates pupils and relieves pain topical antibiotics- erythromycin pressure patch? - may help with comfort and photophobia cover for pseudomonas in soft contact lens wearers- tobramycin or flouroquinolone NO TOPICAL ANESTHETICS

corneal abrasion tx

cycloplegics to dilate pupil and relieve pain topical abx (erythromycin or tobramycin or fluoroquinolone f/u 24 hr and avoid contacts for 1 week

cholesteatoma

cyst in the middle of ear and skull bone, TM retracts due to cyst and tissue/skin fills in space

CMV retinitis

cytomegalovirus attacks the eye; less common when HIV is treated; causes retinal opacification, retinal hemorrhages; treated with valganciclovir, ganciclovir, foscarnet

esotropia

deviating eye turns inward

exotropia

deviating eye turns outward

What are the RFs for having a central retinal VEIN occlusion?

diabetes, hyperlipidemia, glaucoma, hyperviscosity states (polycythemia, leukemia)

What is the leading cause of blindeness in adults in the USA?

diabetic retinopahty

monospot

diagnostic test for infectious mononucleosis

What are the signs of hypertensive retinopathy?

diffuse ateriolar narrowing, copper or silver wiring, arteriovenous nicking (artherosclerosis)

MC cause of epistaxis

digital trauma

spread of bacterial conjunctivitis

direct contact--can infect other eye, fomites

dx of epiglottitis

direct visualization of epiglottis is dx but manipulation may initiate sudden fatal airway obstruction CBC, epiglottic cultures, after airway secure thumbprint sign on lateral neck radiograph

Critical: Optic disk

disc is yellowish-orange, margins are sharp cup to disc ratio is 1 to 2.

tx of meniere's ds

diuretics NaCl restriction

dx of vertigo

dix hallpike maneuver + with peripheral vertigo audiometry caloric stimulation CT/MRI

what test to dx BPPV

dix halpike

how to diagnose bppv

dix-hallpike maneuver

sx of cholesteatoma

dizzy, ear drainage, hearing loss, perforation

stensen duct

drains parotid glands

wharton duct

drains submandibular glands; most common duct for sialolithiasis

peritonsillar abscess

drooling, hot potato voice, deviatio of uvula toward abscess Tx: I&D, clindamycin

epiglottitis

drooling, tripod, thumb print sign on lateral x-ray

FB ear:

drown bug in lidocaine kids with chronic smelly otorrhea

what is the most common type of macular degeneration

dry (atrophic)

keratoconjuncivitis sicca

dry eyes; treated with artificial tears, restasis

If a chronic dacryocystitis, what other intervention may be warrented?

duct cannulation, dacryoplasty, stenting, other surgery

debrox

ear solution that softens wax; used to treat cerumen impaction; Generic Name: carbamide peroxide

The edge of the eyelid everts secondary to advanced age, trauma, infection or palsy of the facial nerve. Is this an entropion or an ectropion?

ectropion

Ectropion/entopion

ectropion- eye lid turned outward entopion- eye lid turned inward

What are the RFs for developing a central retinal artery occlusion?

emboli, thrombotic phenomenon and vasculitides

orbital cellulitis

emergency infection of tissue behind eye proptosis pain with movement decreased EOM

What is the treatment for retinal detachment?

emergency optho to discuss laser surgery or cryosurgery remain supine with head turned to the side of the retinal detachment

What is the trx for a central rentinal artery occlusion?

emergency optho. recumbent position an dgentle ocular massage. vessel dilation and paracentesis are attempted to save the eye. Work up and mgmt of artherosclerotic dz or arrythmias is warranted to reduce the risk of reccurence

menieres affects what part of ear

endolymphatic compartment of inner ear

thumbprint sign

enlarge epiglottitis seen on X-ray

double vision in blowout fracture is due to what

entrapment of inferior rectus muscle

The lid and lashes are turned in secondary to scar tissue or a spam of the orbicularis oculi muscles. Is this an entropion or an ectropion?

entropion

how to treat bppv

epley maneuver

cholesteatoma

erosion of ossicles from long term retraction of TM hearing loss requires surgery

corneal ulcer

erosion of the cornea; causes pain, photophobia, tearing, & reduced vision

corneal ulcer

erosion or sore of the outer layer of the cornea (usually from infection)

sx of viral conjunctivitis

erythema of conjuctiva, copious watery discharge, tender PREAURICULAR lymps

What is the tx for BACTERIAL laryngitis?

erythomycin, cefuroxime or augmentin oral or IM cortico steroids can hasten recovery for performers but req. vocal coard eval

glycosylation

excess sugar attaching to proteins such as the collagen of the blood vessels; leads to diabetic retinopathy

what predisposes a person to cataract development

excess sun

vertigo

experience of false sensation of movement

Acute/chronic otitis media/ externa

externa MC- Pseudomonas. otitis media= Step Pnuemonia***= infection of middle ear, temporal bones and mastoid air cells. MC preceded by URI** URI CAUSES EUSTACHIAN TUBE EDEMA- NEGATIVE PRESSURE- TRANSUDATION OF FLUID AND MUCOUS IN MIDDLE EAR, COLONIZATION BY BAC. -sx- tender tragus/oracle -tx- externa- Keep dry+ ciproflomacin (if TM perforated= ofloxacin)= immuno compromised (foul discharge)--- CT--- hospitalization+ antibiotics media- amox, augmentin

This is caused by the inflammation and infection of the glands of the Moll or Zeis, with pustular formation in those glands. They are situated immediately adjacent to the palpebral margin. Is this an internal or extrenal hordeola? (STYE)

external

treatment for cataracts

extraction and lens replacement

What is the tx for viral conjunctivitis?

eye lavage with normal saline twice a day for 7 to 14 days, vasoconstrictor-antihistamine drops warm to cool compresses ophthalmic sulfonamide drops

tx of viral conjuctivitis

eye lavage with normal saline x 2 days vasoconstrictor-antihistamine warm/cool compress sulfonamide drops to prevent bacteria

Pts with retinal detachment may complain of a "curtain" falling over their eye, or if smaller, they may complain of...?

flashes of light Floaters

corneal abrasion or ulcer

flourescein stain contacts? think PSEUDOMONAS Tx: tobramycin for abrasion ophth referral for ulcer

cotton wool spots

fluffy, gray-white spots from micro-infarction of the nerve fiber layer; seen with nonproliferative diabetic retinopathy

-floxacin

fluoroquinolones used to treat bacteria keratitis (levo/o/nor/cipro)

atrophic candidiasis

follows thrush or antibiotic use; causes oral mucosal burning & sensitivity to foods; treated with antifungals

vestibular neuronitis

follows viral infection n/v, vertigo DOES NOT AFFECT HEARING

80-90

for acute otitis media, amoxicillin is given at _______mg/kg/day BID x 10-14 days

removal of ear foreign body

forceps, irrigation but not if material is organic; 2% lidocaine if the insect is live

proptosis

forward protusion of the eye; occurs in 50% of pts with hyperthyroidism; treated with systemic steroids, radiotherapy or surgical decompression

retinal detachment vs amaurosis fugax

fugax: transient detach: continuous until surgery to fix it

how to diagnose retinal detachment

fundoscopic exam and ultrasound

how to diagnose central retinal artery occlusion

funduscopic exam is usually confirmatory; fluorescein angiography. once the diagnosis is made, a carotid doppler and echo should be done to locate embolic source

mucormycosis

fungi that invades the sinuses & may enter the CNS; seen in immunocompromised patients; treated with amphotericin B

laryngopharyneal reflux

gastroesophageal reflux into the larynx; causes chronic hoarseness/cough, throat discomfort, asthma

monocular vision loss is seen in:

giant cell arteritis retinal artery occlusion multiple sclerosis

what is a possible consequence of a hyphema

glaucoma by obstructing the trabecular meshwork

do not palpate eye if suspect

globe rupture

treatment of labyrinthitis

goes away on its own; meclizine, antihistamine

Macular degeneration

gradual CENTRAL vision loss DRY DRUSDEN causes: age, smoking

atrophic (dry) degeneration

gradual breakdown of the macula leading to gradual blurring of central vision; presents with drusen; treated with amsler grid & vitamins to slow progression

What is the chief clinical feature of macular degeneration?

gradual loss of central vision

central vertigo

gradual onset of "dizziness"; becomes progressively worse & debilitating; causes vertical nystagmus, gait ataxia & CNS symptoms

atrophic (dry) mac degen ssx

gradual progressive drusen (yellow deposits) retinal pigment atrophy

dx of bacterial conjunctivitis

gram stain

What diagnostic tests are indicated in bacterial conjunctivitis?

gram stain if rare bacteria is suspected, gram stain and giemsa stain

chronic sinusitis lasts how long

greater than 12 weeks

diptheria

grey pseudomembrane Tx: Antitoxin, erythromycin

Hair and scalp palpation

hair texture is normal, scalp has no scaling, lesions or areas of tenderness

What is the tx for bacterial conjunctivitis?

hand washing, application of topical antibx (gentamicin, tobramycin, ciprofloxacin, ofloxacin) rare pathogens may need systemic antibx (azithromycin, penicillin etc)

coxsackievirus

hand, foot & mouth disease seen in children

what are the findings of diabetic retinopathy on fundoscopic exam

hard exudates, micro aneurysms,

A 45-year-old male presents with purulent discharge from his right ear for three weeks. He states that despite being treated by his family doctor for an ear infection one month ago, the problem continues to get worse. Upon exam, you note purulent discharge in the ear canal, an erythemic tympanic membrane, and a possible perforation. What are the pathogens most likely to culture positive?

he clinical vignette describes a chronic otitis media. Usually, this refers to a complication of acute otits media with perforation. Pathogens that culture from these infections are usually pseudomonas, proteus, or staphylococcus aureus. Strep pneumoniae is often seen in acute otitis media. E.coli is a urinary tract pathogen. Candida albicans is a cause of vaginitis, and mycoplasma is a respiratory pathogen.

Menieres disease

hearing loss vertigo tinnitus aural fullness

hereditary hearing loss

hearing loss that is seen in children; 2/3 are nonsyndromic & 1/3 are syndromic

conductive hearing loss

hearing loss usually from disorders in the auricle, external auditory canal & middle ear; can be caused by obstruction, mass loading, stiffness effect or discontinuity; often correctable with medical or surgical therapy

sensorineural hearing loss

hearing loss usually from disorders in the inner era & central auditory pathways; many causes; diagnosed with whisper, weber & rinne tests; treated with hearing aids, cochlear implants, etc.

clinical features of labyrinthitis

hearing loss, dizziness, vertigo

signs and symtpms of cholesteatoma

hearing loss, painless drainage that is unremitting, and muscle paralysis

pulsatile tinnitus

hearing one's own heartbeat in ears; suspicious for vascular disease

hyphema

hemorrhage into the anterior chamber of the eye; acute eye pain, irregular pupil shape; treated with rest, elevate head, topical corticosteroids or atropine

oral herpes simplex

herpes cold sore; causes clusters of lesions on lips or vermillion border & adjacent skin; can also cause malaise, headache, fever, etc; treated with topical antiviral meds

herpes simplex keratitis

herpes infection of the cornea; diagnosed with dendritic ulcer; treated with simple debriment & patching

what is hutchinsons sign

herpes zoster on tip of nose = eye involvement

tx of cavernous sinus thrombosis

high dose IV ABs

sx of laryngitis

hoarseness--HALLMARK little to no pain with loss of voice

Patient presents with acute onset of pain and edema in the involved eyelid. There is a palpable indurated area in the involved eyelid which has a central area of purulence with surround erythema. What is the likely dx?

hordeolum (stye)

Diabetics or immunocompromised patients may develop malignant OE. What treatment will this require?

hospitalization and parenteral antibx

oral lichen planus

idiopathic cell-mediated autoimmune response seen in HIV patients; presents with a lacy leukoplakia lesion of oral mucosa; treated with local or systemic corticosteroids

meniere disease

idiopathic distention of endolymphatic compartment of the inner ear by excess fluid leading to increased pressure within the inner ear; presents with episodic (peripheral) vertigo lasting minutes to hours + fluctuating hearing loss, tinnitus & ear fullness; diagnosed with a dix-hallpike maneuver & transtympanic electrocochleography

acanthamoeba keratitis

important cause of keratitis in contact lens wearers; causes severe pain; treated with topical biguanides

prostaglandin analogs used

in chronic but not acute glaucoma

bone

in conductive hearing loss, the rinne test will present with _____ conduction greater

affected

in conductive hearing loss, the weber test will present with lateralization to the ______ ear

what is amblyopia

in older children with strabismus the brain may learn to ignore input from one eye and cause permanent decrease in vision

air

in sensorineural hearing loss, the rinne test will present with _____ conduction greater

normal

in sensorineural hearing loss, the weber test will present with lateralization to the ________ ear

barotrauma

inability to equalize the barometric stress exerted on the middle ear by air travel, rapid altitudinal change, or underwater diving; causes ear pain, clogged ears, tinnitus, hearing loss & ruptured TM; treated with swallow, yawn, pseudoephed & phenylephrine

trismus

inability to open the mouth fully; seen in ludwig angina & peritonsillar abscess

anterior ischemic optic neuropathy

inadequate perfusion of posterior ciliary arteries that supply anterior portion of optic nerve; can be caused by giant cell arteritis

papilledema

increased ICP ie: tumor, bleed Dx: increased pressure seen with LP

Papilledema

increased intracranial pressure

glaucoma

increased intraocular pressure (>21mmHg) with optic nerve damage (canal of schlemm blocked, decreased flow of aqueous humor)

what are the physical finddings for optic neuritis

indistinct optic disc border with raised ridge, visual acuity reduced, color vision altered, afferent pupillary defect, and optic nerve inflammation ( most inflammation cannot be seen because inflammation is retrobulbar)

postseptal cellulitis associated with what illness

infection from paranasal sinuses

cause of Tympanic membrane perforation (TMP)

infection most common -also blow to ear, severe atmospheric overpressure, exposure to excessive water pressure (eg, in scuba divers), and improper attempts at wax removal or ear cleaning.

Hordeolum

infection of external sabecous gland. Staph areus -sx- small painful nodule on eye lid -tx- warm compresses several times a day for 48 hours

dacryocystitis

infection of lacrimal sac; infants & >40y; presents with tenderness, edema & redness to the nasal side of the lower llid; treated with a warm compress, topical & systemic antibiotics

lymphadenitis

infection of lymph nodes; can be caused by TB, cat-scratch disease, sacroidosis; treated with antibiotics or surgical excision

mastoiditis

infection of the mastoid; caused by s pneumoniae, H influenzae & s pyogenes; causes postauricular pain, redness, & swelling; treat with cefazolin

Acute OM

infection of the middle ear (70-90% bacterial) kids more likely to get b/c shorter, more horizontal canal also increased with pacifer use or down syndrome

dental abscess

infection of the mouth, jaw or face; toothache in a tooth; treated with antibiotics, pain meds & dental block

peritonsillar abscess

infection of the peritonsillar space; causes severe sore throat, fever, "hot potato voice", unilateral tonsil hypertrophy & deviated uvula; treated with IV antibiotics, IV fluids, I&D, tonsillectomy

periodontitis

infective, inflammatory process of supporting structure presenting with gum recession & formation of "pockets" at tooth base/gumline; may lead to tooth loss & systemic disease

Dacryoadenitis/Dacryocystitis

inflamed nasolacrimal duct as a result of obstruction. Common pathogens are Staph aureus, Group B strep, Staph epidermidis and Candida. May present with pain, swelling, tenderness, redness, and prudent discharge Tx: warm compresses or ABX gentamicin, tobramycin, erythromycin for infection.

blepharitis

inflammatin of both eyelids--- DOWNS+ ECEZEMA -anterior- staph a./ seborrhic, posterior- meiobian gland -sx- eyelash flaking, red rimming of eyelids -tx- anterior- eyelid hygiene, warm compresses, eyelid scrubbing. posteior- eyelid massage

epiglottitis

inflammation & infection of epiglottis; presents with rapid onset sore throat, fever, odynophagia, respiratory distress

blepharitis

inflammation of BOTH eyelids - common in patients with downs syndrome and eczema - posterior: dysfunction of meibomian gland ( associated with rosacea and allergic dermatitis) - eye irritation/itching - eyelid changes: burning, erythema with crusting, scaling, red-rimming of eyelid and eyelash flaking - +/- entropion or ectropion (esp with posterior) anterior: eyelid hygience: warm compresses, eyelid scrubbing, baby shampoo, possible abx posterior: eyelid massage/expression of meibomian gland regularly

when is treatment of mastoiditis surgical?

inflammation of bone and connective tissue surrounding bobne

etiology of labyrinthitis

inflammation of inner ear that usually occurs after cold or flu

parotitis

inflammation of parotid gland; "mumps"; causes parotid tenderness & enlargement; diagnosed with IgG detection; treated with pain relievers

acute pharyngitis

inflammation of pharynx/tonsils; causes sore throat with additional URI systems; can be viral (rhinovirus/coronavirus) or bacterial (strep)

dacryoadenitis

inflammation of sac -sx- swelling of outportion of eye lid -tx- topical ocular and oral anti-staphylococcal antibiotics= clinda+ Cefalexin/cefaxlone. warm compresses, anagelsics defintiveremoval of sac= dacrosysetetomy

Acute Sinusitis

inflammation of sinuses usually following URI

gingivitis

inflammation of soft tissues surrounding teeth caused by the lack of proper oral hygiene with accumulation of plaque & calculus; treated with thorough, professional cleaning, repair, good dental health

vestibular neuritis

inflammation of teh vestibular portion of CN8 in the inner ear; causes peripheral vertigo- single attack of vertigo with no accompanying auditory symptoms; treated with meclizine & valium

conjunctivitis

inflammation of the conjunctiva; most common eye disease; red irritated eyes

Blepharitis is . . .

inflammation of the eyelid(s)

etiology of dacryoadenitis

inflammation of the lacrimal glands via viral or bacterial infection

A Chalazion is . . .

inflammation of the meibomian gland

medicamentosa rhinitis

inflammation of the nose caused by cocaine or afrin abuse

gustatory rhinitis

inflammation of the nose caused by eating spicy foods

vasomotor rhinitis

inflammation of the nose caused by irritants such as smoke or perfume

optic neuritis

inflammation of the optic nerve

optic neuritis

inflammation of the optic nerve; associated with MS, autoimmune disorders, etc.; causes unilateral vision loss that presents over a few days; treated with methylprednisone

dacryoadenitis

inflammation of the tear-producing gland (lacrimal gland)

glossitis

inflammation of the tongue; caused from nutritional deficiencies, drug reactions, dehydration, etc.; causes red, smooth-surfaced tongue

keratitis

inflammation/infection of the cornea; causes extreme pain; mostly due to bacteria, viruses & fungi; presents with pain, photophobia & reduced vision

what is retinitis pigmentosa

inherited autosomal dominant recessive or Xlinked night blindness in childhood

what is blepharitis

inlfmmation of the lid margin

tympanic membrane signs and symptoms

instant relief from pain associated with middle ear effusion

dx of OM

insufflation of TM to look for immobility clinical bulging TM

These are caused by the inflammation and infection of a meibomian gland, with pustular formation in that gland. They are situated deep from the palpebral margin. is this an internal or external hordeolum (STYE)

internal

types of hordeolum

internal --> inf of meibomian gland, rare external --> sty --inf of moll or zeis gland, edge of palpebral margin

ipratropium

intranasal anticholinergic used to treat viral rhinosinusitis

oxymetazoline

intranasal decongestant used to treat Eustachian tube dysfunction

chronic glaucoma

intraocular pressure is elevated due to reduced drainage of aqueous fluid through trabecular meshwork; presents with slow, progressive bilateral peripheral vision loss; treated with prostaglandin analogs or surgery

The initial treatment for menieres dz is a low sodium diet and diuretics (acetazolamide). What if the case is unresponsive?

intratympanic corticosteroid

how to differentiate papilledema and optic neuritis

isolated disk edema without elevated CSF pressure is not considered papilledema

allergic conjunctivitis

itch bilateral

how do you keep RBCs from staining the cornea in a hyphema?

keep HOB at 45 deg

ear wick

keeps EAC open so drops can get into ear; used to treat otitis externa

treatment for periorbital cellulitis

keflex

What is the most common location of epistaxis?

keissenbachs plexus

This disorder of the eye presents with redness and irritation, fever, sore throat, cough and *decreased visual acuity*. What is the likely dx? conjunctivitis, uveitis, keratitis

keratitis (note the decreased visual acuity is not usu. present in conjunctivitis)

When viral conjuncitvitis is transmitted in swimming pools what is it called?

kertaocojunctivitis

dysfunction of eustachian tube

kids have short tubes down syndrome is risk factor leads to chronic ear infections

MC site of epistaxis

kiesselbach's plexus

labyrinthitis vs menieres

lab: CONTINUOUS men: EPISODIC

peripheral vertigo causes

labyrinthitis, meniere's ds, positional vertigo, vestibular neuronitis, migrainous vertigo, obstructing anatomic abnormality

definititive tx for glaucoma

laser trabeculoplasty surgery

photocoagulation

laser treatment for retinal detachment, retinal vein occlusion, proliferative diabetic retinopathy & sickle-cell retinopathy

cranial nerves

lateral rectus- CN VI abducens superior oblique- CN IV trochlear

what are the results of a weber test for unilateral conductive loss

lateralizes to affected ear

what are the results of a weber test for unilateral sensorineural loss

lateralizes to the normal ear or side you hear better in

macular degneration

leading cause of irreversible CENTRAL visual loss

proliferative (neovascular) diabetic retinopathy

less common diabetic retinopathy, but severe vision loss; causes new, abnormal blood vessel growth & vitreous hemorrhages; treated with laser photocoagulation or intravitreal injection of VEGF inhibitors

entropion

lid margin rolls in toward the eye; treated with surgery or botox

ectropion

lid margin rolls outwards; treated with surgery

Blepharitis tx

lid scrub (baby shampoo) warm compresses oral tetracycline or topical erythromycin or bacitracin

What is the tx for blephoritis?

lid scrubs using diluted baby shampoo on cotton tipped sswabs. Massage to express meibomina glands. Topical antibx if infection is suspected.

what is the treatment for blepharitis

lid scrubs with diluted baby shampoo with cotton tipped swabs. use topical antibiotics if infection is suspected

epiglottitis

life-threatening infection of epiglottis

Lips

lips are pink and moist without ulcers or cracking

homonymous hemianopia

loss half of vision field in each eye, associated with central vertigo

corneal abrasion

loss of superficial epithelium of cornea; treated with topical antibiotics, topic antiinflammatories & oral analgesics

In menieres dz, with caloric testing, nystagmus is gained or lost on the impaired side?

lost

menieres dz tx

low salt diet anti-vert (dimenhydrinate, meclizine, diazepam, or scopolamine patch) diuretics (HCTZ)

treatment for meniere's disease

low salt diet, diuretics, steroid or gentamycin injection

What is the treatment for menieres dz initially (non-pharm)?

low sodium diet and diuretics (acetazolamide)

conjunctival foreign bodies

low velocity projectile into the eye

What are the usu. bacterial causes of laryngitis?

m. catarrhalis and h.ifluenzae

Cholorquine or phenothiazine are drugs that can cause this disorder of the eye.

macular degeneration

what is leading cause of blindness in elderly

macular degeneration

causes of papilledema

malignant HTN hemorrhagic strokes acute subdural hematoma psuedotumor cerebri

In diabetic or immunocompromised patient who present with auricle pain on movement, and erythematous, swollen ear canal suspect this condition which is a necrotizing infection extending to the blood vessels, bone and cartilage. This requires hopsitalization and parenteral antibx.

malignant OE

mallet finger/ boutonniere deformity

mallet finger- avulsion of extensor tendon -tx- splint DIP uninterrupted extension X 6 weeks. boutonniere- sharp force against tip of paritally extended digit- hyperflexion of PIP, extend DIP -tx- splint PIP in extension X 4-6 weeks.

Mc complication of AOM

mastoiditis

postauricular pain and erythema, fever, bulging tm, what dx?

mastoiditis

MC sinus affected in acute sinusitis

maxillary (1. maxillary, 2. ethmoid, 3. frontal, 4. sphenoid)

otitis externa "swimmers ear"

mechanical obstruction + infectious agent (pseudomonas, proteus)

tx of labyrinthitis

meclizine (dizzyness) promethazine (motion sickness) dimenhydrinate (motion sickness) usually resolves in a few weeks

What is the treatment for chronic OM?

medical - remove infected debris, avoid water exposure and topical antibx drops definitive treatment will include surgery and TM repair/reconstruction

internal hordeolum

meibomian gland abscess that usually points to the conjunctival side of the eyelid

PAtient presents with recurrent vertigo (lasting minutes to hours), with lower range hearing loss, tinnitus and one sided aural pressure. Likely dx?

menieres dz

This condition is also known as endolympthatic hydrops

menieres dz

The phenomenon of wavy or distorted vision and can be measured with an amsler grid.

metamorphopsia

A 34-year-old woman presents to the clinic with complaints of intermittent flushing and blushing that started 3 to 4 weeks ago. Since then, she has noticed several inflammatory papules on the cheeks, nose, and chin. Upon exam, you notice an overall rosy hue to the face and the absence of any comedones. Which of the following would be the best course of topical therapy at this time?

metronidazole gel Metronidazole is the topical treatment of choice for rosacea, which is consistent with the clinical findings in this 34-year-old female patient. Mupirocin ointment is a treatment option for impetigo, whereas permethrin is indicated for scabies. Tretinoin is effective and is indicated for comedonal acne. Topical hydrocortisone has not been shown to be effective for rosacea.

tzanck smear

microscopic exam of vesicle fluid or base of lesion for virally infected cells; diagnosis for oral herpes simplex

non-proliferative diabetic retinopathy

mild, moderate or severe diabetic retinopathy; causes microaneurysms, retinal hemorrhages, venous beading, retinal edema, & cotton-wool spots; treated wtih fluorescein angiography or vitrectomy

tx of hyphema

mild: blood will absorb acetominophen for pain (NSAIDS cause rebleeding) topical cycloplegics patch

strabismus

misalignment of the eyes; diagnosed with a corneal light reflex & cover test; need early treatment to prevent amblyopia

corneal foreign bodies

moderate to high velocity projectile into the eye

staph aureus

most common bacteria that causes chronic sinusitis

Hib (H influenza B)

most common cause of epiglottitis

laryngitis

most common cause of hoarseness; usually viral; can be treated with corticosteroids

allergies

most common cause of rhinitis

presbyacusis

most common cause of sensorineural hearing loss

adenovirus

most common cause of viral conjunctivitis

benign paroxysmal positional vertigo

most common form of vertigo; onset of peripheral vertigo a few secs after assumption of a certain head position; diagnosed with a dix-hallpike maneuver

adenoid cystic carcinoma

most common malignant tumor of the mouth; treated with excision of gland or fine needle aspiration biopsy

maxillary

most common sinus site for an acute sinusitis; presents with cheek pain/pressure that may radiate to the upper incisors

rhegmatogenous

most common type of retinal detachment; retinal inner layer detaches from the choroid plexus

tx of TMP

most heal on their own within a few weeks paper-patch method, fat-plug tympanoplasty, a Gelfoam plug avoid some AB eardrops as they can be ototoxic

what is positive dix halpike test

move patient from sitting to supine with head turned 45deg and tilted back 20 deg and you lay them down and they have nystagmus

optic neuritis MC etiology

multiple sclerosis

MCC of optic neuritis

multiple sclerosis other causes: syphillis, lyme, herpes, lupus, IBD, vasculitis, DM, B12 deficiency

samter triad

nasal polyps, asthma & aspirin sensitivity

phenylephrine

nasal spray decongestant used to treat barotrauma

myopia

near-sightedness, close objects are clear, far away objects are blurry; corrected with minus lenses

cortisporin otic suspension

neomycin/polytrim B/hydrocortisone otic; antibiotic used to treat otitis externa; 3 drops TID-QID x 7-10d; NOT used if TM perforated

gonorrhea

neonatal conjunctivitis in days 2-5 of life is mostly due to __________

chlamydia

neonatal conjunctivitis in days 5-7 of life is mostly due to _________

HSV

neonatal conjunctivitis in days 7-11 of life is mostly due to __________

silver nitrate

neonatal conjunctivitis in the first day of life is mostly due to _____________

neovascular (wet) degeneration

new, abnormal vessels grow leading to accumulation of serous fluid, hemorrhage, & fibrosis; loss of central vision; treated with anti-angiogenics

Anterior chamber

no crescent shadow noted

Cranial nerve 1

no deficit of the olfactory nerve

Critical: Tongue

no discoloration or lesions of the tongue

What is the treatment for macular degeneration?

no effective treatment, if detected early laser therapy or intravitreal injections of monoclonal antibody drugs may slow progression

Critical: Fundus

no hemorrhages or exudates

conjunctiva, sclera and cornea

no injection, icterus, lesions, edema or foreign bodies, cornea is clear

results of a normal weber test

no lateralization of sound at all

Gingiva, teeth, breath odor

no lesions, swelling or discoloration of the gingiva, teeth are in good repair, non-tender with good oral hygiene and no abnormal breath odor

Neck Inspection

no lesions, swelling, or discoloration

Paranasal sinuses

no maxillary or frontal sinus tenderness

Cranial Nerve 5 sensory

no sensory deficit of cranial nerve 5

Lacrimal ducts

no swelling or regurgitation

External ear canal

no swelling, redness or obstruction

Auditory Acuity

no unilateral deficits

barotrauma to ear

normally the middle ear has same pressure as outside, if different =trauma the problem often occurs with altitude changes, such as flying, scuba diving, or driving in the mountains. If you have a congested nose from allergies, colds, or an upper respiratory infection, you are more likely to develop barotrauma.

Nose inspection

nose is midline with out lesions

micropsia

object seen by the affected eye looks smaller than in the unaffected eye; seen with macular degeneration

thyroglossal duct cyst

occur along embryologic course of thyroid's descent from tongue to lower neck; most common before age 20; treated with surgical excision

fungal keratitis

occurs after corneal injury involving plant material or agricultural setting; diagnosed with scrapings of the site; treated with topical antifungals

Auricular hematoma

occurs after direct trauma to the ear, typically during sports and can result in cauliflower ear

acute angle-closure glaucoma

occurs when pt has a preexisting narrow anterior chamber angle; can be caused by pupillary dilation (darkened room), stress, or from certain medications; causes red eye with cloudy cornea, halows around lights & peripheral loss of vision; treated with acetazolamide

astigmatism

occurs when the refractive error in the horizontal & vertical axes are different

Elevated IOP without optic disc damage is known as...

ocular hypertension

sx of Peritonsillar abscess

one sided neck/throat pain fever HA dysphagia change in voice odynophagia deviation of uvula MUFFLED VOICE tracheal "rock sign"

when should you patch for a corneal abrasion

only for large abrasions measured 5-10 mm to promote healing

cataract

opacity of crystalline lens; #1 cause of blindness worldwide; can be caused by congenital, traumatic, systemic disease or age-related/senile; causes blurred vision & halos around lights; treated with surgical removal or intraocular lenses

What type of glaucoma is most common?

open angle affects ppl > 40 and african americans

what nerve is herpes zoster ophthalmicus involves

ophthalmic branch of the trigeminal nerve

what do you see on physical examination for open angle glaucoma

optic cupping and elevated intraocular pressure and loss of peripheral vsion

papilledema

optic disk swelling commonly from hypertensive retinopathy; sign of hypertensive emergency

herpes zoster tx

oral acyclovir - topical steroids if posterior uveitis present

oropharyngeal mucosa

oral mucosa is pink and without ulcers or other lesions

acyclovir/valcyclovir

oral or topical antivirals used to treat herpes simplex keratitis & herpes zoster ophthalmicus

acute/chronic sinusitis tx

oral/nasal decongestants Abx: amoxi/clav. (acute: 10-14 days. chronic: 2-3wks) 2nd line: levofloxacin, clindamycin

These are serious complications of sinusiits.

orbital cellulits, osteomyelitis or cavernous sinus thrombosis

blowout fracture

orbital floor fx to orbital rim blunt trauma inferior rectus muscle entrapment enophthalmos

Inflammation of the area near the __________ complex is an important component differentiating sinusist sfrom allergic or viral rhinitis.

osteomeatal

malignant otitis externa

osteomyelitis at skull base d/t psuedomonoas (DM and immunocompromised pts) - IV abx (cipro)

malignant otitis externa

osteomyelitis at the skull base secondary to pseudomonas infection; seen in diabetic or immunocompromised; causes persistent greenish, foul-smelling discharge, deep ear pain, mastoid tenderness; treated with IV cipro

otitis externa tx

otic neomycin, polymyxin if fungal: amphotericin B

Ramsay Hunt-

otitis externa caused by herpes zoster

serous otitis media

otitis media with effusion; more common in children; causes dull & hypomobile TM & air bubbles; treated with prednisone or amoxicillin

ruptured globe

outer membranes of the eye are disrupted due to penetrating trauma to the eye; presents with misshaped eye, enophthalmos, severe conjunctival hemorrhage, irregular shaped pupl & positive seidel's test; treated with eye shield & consult ophthalmology

cerumen impaction

overproduction of cerumen; causes feeling of fullness, hearing loss & pain/discharge; treated with irrigation, ear curettes & hydrogen peroxide

sx of optic neuritis

pain on eye movement sudden loss of vision (usually UNILATERAL) blurry vision change in color persception

sx of otitis externa

pain, tenderness with tragus, edematous canal, purulent debris

clinical features of angle-closure glaucoma

painful eye loss with injection, steamy cornea, and fixed mid dilated pupil. decreased visual acuity, nausea, and vomiting. usually unilateral

etiology of a hordeolum

painful inflammation of the gland or hair follicles of the eyelids typically due to staph aureus

how does dacryoadenitis present?

painful swelling in outer portion of upper eyelid plus preauricular lymphadenopathy

clinical features of macular degeneration

painless progressive central vision loss and color loss

thyroid CA

papillary = MC, best prognosis anaplastic = rare, most deadly medulary thyroid CA in MEN 2A, 2B

mumps

paramyxovirus parotid gland self limited orchitis

Parotid glands

parotid glands without nodules or irregularity

seidel's test

parting of the fluorescein dye by a clear stream of aqueous humor from the anterior chamber

tripod position

patient leaning forward on hand to ease breathing; seen with epiglottitis

intraocular foreign bodies

penetrating eye injury usually involving hammering on metal equipment, metal grinding, or other ocular trauma; dx w/ CT

tinnitus

perception of sound in the absence of actual sound; "ringing in the ears"; causes pulsatile or tonal tinnitus; treated with antidepressants, surgery, masking efforts

What are the main finding of CHRONIC OM?

perforated TM and chronic ear discharge with our w/o pain. May result in condunctive hearing loss

labyrinthitis

peripheral cause of vertigo associated with ear infection

Meniere syndrome, labyrinthitis, vestibular neuronitis, and perilymphatic fistula are causes of?

peripheral vertigo.

movement-related vertigo

peripheral vertigo; onset of vertigo with almost any position change

20 year old female complains of a cold, sinus pain and difficult breathing. She's gained some weight recently. You look in your mouth and you see deviation of the soft palate and assymetric rise of the uvula...likely dx?

peritonsillar abscess deviation of the soft palate and assymetric rise of uvula highly suggestive

Trismus

peritonsillar abscess ludwig's angina dental abscess tetanus

Patient presents with a significant sore throat, pain with swallowing, trismus, deviation of the soft palate and uvula and muffled "hot potato voice". What is the diagnosis?

peritonsollar abscess

Sore throat, a common reason for outpt visits, is assoc w/ about half of outpatient anitbiotic use.

pharyngitis

abnormal growth of tissue only present over the sclera

pinguecula

how to dx orbital fracture

plain films or CT

foreign body tx after removal

polymyxin-bacitracin or erythromycin ointment

tarsal tunnel syndrome

post tibial nerve compression from overuse, restrictive footwear -sx- pain/numbness @ medial mallelous and sole, tarsal tunnel syndrome increases pain through out day (plantar facititis- decreases throughout day) -dx- tinels sign

oral leukoplakia

precancerous hyperkeratosis due to chronic irritation ( tobacco, ETOH) -sx- white patchy lesions that cannot be rubbed off, painless -tx- cryotherapy

leukoplakia

precancerous hyperkeratosis due to chronic irritation (tobacco, smoking, ETOH, dentures); white patchy lesion that cannot be rubbed off, painless; treated with cryotherapy & laser ablation

erythroplakia

precancerous lesions similar to leukoplakia but with erythema

sensorineural hearing loss

presbycusis acoustic neuroma syphillis lyme dz

meniere's ds

pressure in fluid tubes too high

diabetic retinopathy

progressive damage to the retinal blood vessels from glycosylation; non-proliferative or proliferative

Your diabetic pt has neovascularization and vitreous humor on fundo exam. Is this a sign of non proliferative or proliferative diabetic retinopathy?

proliferative

what do you use to eval foreign body with a fluoroscein stain

proparacaine

-posts

prostaglandin analogs used to treat chronic glaucoma (bimato/latano/taflu/travo-)

late SCC

protruding masses with central necrosis, ulceration in the mouth

otitis externa clinical features

pruritis, pain, tenderness to pinna/tragus

causes of corneal abrasion

pseudomonas acanthamoeba keratitis (contact users) herpes keratitis (dendritic ulcer) foreign body scratches allergies bells palsy (eye open) autoimmune ds (RA)

step pneumo, h. influenzae, moraxella catarrhalis, strep pyogenes These are the most common bugs in acute OM. What are the most common bugs in chronic OM?

pseudomonas aeruginosa, s. aureus, proteus, anaerobes

What bugs most commonly cuase OE?

pseudomonas, proteus and fungi

common otitis externa orgs

pseudomonas, staph, proteus, fungi (aspergillus)

chronic OM orgs

psuedomonas, staph aureus

abnormal growth of tissue encroaching on the cornea

pteerygium

Oral Anti-histamines should be avoided in what population?

pts with Urinary retention Anti-histamines are anti-cholinergics

types of tinnitus

pulsatile and non-pulsatile (vascular cause)

what 3 triggers in acute glaucoma

pupillary dilation pharmacologic mydriasis anticholinergic meds

Iris and Pupils, RAPD, and Position and alignment

pupils are equal, round, and reactive to light, no RAPD, no tropia

bacterial conjunctivitis

purulent d/c staph PIC organisms strep pneumo, H. flu, M. cat Tx: sulfacetamide, erythromycin, cipro, ofloxacin, tobramycin, polytrim rule out gonorrhoea

clinical features of bacterial conjunctivitis

purulent discharge from both eyes. eyes glued shut on awakening

exudative (wet) mac degen ssx

rapid onset hemorrhages neovascularization

dx of pharyngitis

rapid strep test antigen assay, strep culture

Patient presents w/ a corneal abrasion. What should you do before examining or treating?

record visual acuity

Red reflex and clarity

red reflex is present, cornea, anterior chamber, lens and vitreous are clear

clinical features of blepharitis

red rims, dandruff like depositis or scales near eyelashes; conjunctiva is clear

cherry red macula

red spot seen with retinal artery occlusion

marcus gunn pupil

relative afferent pupillary defect; when bright lights shone from the unaffected eye to the affected eye, the pupils appear to dilate rather than constrict; seen with optic neuritis

laryngectomy

removal of part of the larynx; used to treat advanced cases of squamous cell carcinoma of larynx

sialoendoscopy

removal of salivary gland to treat chronic cases of sialolithiasis

metal foreign body tx

remove within 24 hr

treatment for nystagmus

removing inciting etiology, botox, or surgery

macula

responsible for central vision (detail & color vision)

cochlea

responsible for hearing; converts wave impulses from the middle ear into auditory nerve impulses

dental caries

result of mineral dissolution of tooth by bacterial products; most common cause of tooth loss before 35

Your 45 year old patient presents with acute onset of painless blurred or blackened vision that occurs over several mintues to hours and progresses to complete or partial MONOCULAR blindness. Likely dx?

retinal deatchment

"A curtain being drawn over the eye from the top to bottom" is the classic description of...

retinal detachment

In this condition the patient may sense "floaters" or flashing lights at the initiation of symptoms. Intraocular pressure is normal or reduced.

retinal detachment

_____________ is cuased by the overzealous use of decongestant drops or sprays containing oxymetazoline or phenyephrine. This cuases a rebound congestion which prompts increased use of the agent, creating a vicious cycle.

rhinits medicamentosa

typical orgs in viral acute sinusitis

rhinovirus, parainfluenza, influenza, RSV

tinnitus

ringing 90% associated with sensorineural hearing loss causes: ASPIRIN LOOP DIURETICS aminoglycosides CCB

test for conductive hearing loss

rinne test

what two viruses can cause congenital cataracts

rubella, cmv

what do you see on the fundus exam for retinal detachment?

rugae or orange peel appearance to retinal surface

What pathogen usu. causes a hordeolum

s. aureus

treatment for viral conjunctivitis

saline eye lavage twice a day for 7-14 days. warm compress to reduce discomfort

tx of acute sinusitis

saline nasal spray decongestants steam AB (amoxicillin)

causes of blepharitis

seborrhea, staph, strep, dysfuncton of meibomian glands (sebaceous gland)

what are the causes of blepharitis

seborrhea, staph/strep, or dysfunction of the meibomian blands. Seborrheic blepharitis is commonly associated with seborrhea of the face, eyebrows, external ears, and scalp

Though bacterial conjunctivitis is usu. self limiting, this serious infection can develop.

secondary keratitis

etiology of chalazion

secondary to chronic inflammation of the meibomian gland

tx of epiglottitis

secure airway broad spectrum ABs (cephlasporin 7-10 days)

which generation antihistamines are non sedating? sedating?

sedating: gen 2 non sedating: gen 1

vertigo

sensation of movement (spinning, tumbling, falling) or sensation of objects spinning around him/her

retinal detachment

separation of retina from pigmented epithelial (appears to flap in vitreous humor)

sx of labyrinthitis

severe vertigo, hearling loss, tinnitus, N/V

orbital cellulitus

sinus infection through ethmoid bone

woodruff plexus

site where posterior nose bleeds come from

FB in eye

slit lamp always irrigate

pterygium

slow thickening of bulbar conjuctiva CAN COVER CORNEA--comes from nasal side

vocal fold nodules

smooth, paired lesions that form at the junction of anterior 1/3 & posterior 2/3 of vocal folds

brachial cleft cyst

soft cystic mass along SCM muscle; treated with complete excision

If pressure doesn't stop the posterior nosebleed what is aonther option?

specialist eval, surgery packing is dangerous

treatment for tympanic membrane perforation

spontaneous healing within a few days; if not then surgical repair called tympanoplasty; antibiotic drops if infected

retinal detachment

spontaneous tear in peripheral retina due to degenerative changes; causes "curtain being pulled UP over eye"; diagnosed with retina hanging in vitreous & positive schaffer's sign

treatment for foreign body of eye

stain with fluorescein, observe with woods lamp, and remove with irrigagion or cotton tipped applicator.

most common cause of hordeolum

staph aureus

chronic sinusitis common orgs

staph aureus, anaerobes, h flu, fungi

Ophthalmoscope exam

stay steady! 2 deep breaths!

tx of retinal detachment

stay supine with head turned toward side with retinal detachment ER ophthalmologist laser surgery 80% recover 5% blind

What are the most common offending agents in otitis media?

step pneumo, h. influenzae, moraxella catarrhalis, strep pyogenes

optic neuritis tx

steroids

metamorphopsia

straight lines appear bent; seen with macular degeneration

Cranial nerve 11

strength is 5 out of 5 and symmetric

Patient presents with a fever over 38C, tender anterior cervical adenopathy, LACK OF COUGH and pharngotonsillar exudate. What is the likely dx?

strep GABHS

Mc AOM org

strep pneumo

otomastoiditis

strep pneumo Dx: CT air cells surgical debridement can reach brain protrusion of ear (asymmetrical ears)

bacterial conjunctivitis common orgs

strep pneumo staph aureus H flu

all AOM orgs

strep pneumo, H flu, M catarrhalis, or viral

typical orgs in bacterial acute sinusitis

strep pneumo, h flu, m. catarrhalis

what are the usual bacterial offenders for sinusitis

strep pneumo, h. flu, and m. cat

bacterial conjunctivitis

strep pneumo, s. aureus, H. aegyptius, moraxella, rare = G/C

What are the common pathogens that cause bacterial conjunctivitis?

strep pneumo, staph aureus, haemophilus aegyptius and moraxella

what are the common pathogens for otitis media

strep pnumo, m cat, h. flu

sx of epiglottitis

sudden high fever respiratory distress severe dysphagia drooling muffled voice mild stridor sit upright with neck distended

How would a retinal vein occlusion present?

sudden loss or blurring of vision in one eye Think Central retinal vein occlusion (CRVO)

peripheral vertigo

sudden onset of "dizziness" with nausea/vomiting; causes horizontal nystagmus, tinnitus & hearing loss

sx of Retinal artery occlusion

sudden, painless, unilateral vision loss, pale retina, afferent pupil defect, arteriolar narrowing retinal edema, separation of a. flow (box-carring), CHERRY red spot

sx of Retinal vein occlusion

sudden, unilateral, painless, blurred vision or visual loss "blood and thunder" retina (dilated v, hemorrhages, edema, exudates)

tx of laryngitis

supportive don't overuse voice to prevent formation of vocal nodules

TM perforation

supportive tx

What is the treatment for VIRAL laryngitis?

supportive tx - vocal rest and avoidance of singing or shouting

What is the recommended tx for viral sinusitis?

supportive, NSAIDs for pain, saline washes, steam and oral and or nasal decongestants (mucinex). intranasal corticosteroids can be helpful (fluticasone)

ectropion tx

surgery

entropion tx

surgery

vitrectomy

surgery involving removing vitreous fluid; treatment for non-proliferative diabetic retinopathy

tx of cholesteatoma

surgery to remove cyst

trabeculoplasty

surgery/laser done to treat chronic glaucoma

papilledema stems from

swelling of the optic disk due to increased intracranial pressure

tympanic membrane perforation

sx-Hx of closed AOM, head injury or direct ear trauma Hearing loss ≥40 dB serious middle ear injury. Hearing loss <40 dB w/o CSF, vestibular symptoms (eg, nystagmus, vertigo), or facial nerve injury suggests TM Perf w/o middle ear injury, middle ear fluid, or impaired baseline hearing. -dx- Physical Exam. CT if: ●Basilar skull fracture suspected ●Trauma & facial nerve dysfunction ●Blunt / penetrating middle ear trauma w/ hearing loss at ≥40 dB or with vestibular symptoms

hypertensive retinopathy

systemic HTN affect both retinal & choroidal circulation; most noticeable in young patients with abrupt elevations in BP; marker for current & future end-organ damage; causes arterial narrowing & AV nicking

pseudoephedrine

systemic decongestant used to treat Eustachian tube dysfunction, barotrauma, & viral/bacterial rhinosinusitis

amaurosis fugax

temporary monocular vision loss with complete recovery; feeling of "curtain being pulled down over eye"; caused by retinal emboli or ischemia

dix-hallpike maneuver

test to diagnose BPPV & meniere's disease

Children with allergic rhinits may develop a horizontal nasal crease called __________________ from habitually rubbing the nose.

the allergic salute

viral rhinosinusitis

the common cold; caused by the rhinovirus or adenovirus

emmetropia

the normal state of vision

Critical: Bimanual exam

there are no stones, masses or areas of induration

Middle ear

there is no visible evidence of fluid in the middle ear

Your patient presents with a fever, pharyngotonsillar exudate, lack of cough and anterior cervical adenopathy. You are soooo sure its strep! Rapid strep test comes back negative. What test can you do now to confirm it?

throat culture

central retinal vein occlusion

thrombus causes fluid backup in retina; presents with acute, sudden monocular vision loss; diagnosed with extensive retinal hemorrhages, retinal vein dilation & macular edema; no specific treatments

central retinal artery occlusion

thrombus or embolus blocks artery causing acute, sudden monocular vision loss; diagnosed with cherry red macula; treated with acetazolamide, lie on back, massage orbit & vessel dilation

oral candidiasis

thrush; common fungal disease caused by candida albicans; causes cottage cheese-like plaque readily scraped off to reveal erythematous base which may bleed; treated with antifungals

Critical: Thyroid

thyroid is normal size, shape, and consistency without nodules or tenderness

signs and symptoms f meniere's disease

tinnitus, hearign loss, vertigo that lasts for several hours, and nausea

Tonsils and uvula, Cranial nerve 9 and 10

tonsils are grade zero and equal with midline uvula, no exudate or lesions, uvula and soft palate rise symmetrically, gag reflex active

tx of bacterial conjunctivitis

topical AB, sys AB for rare pathogen

Tx of closed angle glaucoma

topical B blocker (timolol) alpha agonist (apraclonidine) carbonic anhydrase inhibitor (-amide) osmotic diuresis IV acetazolamide, IV mannitol --to reduce intraocular pressure laser or surgical iridotomy AVOID decongestants and anticholinergic medications

tx of corneal abrasion

topical anesthetic (not Rx) acetominophen for pain AB ointment (polymixin/bacitracin) FU in 1-2 days, patching no longer than 24 hours

diagnosis for corneal abrasion

topical anesthetic to confirm. slit lamp exam or fluorescein staining will reveal epithelial defect but a clear cornea

What is the treatment for a corneal abrasion?

topical anesthetic will provice immediate relief, but will retard healing. -saline irrigation - antibiotic ointment (gentamicin) patch for no longer than 24hrs daily f/u

ketorolac tromethamine

topical anti-inflammatory used to treat corneal abrasions

treatment for bacterial conjunctivitis

topical antibiotics

optivar

topical antihistamine used to treat allergic conjunctivitis

open angle glaucoma tx

topical beta blocker (timolol or betaxolol) prostaglandin analogs carbonic anhydrase inhibitors (acetazolamide)

If pressure and position change doesn't stop a nose bleed (anterior), what are options?

topical cocaine (anesthetic and vasoconstrictor), oxymetazoline and topical anesthtetics (lidocaine) rhinorocket

flunisolide

topical intranasal corticosteroids used to treat bacterial & viral rhinosinusitis

patanol

topical mast cell stabilizer used to treat allergic conjunctivitis

Avoid this treatment in a corneal ulcer because they will cause further tissue loss and increase risk of perforation

topical steroids

acute iritis tx

topical steroids, analgesics, refer to ophth.

What is the tx for Apthous ulcers (canker sores, ulcerative stomatitis)?

topical therapy such as corticosteroids can provide symptomatic relief 1 week oral prednisone taper can be helpful cimetidine can be used as maintenance tx

biguanides

topical treatment for acanthamoeba keratitis

transient vs sudden vs gradual vision loss

transient: TIA, emboli ( amoursosis fugax), or temporal arteritis sudden: central retinal vein or branch vein occlusion, optic neuritis, papillitis, and retrobulbar neuritis gradual: macular dengeration, tumors, cataracts, glaucoma

necrotizing ulcerative gingivitis

trench mouth; common in young adults during stress; causes painful, bleeding & necrosis gums; treated with 1/2 strength peroxide rinse

true or false: strabismus is normal in newborns

true

vocal fold cysts

true cysts or pseudocysts on the vocal folds, may fluctuate in size

tx of vertigo

tx cause

sialolithiasis & sialadenitis

tx: cephalosporin, increased hydration, sialogogues can be caused by dehydration- think elders stenson's duct (top) wharton's duct (bottom)

cortical cataract seen in what dz

type 1 dm

viral conjunctivitis

type of conjunctivitis; May be part of a viral prodrome followed by adenopathy, fever, pharyngitis, & URI; treat underlying URI/ virus symptoms

allergic conjunctivitis

type of conjunctivitis; bilateral redness, itching, watery & chemosis; treated with antihistamines & topical/systemic corticosteroids

chemical conjunctivitis

type of conjunctivitis; hx of chemical exposure to eyes; pain, redness, tearing, photophobia; tx by removing foreign matter, irrigation of eyes, topical antibiotics & pain relief

bacterial conjunctivitis

type of conjunctivitis; most commonly caused by strep/staph; presents with purulent discharge & lid crusting, usually unilateral; treated with topical erythromycin

exudative

type of retinal detachment; fluid accumulates beneath the retina leading to detachment

posterior uveitis

type of uveitis; causes gradual loss of vision with no pain; treated with systemic corticosteroids

anterior uveitis

type of uveitis; causes unilateral ocular pain, redness, photophobia & excessive tearing; treated with topical corticosteroids & mydriatics

labyrinthitis tx

typically self limiting, but may use diazepam/meclizine/or dimenhydrinate

is wet macular degeneration unilateral or bilateral?

unilateral

vocal fold polyps

unilateral masses that form within vocal fold

clinical features of viral conjunctivitis

unilateral or bilateral erythema of conjunctiva

Rheumatic fever

untreated strep throat (GAS)

avoid oral antihistamines in pts with:

urinary retention

weber test

used to diagnose hearing loss; practioner holds tuning fork on top of patient's head, sound should be equal in both ears

rinne test

used to diagnose hearing loss; practitioner holds tuning fork on mastoid bone, then moves to in front of ear once sound diminishes, air conduction should be greater than bone conduction

whisper test

used to diagnose hearing loss; practitioner stands behind patient, asks to cover 1 ear, whispers 3 words & asks patient to repeat them

dental block

used to provide pain relief for 12 hrs from a dental abscess

mastoiditis

usually from OM that extends to mastoid bone

corneal abrasion

usually from minor trauma (fingernail, contact, eyelash, small foreign body)

what is the treatment for hyphema

usually resolves spontaneously. AVOID NSAIDS because of rebleeding of that vessel

Retinal vein occlusion

usually secondary to thrombotic event

etiology of mastoiditis

usually untreated bacterial otitis media

acute pharyngitis

usually viral, or s. pyogenes

MCC of viral otitis externa:

varicella zoster

Rhinorrhea cuased by increased secretion of mucus from the nasal mucosa. It may be precipitated by changes in temperature or humidity, odors or alcohol or result from a neurovascular imbalance.

vasomotor rhinitis

vertebral basilar insufficiency:

vertigo + syncope

labyrnthitis

vestibular neuritis + hearing loss/tinnitus; presents with acute onset of continuous, severe peripheral vertigo lasting several days to a week; treated with antibiotics, meclizine, valium

tx of pharyngitis

viral = supportive bacterial = PCN tx done to avoid rheumatic fever and post-strep glomerulonephritis

tx of dacryoadenitis

viral: self limiting bacterial: cephlasporin chronic: tx underlying systemic condition

clinical features of optic neuritis

vision loss and mild eye pain

tx of of optic neuritis

vision usually spontaneously returns high dose steroids may speed return

dx of corneal abrasion

visual acuity slit lamp fluorescein staining

dx of hyphema

visual acuity tonometry (measure pressure) b scan ocular US

Visual fields

visual fields are full and equal by confrontation

These supplments can reduce the progression of macular degeneration.

vitamins, antioxidants, zinc, copper and omega 3 fatty acids

How do you resolve a CRVO?

w vision typically is resolved w/ time at least partially. a work up for further thrombosis is indicated. neovascularization can be treated with intravitreal injection of vascular endothelial growth factor inhibitors

bact conjunctivitis tx

warm compress trimethoprim/polymyxin

treatment for chalazion

warm compress. elective excisionif not resolved

hordeolum tx

warm compresses abx ointment (e-mycin or bacitracin) no improvement: I&D

chalazion tx

warm compresses topical erythromycin i&D

treatment for hodeolum

warm compresses, topical antibiotics, incision and drainage if no resolution

sensorineural hearing loss weber and rinne findings

weber: lateralizes to UNaffected ear rinne: AC > BC

conductive hearing loss weber and rinne findings

weber: sound lateralizes to affected ear rinne: BC > AC

which is the most severe type of macular degeneration?

wet (exudative) which causes more rapid and more severe visual loss

When is surgical intervention necessary for retinal detachment?

when a tear occurs

Entropion is . . .

when the eyelid and lashes are turned *inwards*, irritating the conjunctiva

Ectropion is . . .

when the eyelid and lashes are turned *outwards, exposing the conjunctiva*

oral hairy leukoplakia

white patch with a hairy surface on the tongue that cannot be scraped off; associated with EBV; treated with acyclovir

laryngeal leukoplakia

white spots/plaques on the larynx; commonly found in smokers with hoarseness; treated with serial resection

roth spots

white-centered retinal hemorrhages, seen in leukemia & bacterial endocarditis

dentail carie

whitish discoloration at the gingiva margin

posterior bleed in the nose

woodruff's plexus--this is an ER! you will see blood in posterior pharynx posterior pack increases risk for TSS

tx of barotrauma to ear

yawn, swallow, chew gum,

hard exduates

yellow spots with sharp margins often circinate due to lipid or lipoprotein deposits from leaky blood vessels; seen with nonproliferative diabetic retinopathy

pinguecula

yellow, elevated nodule on nasal side of eye (fat and protein)- DOES NOT GROW

pinguecula

yellowish, slightly raised thickening of the sclera; treated with avoiding irritants & lubricating eye drops

blowout fracture = direct trauma to what structure

zygomatic prominence


Ensembles d'études connexes

English File Beginner Common verb phrases 2

View Set

Terminologies Part 2 CONSTRUCTION METHODS AND MANAGEMENT

View Set

Microeconomics Chapter 10 Review

View Set

REGLAS Y REGULACIOES DEL HOGAR CUIDADO DE NINOS INFANTIL Y FAMILIAR (HOME)

View Set

FBLA Computer Problem Solving (TYPES OF PRINTERS)

View Set

Module 4: Oral and Topical Medications

View Set

renaissance to modern art final exam

View Set